You are on page 1of 90

EN BANC SAMMY SALNUNGAN, ANTONIO D.

EMBA, NORMA
[G.R. No. 135385. December 6, 2000] MAPANSAGONOS, ROMEO SALIGA, SR., JERSON P. GERADA,
RENATO T. BAGON, JR., SARING MASALONG, SOLEDAD M.
ISAGANI CRUZ and CESAR EUROPA, petitioners, vs. SECRETARY OF GERARDA, ELIZABETH L. MENDI, MORANTE S. TIWAN, DANILO
ENVIRONMENT AND NATURAL RESOURCES, SECRETARY OF M. MALUDAO, MINORS MARICEL MALID, represented by her father
BUDGET AND MANAGEMENT and CHAIRMAN and CORNELIO MALID, MARCELINO M. LADRA, represented by her
COMMISSIONERS OF THE NATIONAL COMMISSION ON father MONICO D. LADRA, JENNYLYN MALID, represented by her
INDIGENOUS PEOPLES, respondents. father TONY MALID, ARIEL M. EVANGELISTA, represented by her
HON. JUAN M .FLAVIER, HON. PONCIANO BENNAGEN, BAYANI mother LINAY BALBUENA, EDWARD M. EMUY, SR., SUSAN
ASCARRAGA, EDTAMI MANSAYANGAN, BASILIO WANDAG, BOLANIO, OND, PULA BATO BLAAN TRIBAL FARMERS
EVELYN DUNUAN, YAOM TUGAS, ALFREMO CARPIANO, ASSOCIATION, INTER-PEOPLES EXCHANGE, INC. and GREEN
LIBERATO A. GABIN, MATERNIDAD M. COLAS, NARCISA M. FORUM-WESTERN VISAYAS, intervenors.
DALUPINES, BAI KIRAM-CONNIE SATURNO, BAE MLOMO- COMMISSION ON HUMAN RIGHTS, intervenor.
BEATRIZ T. ABASALA, DATU BALITUNGTUNG-ANTONIO D.
LUMANDONG, DATU MANTUMUKAW TEOFISTO SABASALES, IKALAHAN INDIGENOUS PEOPLE and HARIBON FOUNDATION FOR THE
DATU EDUAARDO BANDA, DATU JOEL UNAD, DATU RAMON CONSERVATION OF NATURAL RESOURCES, INC., intervenor.
BAYAAN, TIMUAY JOSE ANOY, TIMUAY MACARIO D. SALACAO,
TIMUAY EDWIN B. ENDING, DATU SAHAMPONG MALANAW VI, RESOLUTION
DATU BEN PENDAO CABIGON, BAI NANAPNAY-LIZA SAWAY,
BAY INAY DAYA-MELINDA S. REYMUNDO, BAI TINANGHAGA PER CURIAM:
HELINITA T. PANGAN, DATU MAKAPUKAW ADOLINO L. SAWAY,
DATU MAUDAYAW-CRISPEN SAWAY, VICKY MAKAY, LOURDES
Petitioners Isagani Cruz and Cesar Europa brought this suit for prohibition and
D. AMOS, GILBERT P. HOGGANG, TERESA GASPAR, MANUEL S.
mandamus as citizens and taxpayers, assailing the constitutionality of certain provisions
ONALAN, MIA GRACE L. GIRON, ROSEMARIE G. PE, BENITO
of Republic Act No. 8371 (R.A. 8371), otherwise known as the Indigenous Peoples
CARINO, JOSEPH JUDE CARANTES, LYNETTE CARANTES-VIVAL,
Rights Act of 1997 (IPRA), and its Implementing Rules and Regulations (Implementing
LANGLEY SEGUNDO, SATUR S. BUGNAY, CARLING DOMULOT,
Rules).
ANDRES MENDIOGRIN, LEOPOLDO ABUGAN, VIRGILIO
CAYETANO, CONCHITA G. DESCAGA, LEVY ESTEVES, ODETTE G. In its resolution of September 29, 1998, the Court required respondents to
ESTEVEZ, RODOLFO C. AGUILAR, MAURO VALONES, PEPE H. comment.[1] In compliance, respondents Chairperson and Commissioners of the National
ATONG, OFELIA T. DAVI, PERFECTO B. GUINOSAO, WALTER N. Commission on Indigenous Peoples (NCIP), the government agency created under the
TIMOL, MANUEL T. SELEN, OSCAR DALUNHAY, RICO O. IPRA to implement its provisions, filed on October 13, 1998 their Comment to the
SULATAN, RAFFY MALINDA, ALFREDO ABILLANOS, JESSIE Petition, in which they defend the constitutionality of the IPRA and pray that the petition
ANDILAB, MIRLANDO H. MANGKULINTAS, SAMIE SATURNO, be dismissed for lack of merit.
ROMEO A. LINDAHAY, ROEL S. MANSANG-CAGAN, PAQUITO S.
LIESES, FILIPE G. SAWAY, HERMINIA S. SAWAY, JULIUS S. On October 19, 1998, respondents Secretary of the Department of Environment and
SAWAY, LEONARDA SAWAY, JIMMY UGYUB, SALVADOR Natural Resources (DENR) and Secretary of the Department of Budget and Management
TIONGSON, VENANCIO APANG, MADION MALID, SUKIM MALID, (DBM) filed through the Solicitor General a consolidated Comment. The Solicitor
NENENG MALID, MANGKATADONG AUGUSTO DIANO, General is of the view that the IPRA is partly unconstitutional on the ground that it grants
JOSEPHINE M. ALBESO, MORENO MALID, MARIO MANGCAL, ownership over natural resources to indigenous peoples and prays that the petition be
FELAY DIAMILING, SALOME P. SARZA, FELIPE P. BAGON, granted in part.
On November 10, 1998, a group of intervenors, composed of Sen. Juan Flavier, one (4) Section 7 which recognizes and enumerates the rights of the indigenous
of the authors of the IPRA, Mr. Ponciano Bennagen, a member of the 1986 Constitutional peoples over the ancestral domains;
Commission, and the leaders and members of 112 groups of indigenous peoples (Flavier,
et. al), filed their Motion for Leave to Intervene. They join the NCIP in defending the (5) Section 8 which recognizes and enumerates the rights of the indigenous
constitutionality of IPRA and praying for the dismissal of the petition. peoples over the ancestral lands;

On March 22, 1999, the Commission on Human Rights (CHR) likewise filed a (6) Section 57 which provides for priority rights of the indigenous peoples in
Motion to Intervene and/or to Appear as Amicus Curiae. The CHR asserts that IPRA is the harvesting, extraction, development or exploration of minerals and
an expression of the principle of parens patriae and that the State has the responsibility to other natural resources within the areas claimed to be their ancestral
protect and guarantee the rights of those who are at a serious disadvantage like domains, and the right to enter into agreements with nonindigenous
indigenous peoples. For this reason it prays that the petition be dismissed. peoples for the development and utilization of natural resources therein
for a period not exceeding 25 years, renewable for not more than 25 years;
On March 23, 1999, another group, composed of the Ikalahan Indigenous People and
and the Haribon Foundation for the Conservation of Natural Resources, Inc. (Haribon, et
al.), filed a motion to Intervene with attached Comment-in-Intervention. They agree with (7) Section 58 which gives the indigenous peoples the responsibility to
the NCIP and Flavier, et al. that IPRA is consistent with the Constitution and pray that maintain, develop, protect and conserve the ancestral domains and
the petition for prohibition and mandamus be dismissed. portions thereof which are found to be necessary for critical watersheds,
mangroves, wildlife sanctuaries, wilderness, protected areas, forest cover
The motions for intervention of the aforesaid groups and organizations were or reforestation.[2]
granted.
Petitioners also content that, by providing for an all-encompassing definition of
Oral arguments were heard on April 13, 1999. Thereafter, the parties and ancestral domains and ancestral lands which might even include private lands found
intervenors filed their respective memoranda in which they reiterate the arguments within said areas, Sections 3(a) and 3(b) violate the rights of private landowners.[3]
adduced in their earlier pleadings and during the hearing.
In addition, petitioners question the provisions of the IPRA defining the powers and
Petitioners assail the constitutionality of the following provisions of the IPRA and jurisdiction of the NCIP and making customary law applicable to the settlement of
its Implementing Rules on the ground that they amount to an unlawful deprivation of the disputes involving ancestral domains and ancestral lands on the ground that these
States ownership over lands of the public domain as well as minerals and other natural provisions violate the due process clause of the Constitution. [4]
resources therein, in violation of the regalian doctrine embodied in Section 2, Article XII
of the Constitution: These provisions are:

(1) Section 3(a) which defines the extent and coverage of ancestral domains, (1) sections 51 to 53 and 59 which detail the process of delineation and
and Section 3(b) which, in turn, defines ancestral lands; recognition of ancestral domains and which vest on the NCIP the sole
authority to delineate ancestral domains and ancestral lands;
(2) Section 5, in relation to section 3(a), which provides that ancestral
domains including inalienable public lands, bodies of water, mineral and (2) Section 52[i] which provides that upon certification by the NCIP that a
other resources found within ancestral domains are private but particular area is an ancestral domain and upon notification to the
community property of the indigenous peoples; following officials, namely, the Secretary of Environment and Natural
Resources, Secretary of Interior and Local Governments, Secretary of
(3) Section 6 in relation to section 3(a) and 3(b) which defines the Justice and Commissioner of the National Development Corporation, the
composition of ancestral domains and ancestral lands; jurisdiction of said officials over said area terminates;
(3) Section 63 which provides the customary law, traditions and practices of Seven (7) voted to dismiss the petition. Justice Kapunan filed an opinion, which the
indigenous peoples shall be applied first with respect to property rights, Chief Justice and Justices Bellosillo, Quisumbing, and Santiago join, sustaining the
claims of ownership, hereditary succession and settlement of land validity of the challenged provisions of R.A. 8371. Justice Puno also filed a separate
disputes, and that any doubt or ambiguity in the interpretation thereof opinion sustaining all challenged provisions of the law with the exception of Section 1,
shall be resolved in favor of the indigenous peoples; Part II, Rule III of NCIP Administrative Order No. 1, series of 1998, the Rules and
Regulations Implementing the IPRA, and Section 57 of the IPRA which he contends
(4) Section 65 which states that customary laws and practices shall be used to should be interpreted as dealing with the large-scale exploitation of natural resources and
resolve disputes involving indigenous peoples; and should be read in conjunction with Section 2, Article XII of the 1987 Constitution. On the
(5) Section 66 which vests on the NCIP the jurisdiction over all claims and other hand, Justice Mendoza voted to dismiss the petition solely on the ground that it
disputes involving rights of the indigenous peoples.[5] does not raise a justiciable controversy and petitioners do not have standing to question
the constitutionality of R.A. 8371.
Finally, petitioners assail the validity of Rule VII, Part II, Section 1 of the NCIP
Administrative Order No. 1, series of 1998, which provides that the administrative Seven (7) other members of the Court voted to grant the petition. Justice
relationship of the NCIP to the Office of the President is characterized as a lateral but Panganiban filed a separate opinion expressing the view that Sections 3 (a)(b), 5, 6, 7
autonomous relationship for purposes of policy and program coordination. They contend (a)(b), 8, and related provisions of R.A. 8371 are unconstitutional. He reserves judgment
that said Rule infringes upon the Presidents power of control over executive departments on the constitutionality of Sections 58, 59, 65, and 66 of the law, which he believes must
under Section 17, Article VII of the Constitution.[6] await the filing of specific cases by those whose rights may have been violated by the
IPRA. Justice Vitug also filed a separate opinion expressing the view that Sections 3(a), 7,
Petitioners pray for the following: and 57 of R.A. 8371 are unconstitutional.Justices Melo, Pardo, Buena, Gonzaga-Reyes,
and De Leon join in the separate opinions of Justices Panganiban and Vitug.
(1) A declaration that Sections 3, 5, 6, 7, 8, 52[I], 57, 58, 59, 63, 65 and 66
and other related provisions of R.A. 8371 are unconstitutional and invalid; As the votes were equally divided (7 to 7) and the necessary majority was not
obtained, the case was redeliberated upon. However, after redeliberation, the voting
(2) The issuance of a writ of prohibition directing the Chairperson and remained the same.Accordingly, pursuant to Rule 56, Section 7 of the Rules of Civil
Commissioners of the NCIP to cease and desist from implementing the Procedure, the petition is DISMISSED.
assailed provisions of R.A. 8371 and its Implementing Rules;
Attached hereto and made integral parts thereof are the separate opinions of
(3) The issuance of a writ of prohibition directing the Secretary of the Justices Puno, Vitug, Kapunan, Mendoza, and Panganiban.
Department of Environment and Natural Resources to cease and desist
from implementing Department of Environment and Natural Resources SO ORDERED.
Circular No. 2, series of 1998;
Davide, Jr., C.J., Bellosillo, Melo, Quisumbing, Pardo, Buena, Gonzaga-Reyes,
(4) The issuance of a writ of prohibition directing the Secretary of Budget and Ynares-Santiago, and De Leon, Jr., JJ., concur.
Management to cease and desist from disbursing public funds for the Puno, Vitug, Kapunan, Mendoza and Panganiban JJ., see separate opinion
implementation of the assailed provisions of R.A. 8371; and
(5) The issuance of a writ of mandamus commanding the Secretary of
Environment and Natural Resources to comply with his duty of carrying
out the States constitutional mandate to control and supervise the
exploration, development, utilization and conservation of Philippine
natural resources.[7]
After due deliberation on the petition, the members of the Court voted as follows:
SEPARATE OPINION D. The Philippine Constitutions
PUNO, J.:
PRECIS II. The Indigenous Peoples Rights Act (IPRA).

A classic essay on the utility of history was written in 1874 by Friedrich A. Indigenous Peoples
Nietzsche entitled "On the Uses and Disadvantages of History for Life."
Expounding on Nietzsche's essay, Judge Richard Posner[1] wrote:[2] 1. Indigenous Peoples: Their History

"Law is the most historically oriented, or if you like the most backward-looking, 2. Their Concept of Land
the most 'past-dependent,' of the professions. It venerates tradition, precedent,
pedigree, ritual, custom, ancient practices, ancient texts, archaic terminology,
III. The IPRA is a Novel Piece of Legislation.
maturity, wisdom, seniority, gerontocracy, and interpretation conceived of as a
method of recovering history. It is suspicious of innovation, discontinuities,
'paradigm shifts,' and the energy and brashness of youth. These ingrained A. Legislative History
attitudes are obstacles to anyone who wants to re-orient law in a more
pragmatic direction. But, by the same token, pragmatic jurisprudence must IV. The Provisions of the IPRA Do Not Contravene the Constitution.
come to terms with history.
A. Ancestral domains and ancestral lands are the private
When Congress enacted the Indigenous Peoples Rights Act (IPRA), it property of indigenous peoples and do not constitute part of the land
introduced radical concepts into the Philippine legal system which appear to of the public domain.
collide with settled constitutional and jural precepts on state ownership of land
and other natural resources. The sense and subtleties of this law cannot be 1. The right to ancestral domains and ancestral lands: how
appreciated without considering its distinct sociology and the labyrinths of its acquired
history. This Opinion attempts to interpret IPRA by discovering its soul
shrouded by the mist of our history. After all, the IPRA was enacted by 2. The concept of native title
Congress not only to fulfill the constitutional mandate of protecting the
indigenous cultural communities' right to their ancestral land but more (a) Cario v. Insular Government
importantly, to correct a grave historical injustice to our indigenous people.
(b) Indian Title to land
This Opinion discusses the following:
(c) Why the Cario doctrine is unique
I. The Development of the Regalian Doctrine in the Philippine Legal
System. 3. The option of securing a torrens title to the ancestral land
A. The Laws of the Indies B. The right of ownership and possession by the ICCs/IPs to their
ancestral domains is a limited form of ownership and does not
B. Valenton v. Murciano include the right to alienate the same.

C. The Public Land Acts and the Torrens System 1. The indigenous concept of ownership and customary law
C. Sections 7 (a), 7 (b) and 57 of the IPRA do not violate the Regalian Cedulas. The Laws of the Indies, i.e., more specifically, Law 14, Title 12, Book
Doctrine enshrined in Section 2, Article XII of the 1987 4 of the Novisima Recopilacion de Leyes de las Indias, set the policy of the
Constitution. Spanish Crown with respect to the Philippine Islands in the following manner:

1. The rights of ICCs/IPs over their ancestral domains and lands "We, having acquired full sovereignty over the Indies, and all lands, territories,
and possessions not heretofore ceded away by our royal predecessors, or by us,
2. The right of ICCs/IPs to develop lands and natural resources or in our name, still pertaining to the royal crown and patrimony, it is our will
within the ancestral domains does not deprive the State of that all lands which are held without proper and true deeds of grant be restored
ownership over the natural resources, control and to us as they belong to us, in order that after reserving before all what to us or
supervision in their development and exploitation. to our viceroys, audiencias, and governors may seem necessary for public
squares, ways, pastures, and commons in those places which are peopled,
(a) Section 1, Part II, Rule III of the Implementing Rules goes taking into consideration not only their present condition, but also their future
beyond the parameters of Section 7(a) of the law on and their probable increase, and after distributing to the natives what may be
ownership of ancestral domains and is ultra vires. necessary for tillage and pasturage, confirming them in what they now have and
giving them more if necessary, all the rest of said lands may remain free and
unencumbered for us to dispose of as we may wish.
(b) The small-scale utilization of natural resources in Section
7 (b) of the IPRA is allowed under Paragraph 3,
Section 2, Article XII of the 1987 Consitution. We therefore order and command that all viceroys and presidents of pretorial
courts designate at such time as shall to them seem most expedient, a suitable
period within which all possessors of tracts, farms, plantations, and estates shall
(c) The large-scale utilization of natural resources in Section
exhibit to them and to the court officers appointed by them for this purpose,
57 of the IPRA may be harmonized with Paragraphs
their title deeds thereto. And those who are in possession by virtue of proper
1 and 4, Section 2, Article XII of the 1987
deeds and receipts, or by virtue of just prescriptive right shall be protected, and
Constitution.
all the rest shall be restored to us to be disposed of at our will."[4]
V. The IPRA is a Recognition of Our Active Participation in the
The Philippines passed to Spain by virtue of "discovery" and conquest.
International Indigenous Movement.
Consequently, all lands became the exclusive patrimony and dominion of the
Spanish Crown. The Spanish Government took charge of distributing the lands
DISCUSSION by issuing royal grants and concessions to Spaniards, both military and
civilian.[5] Private land titles could only be acquired from the government either
I. THE DEVELOPMENT OF THE REGALIAN DOCTRINE IN THE by purchase or by the various modes of land grant from the Crown.[6]
PHILIPPINE LEGAL SYSTEM.
The Laws of the Indies were followed by the Ley Hipotecaria, or the
A. The Laws of the Indies Mortgage Law of 1893.[7] The Spanish Mortgage Law provided for the
systematic registration of titles and deeds as well as possessory claims. The law
The capacity of the State to own or acquire property is the state's power
sought to register and tax lands pursuant to the Royal Decree of 1880. The
of dominium.[3] This was the foundation for the early Spanish decrees
Royal Decree of 1894, or the "Maura Law," was partly an amendment of the
embracing the feudal theory of jura regalia. The "Regalian Doctrine" or jura
Mortgage Law as well as the Laws of the Indies, as already amended by
regalia is a Western legal concept that was first introduced by the Spaniards
previous orders and decrees.[8] This was the last Spanish land law promulgated
into the country through the Laws of the Indies and the Royal
in the Philippines. It required the "adjustment" or registration of all agricultural Quoting the preamble of Law 14, Title 12, Book 4 of the Recopilacion de
lands, otherwise the lands shall revert to the state. Leyes de las Indias, the court interpreted it as follows:

Four years later, by the Treaty of Paris of December 10, 1898, Spain "In the preamble of this law there is, as is seen, a distinct statement that all
ceded to the government of the United States all rights, interests and claims those lands belong to the Crown which have not been granted by Philip, or in
over the national territory of the Philippine Islands. In 1903, the United States his name, or by the kings who preceded him. This statement excludes the idea
colonial government, through the Philippine Commission, passed Act No. 926, that there might be lands not so granted, that did not belong to the king. It
the first Public Land Act. excludes the idea that the king was not still the owner of all ungranted
lands, because some private person had been in the adverse occupation of
B. Valenton v. Murciano them. By the mandatory part of the law all the occupants of the public lands are
required to produce before the authorities named, and within a time to be fixed
In 1904, under the American regime, this Court decided the case by them, their title papers. And those who had good title or showed prescription
of Valenton v. Murciano.[9] were to be protected in their holdings. It is apparent that it was not the intention
of the law that mere possession for a length of time should make the possessors
Valenton resolved the question of which is the better basis for ownership the owners of the land possessed by them without any action on the part of the
of land: long-time occupation or paper title. Plaintiffs had entered into peaceful authorities."[12]
occupation of the subject land in 1860. Defendant's predecessor-in-interest, on
the other hand, purchased the land from the provincial treasurer of Tarlac in The preamble stated that all those lands which had not been granted by
1892. The lower court ruled against the plaintiffs on the ground that they had Philip, or in his name, or by the kings who preceded him, belonged to the
lost all rights to the land by not objecting to the administrative sale. Plaintiffs Crown.[13] For those lands granted by the king, the decree provided for a system
appealed the judgment, asserting that their 30-year adverse possession, as an of assignment of such lands. It also ordered that all possessors of agricultural
extraordinary period of prescription in the Partidas and the Civil Code, had land should exhibit their title deed, otherwise, the land would be restored to the
given them title to the land as against everyone, including the State; and that the Crown.[14]
State, not owning the land, could not validly transmit it.
The Royal Cedula of October 15, 1754 reinforced the Recopilacion when
The Court, speaking through Justice Willard, decided the case on the it ordered the Crown's principal subdelegate to issue a general order directing
basis of "those special laws which from earliest time have regulated the the publication of the Crown's instructions:
disposition of the public lands in the colonies."[10] The question posed by the
Court was: "Did these special laws recognize any right of prescription as "x x x to the end that any and all persons who, since the year 1700, and
against the State as to these lands; and if so, to what extent was it recognized?" up to the date of the promulgation and publication of said order, shall
have occupied royal lands, whether or not x x x cultivated or tenanted,
Prior to 1880, the Court said, there were no laws specifically providing may x x x appear and exhibit to said subdelegates the titles and patents
for the disposition of land in the Philippines. However, it was understood that by virtue of which said lands are occupied. x x x. Said subdelegates will
in the absence of any special law to govern a specific colony, the Laws of the at the same time warn the parties interested that in case of their failure to
Indies would be followed. Indeed, in the Royal Order of July 5, 1862, it was present their title deeds within the term designated, without a just and
decreed that until regulations on the subject could be prepared, the authorities valid reason therefor, they will be deprived of and evicted from their
of the Philippine Islands should follow strictly the Laws of the Indies, lands, and they will be granted to others."[15]
the Ordenanza of the Intendentes of 1786, and the Royal Cedula of 1754.[11]
On June 25, 1880, the Crown adopted regulations for the adjustment of
lands "wrongfully occupied" by private individuals in the Philippine
Islands. Valenton construed these regulations together with contemporaneous States.[20] The term "public land" referred to all lands of the public domain
legislative and executive interpretations of the law, and concluded that whose title still remained in the government and are thrown open to private
plaintiffs' case fared no better under the 1880 decree and other laws which appropriation and settlement,[21] and excluded the patrimonial property of the
followed it, than it did under the earlier ones. Thus as a general doctrine, the government and the friar lands.[22]
Court stated:
Act No. 926 was superseded in 1919 by Act 2874, the second Public
"While the State has always recognized the right of the occupant to a deed if he Land Act. This new law was passed under the Jones Law. It was more
proves a possession for a sufficient length of time, yet it has always insisted comprehensive in scope but limited the exploitation of agricultural lands to
that he must make that proof before the proper administrative officers, Filipinos and Americans and citizens of other countries which gave Filipinos
and obtain from them his deed, and until he did that the State remained the same privileges.[23] After the passage of the 1935 Constitution, Act 2874
the absolute owner."[16] was amended in 1936 by Commonwealth Act No. 141. Commonwealth Act
No. 141 remains the present Public Land Law and it is essentially the same as
In conclusion, the Court ruled: "We hold that from 1860 to 1892 there was no Act 2874. The main difference between the two relates to the transitory
law in force in these Islands by which the plaintiffs could obtain the ownership provisions on the rights of American citizens and corporations during the
of these lands by prescription, without any action by the State."[17] Valenton Commonwealth period at par with Filipino citizens and corporations. [24]
had no rights other than those which accrued to mere possession. Murciano, on
the other hand, was deemed to be the owner of the land by virtue of the grant Grants of public land were brought under the operation of the
by the provincial secretary. In effect, Valenton upheld the Spanish concept of Torrens system under Act 496, or the Land Registration Law of
state ownership of public land. 1903. Enacted by the Philippine Commission, Act 496 placed all public and
private lands in the Philippines under the Torrens system. The law is said to be
As a fitting observation, the Court added that "[t]he policy pursued by almost a verbatim copy of the Massachussetts Land Registration Act of
the Spanish Government from earliest times, requiring settlers on the 1898,[25] which, in turn, followed the principles and procedure of the Torrens
public lands to obtain title deeds therefor from the State, has been system of registration formulated by Sir Robert Torrens who patterned it after
continued by the American Government in Act No. 926."[18] the Merchant Shipping Acts in South Australia. The Torrens system requires
that the government issue an official certificate of title attesting to the fact that
C. The Public Land Acts and the Torrens System the person named is the owner of the property described therein, subject to such
liens and encumbrances as thereon noted or the law warrants or
Act No. 926, the first Public Land Act, was passed in pursuance of the reserves.[26] The certificate of title is indefeasible and imprescriptible and all
provisions of the the Philippine Bill of 1902. The law governed the disposition claims to the parcel of land are quieted upon issuance of said certificate. This
of lands of the public domain. It prescribed rules and regulations for the system highly facilitates land conveyance and negotiation.[27]
homesteading, selling, and leasing of portions of the public domain of the
Philippine Islands, and prescribed the terms and conditions to enable persons to D. The Philippine Constitutions
perfect their titles to public lands in the Islands. It also provided for the
"issuance of patents to certain native settlers upon public lands," for the The Regalian doctrine was enshrined in the 1935 Constitution. One of
establishment of town sites and sale of lots therein, for the completion of the fixed and dominating objectives of the 1935 Constitutional Convention was
imperfect titles, and for the cancellation or confirmation of Spanish concessions the nationalization and conservation of the natural resources of the
and grants in the Islands." In short, the Public Land Act operated on the country.[28] There was an overwhelming sentiment in the Convention in
assumption that title to public lands in the Philippine Islands remained in the favor of the principle of state ownership of natural resources and the
government;[19] and that the government's title to public land sprung from the adoption of the Regalian doctrine.[29] State ownership of natural resources
Treaty of Paris and other subsequent treaties between Spain and the United was seen as a necessary starting point to secure recognition of the state's power
to control their disposition, exploitation, development, or utilization.[30] The The 1987 Constitution reaffirmed the Regalian doctrine in Section 2 of
delegates to the Constitutional Convention very well knew that the concept of Article XII on "National Economy and Patrimony," to wit:
State ownership of land and natural resources was introduced by the Spaniards,
however, they were not certain whether it was continued and applied by the "Sec. 2. All lands of the public domain, waters, minerals, coal, petroleum,
Americans. To remove all doubts, the Convention approved the provision in the and other mineral oils, all forces of potential energy, fisheries, forests or
Constitution affirming the Regalian doctrine.[31] timber, wildlife, flora and fauna, and other natural resources are owned by
the State. With the exception of agricultural lands, all other natural
Thus, the 1935 Constitution, in Section 1 of Article XIII on resources shall not be alienated. The exploration, development and
"Conservation and Utilization of Natural Resources," reads as follows: utilization of natural resources shall be under the full control and
supervision of the State. The State may directly undertake such activities
"Sec. 1. All agricultural, timber, and mineral lands of the public domain, or it may enter into co-production, joint venture, or production-sharing
waters, minerals, coal, petroleum, and other mineral oils, all forces of agreements with Filipino citizens, or corporations or associations at least
potential energy, and other natural resources of the Philippines belong to sixty per centum of whose capital is owned by such citizens. Such
the State, and their disposition, exploitation, development, or utilization agreements may be for a period not exceeding twenty-five years, renewable for
shall be limited to citizens of the Philippines, or to corporations or not more than twenty-five years, and under such terms and conditions as may
associations at least sixty per centum of the capital of which is owned by be provided by law. In cases of water rights for irrigation, water supply,
such citizens, subject to any existing right, grant, lease, or concession at the fisheries, or industrial uses other than the development of water power,
time of the inauguration of the Government established under this beneficial use may be the measure and limit of the grant.
Constitution. Natural resources, with the exception of public agricultural
land, shall not be alienated,and no license, concession, or lease for the x x x."
exploitation, development, or utilization of any of the natural resources shall be
granted for a period exceeding twenty-five years, except as to water rights for Simply stated, all lands of the public domain as well as all natural
irrigation, water supply, fisheries, or industrial uses other than the development resources enumerated therein, whether on public or private land, belong to the
of water power, in which cases beneficial use may be the measure and the limit State. It is this concept of State ownership that petitioners claim is being
of the grant." violated by the IPRA.

The 1973 Constitution reiterated the Regalian doctrine in Section 8, Article XIV II. THE INDIGENOUS PEOPLES RIGHTS ACT.
on the "National Economy and the Patrimony of the Nation," to wit:
"Sec. 8. All lands of the public domain, waters, minerals, coal, petroleum Republic Act No. 8371 is entitled "An Act to Recognize, Protect and
and other mineral oils, all forces of potential energy, fisheries, wildlife, and Promote the Rights of Indigenous Cultural Communities/ Indigenous Peoples,
other natural resources of the Philippines belong to the State. With the Creating a National Commission on Indigenous Peoples, Establishing
exception of agricultural, industrial or commercial, residential, and Implementing Mechanisms, Appropriating Funds Therefor, and for Other
resettlement lands of the public domain, natural resources shall not be Purposes." It is simply known as "The Indigenous Peoples Rights Act of
alienated, and no license, concession, or lease for the exploration, 1997" or the IPRA.
development, exploitation, or utilization of any of the natural resources
shall be granted for a period exceeding twenty-five years, renewable for The IPRA recognizes the existence of the indigenous cultural
not more than twenty-five years, except as to water rights for irrigation, water communities or indigenous peoples (ICCs/IPs) as a distinct sector in
supply, fisheries, or industrial uses other than the development of water power, Philippine society. It grants these people the ownership and possession of
in which cases beneficial use may be the measure and the limit of the grant." their ancestral domains and ancestral lands, and defines the extent of these
lands and domains. The ownership given is the indigenous concept of including Mindoro, Palawan, Romblon, Panay and the rest of the Visayas;
ownership under customary law which traces its origin to native title. Northern and Western Mindanao; Southern and Eastern Mindanao; and Central
Mindanao.[37] The NCIP took over the functions of the Office for Northern
Other rights are also granted the ICCs/IPs, and these are: Cultural Communities and the Office for Southern Cultural Communities
created by former President Corazon Aquino which were merged under a
- the right to develop lands and natural resources; revitalized structure.[38]

- the right to stay in the territories; Disputes involving ICCs/IPs are to be resolved under customary laws
and practices. When still unresolved, the matter may be brought to the NCIP,
which is granted quasi-judicial powers.[39] The NCIP's decisions may be
- the right in case of displacement;
appealed to the Court of Appeals by a petition for review.
- the right to safe and clean air and water;
Any person who violates any of the provisions of the Act such as, but not
limited to, unauthorized and/or unlawful intrusion upon ancestral lands and
- the right to claim parts of reservations; domains shall be punished in accordance with customary laws or imprisoned
from 9 months to 12 years and/or fined from P100,000.00 to P500,000.00 and
- the right to resolve conflict;[32] obliged to pay damages.[40]

- the right to ancestral lands which include A. Indigenous Peoples

a. the right to transfer land/property to/among members of the same The IPRA is a law dealing with a specific group of people, i.e., the
ICCs/IPs, subject to customary laws and traditions of the community Indigenous Cultural Communities (ICCs) or the Indigenous Peoples (IPs). The
concerned; term "ICCs" is used in the 1987 Constitution while that of "IPs" is the
contemporary international language in the International Labor Organization
b. the right to redemption for a period not exceeding 15 years from date (ILO) Convention 169[41] and the United Nations (UN) Draft Declaration on the
of transfer, if the transfer is to a non-member of the ICC/IP and is tainted Rights of Indigenous Peoples.[42]
by vitiated consent of the ICC/IP, or if the transfer is for an
unconscionable consideration.[33] ICCs/IPs are defined by the IPRA as:

Within their ancestral domains and ancestral lands, the ICCs/IPs are "Sec. 3 [h]. Indigenous Cultural Communities/ Indigenous Peoples-- refer to a
given the right to self-governance and empowerment,[34] social justice and group of people or homogeneous societies identified by self-ascription and
human rights,[35] the right to preserve and protect their culture, traditions, ascription by others, who have continuously lived as organized community on
institutions and community intellectual rights, and the right to develop their communally bounded and defined territory, and who have, under claims of
own sciences and technologies.[36] ownership since time immemorial, occupied, possessed and utilized such
territories, sharing common bonds of language, customs, traditions and other
To carry out the policies of the Act, the law created the National distinctive cultural traits, or who have, through resistance to political, social
Commission on Indigenous Peoples (NCIP). The NCIP is an independent and cultural inroads of colonization, non-indigenous religions and cultures,
agency under the Office of the President and is composed of seven (7) became historically differentiated from the majority of Filipinos. ICCs/IPs shall
Commissioners belonging to ICCs/IPs from each of the ethnographic areas-- likewise include peoples who are regarded as indigenous on account of their
Region I and the Cordilleras; Region II; the rest of Luzon; Island groups descent from the populations which inhabited the country, at the time of
conquest or colonization, or at the time of inroads of non-indigenous religions 4. In Region V-- Aeta of Camarines Norte and Camarines Sur; Aeta-Abiyan,
and cultures, or the establishment of present state boundaries, who retain some Isarog, and Kabihug of Camarines Norte; Agta, and Mayon of Camarines Sur;
or all of their own social, economic, cultural and political institutions, but who Itom of Albay, Cimaron of Sorsogon; and the Pullon of Masbate and Camarines
may have been displaced from their traditional domains or who may have Sur.
resettled outside their ancestral domains."
5. In Region VI-- Ati of Negros Occidental, Iloilo and Antique, Capiz; the
Indigenous Cultural Communities or Indigenous Peoples refer to a group Magahat of Negros Occidental; the Corolano and Sulod.
of people or homogeneous societies who have continuously lived as an
organized community on communally bounded and defined 6. In Region VII-- Magahat of Negros Oriental and Eskaya of Bohol.
territory. These groups of people have actually occupied, possessed and
utilized their territories under claim of ownership since time immemorial. They 7. In Region IX-- the Badjao numbering about 192,000 in Tawi-Tawi,
share common bonds of language, customs, traditions and other distinctive Zamboanga del Sur; the Kalibugan of Basilan, the Samal, Subanon and Yakat.
cultural traits, or, they, by their resistance to political, social and cultural
inroads of colonization, non-indigenous religions and cultures, became
8. Region X-- Numbering 1.6 million in Region X alone, the IPs are: the
historically differentiated from the Filipino majority. ICCs/IPs also include
Banwaon, Bukidnon, Matigsalog, Talaanding of Bukidnon; the Camiguin of
descendants of ICCs/IPs who inhabited the country at the time of conquest or
Camiguin Island; the Higa-unon of Agusan del Norte, Agusan del Sur,
colonization, who retain some or all of their own social, economic, cultural and
Bukidnon and Misamis Occidental; the Tigwahanon of Agusan del Sur,
political institutions but who may have been displaced from their traditional
Misamis Oriental and and Misamis Occidental, the Manobo of the Agusan
territories or who may have resettled outside their ancestral domains.
provinces, and the Umayamnon of Agusan and Bukidnon.
1. Indigenous Peoples: Their History
9. In Region XI-- There are about 1,774,065 IPs in Region XI. They are tribes
of the Dibabaon, Mansaka of Davao del Norte; B'laan, Kalagan, Langilad,
Presently, Philippine indigenous peoples inhabit the interiors and T'boli and Talaingod of Davao del Sur; Mamamanua of Surigao del Sur;
mountains of Luzon, Mindanao, Mindoro, Negros, Samar, Leyte, and the Mandaya of the Surigao provinces and Davao Oriental; Manobo Blit of South
Palawan and Sulu group of islands.They are composed of 110 tribes and are as Cotabato; the Mangguangon of Davao and South Cotabato; Matigsalog of
follows: Davao del Norte and Del Sur; Tagakaolo, Tasaday and Ubo of South Cotabato;
and Bagobo of Davao del sur and South Cotabato.
1. In the Cordillera Autonomous Region-- Kankaney, Ibaloi, Bontoc, Tinggian
or Itneg, Ifugao, Kalinga, Yapayao, Aeta or Agta or Pugot, and Bago of Ilocos 10. In Region XII-- Ilianen, Tiruray, Maguindanao, Maranao, Tausug,
Norte and Pangasinan; Ibanag of Isabela, Cagayan; Ilongot of Quirino and Yakan/Samal, and Iranon.[43]
Nueva Vizcaya; Gaddang of Quirino, Nueva Vizcaya, Itawis of Cagayan;
Ivatan of Batanes, Aeta of Cagayan, Quirino and Isabela.
How these indigenous peoples came to live in the Philippines goes back to
as early as 25,000 to 30,000 B.C.
2. In Region III-- Aetas.
Before the time of Western contact, the Philippine archipelago was
3. In Region IV-- Dumagats of Aurora, Rizal; Remontado of Aurora, Rizal, peopled largely by the Negritos, Indonesians and Malays.[44] The strains from
Quezon; Alangan or Mangyan, Batangan, Buid or Buhid, Hanunuo and Iraya of these groups eventually gave rise to common cultural features which became
Oriental and Occidental Mindoro; Tadyawan of Occidental Mindoro; Cuyonon, the dominant influence in ethnic reformulation in the archipelago. Influences
Palawanon, Tagbanua and Tao't bato of Palawan. from the Chinese and Indian civilizations in the third or fourth millenium B.C.
augmented these ethnic strains. Chinese economic and socio-cultural influences Laws were either customary or written. Customary laws were
came by way of Chinese porcelain, silk and traders. Indian influence found handed down orally from generation to generation and constituted the
their way into the religious-cultural aspect of pre-colonial society.[45] bulk of the laws of the barangay. They were preserved in songs and chants
and in the memory of the elder persons in the community. [54] The written laws
The ancient Filipinos settled beside bodies of water. Hunting and food were those that the chieftain and his elders promulgated from time to time as
gathering became supplementary activities as reliance on them was reduced by the necessity arose.[55] The oldest known written body of laws was the Maragtas
fishing and the cultivation of the soil.[46] From the hinterland, coastal, and Code by Datu Sumakwel at about 1250 A.D. Other old codes are the Muslim
riverine communities, our ancestors evolved an essentially homogeneous Code of Luwaran and the Principal Code of Sulu.[56] Whether customary or
culture, a basically common way of life where nature was a primary written, the laws dealt with various subjects, such as inheritance, divorce, usury,
factor. Community life throughout the archipelago was influenced by, and loans, partnership, crime and punishment, property rights, family relations and
responded to, common ecology. The generally benign tropical climate and the adoption. Whenever disputes arose, these were decided peacefully through a
largely uniform flora and fauna favored similarities, not differences. [47] Life court composed by the chieftain as "judge" and the barangay elders as "jury."
was essentially subsistence but not harsh.[48] Conflicts arising between subjects of different barangays were resolved by
arbitration in which a board composed of elders from neutral barangays acted
The early Filipinos had a culture that was basically Malayan in structure as arbiters.[57]
and form. They had languages that traced their origin to the Austronesian
parent-stock and used them not only as media of daily communication but also Baranganic society had a distinguishing feature: the absence of
as vehicles for the expression of their literary moods.[49] They fashioned private property in land. The chiefs merely administered the lands in the
concepts and beliefs about the world that they could not see, but which they name of the barangay. The social order was an extension of the family with
sensed to be part of their lives.[50] They had their own religion and religious chiefs embodying the higher unity of the community. Each individual, therefore,
beliefs. They believed in the immortality of the soul and life after death. Their participated in the community ownership of the soil and the instruments of
rituals were based on beliefs in a ranking deity whom they called Bathalang production as a member of the barangay.[58] This ancient communalism was
Maykapal, and a host of other deities, in the environmental spirits and in soul practiced in accordance with the concept of mutual sharing of resources so that
spirits. The early Filipinos adored the sun, the moon, the animals and birds, for no individual, regardless of status, was without sustenance. Ownership of land
they seemed to consider the objects of Nature as something to be was non-existent or unimportant and the right of usufruct was what
respected. They venerated almost any object that was close to their daily life, regulated the development of lands.[59] Marine resources and fishing grounds
indicating the importance of the relationship between man and the object of were likewise free to all. Coastal communities depended for their economic
nature.[51] welfare on the kind of fishing sharing concept similar to those in land
communities.[60] Recognized leaders, such as the chieftains and elders, by virtue
The unit of government was the "barangay," a term that derived its of their positions of importance, enjoyed some economic privileges and
meaning from the Malay word "balangay," meaning, a boat, which transported benefits. But their rights, related to either land and sea, were subject to their
them to these shores.[52] The barangay was basically a family-based community responsibility to protect the communities from danger and to provide them with
and consisted of thirty to one hundred families. Each barangay was different the leadership and means of survival.[61]
and ruled by a chieftain called a "dato." It was the chieftain's duty to rule and
govern his subjects and promote their welfare and interests. A chieftain had Sometime in the 13th century, Islam was introduced to the
wide powers for he exercised all the functions of government. He was the archipelago in Maguindanao. The Sultanate of Sulu was established and
executive, legislator and judge and was the supreme commander in time of claimed jurisdiction over territorial areas represented today by Tawi-tawi, Sulu,
war.[53] Palawan, Basilan and Zamboanga. Four ethnic groups were within this
jurisdiction: Sama, Tausug, Yakan and Subanon.[62] The Sultanate of
Maguindanao spread out from Cotabato toward Maranao territory, now Lanao now declared to be crown lands or realengas, belonging to the Spanish
del Norte and Lanao del Sur.[63] king. It was from the realengas that land grants were made to non-
Filipinos.[72]
The Muslim societies evolved an Asiatic form of feudalism where
land was still held in common but was private in use. This is clearly The abrogation of the Filipinos' ancestral rights in land and the
indicated in the Muslim Code of Luwaran. The Code contains a provision on introduction of the concept of public domain were the most immediate
the lease of cultivated lands. It, however, has no provision for the acquisition, fundamental results of Spanish colonial theory and law.[73] The concept
transfer, cession or sale of land.[64] that the Spanish king was the owner of everything of value in the Indies or
colonies was imposed on the natives, and the natives were stripped of their
The societies encountered by Magellan and Legaspi therefore were ancestral rights to land.[74]
primitive economies where most production was geared to the use of the
producers and to the fulfillment of kinship obligations. They were not Increasing their foothold in the Philippines, the Spanish colonialists, civil
economies geared to exchange and profit.[65] Moreover, the family basis of and religious, classified the Filipinos according to their religious practices and
barangay membership as well as of leadership and governance worked to beliefs, and divided them into three types . First were the Indios, the
splinter the population of the islands into numerous small and separate Christianized Filipinos, who generally came from the lowland
communities.[66] populations. Second, were the Moros or the Muslim communities, and third,
were the infieles or the indigenous communities.[75]
When the Spaniards settled permanently in the Philippines in 1565,
they found the Filipinos living in barangay settlements scattered along The Indio was a product of the advent of Spanish culture. This class was
water routes and river banks. One of the first tasks imposed on the favored by the Spaniards and was allowed certain status although below the
missionaries and the encomenderos was to collect all scattered Filipinos Spaniards. The Moros and infieles were regarded as the lowest classes.[76]
together in a reduccion.[67] As early as 1551, the Spanish government assumed
an unvarying solicitous attitude towards the natives.[68] The Spaniards regarded The Moros and infieles resisted Spanish rule and Christianity. The
it a sacred "duty to conscience and humanity to civilize these less fortunate Moros were driven from Manila and the Visayas to Mindanao; while the
people living in the obscurity of ignorance" and to accord them the "moral and infieles, to the hinterlands. The Spaniards did not pursue them into the deep
material advantages" of community life and the "protection and vigilance interior. The upland societies were naturally outside the immediate concern of
afforded them by the same laws."[69] Spanish interest, and the cliffs and forests of the hinterlands were difficult and
inaccessible, allowing the infieles, in effect, relative security.[77] Thus,
The Spanish missionaries were ordered to establish pueblos where the the infieles, which were peripheral to colonial administration, were not only
church and convent would be constructed. All the new Christian converts were able to preserve their own culture but also thwarted the Christianization process,
required to construct their houses around the church and the unbaptized were separating themselves from the newly evolved Christian community. [78] Their
invited to do the same.[70] With the reduccion, the Spaniards attempted to own political, economic and social systems were kept constantly alive and
"tame" the reluctant Filipinos through Christian indoctrination using vibrant.
the convento/casa real/plaza complex as focal point. The reduccion, to the
Spaniards, was a "civilizing" device to make the Filipinos law-abiding citizens The pro-Christian or pro-Indio attitude of colonialism brought about a
of the Spanish Crown, and in the long run, to make them ultimately adopt generally mutual feeling of suspicion, fear, and hostility between the Christians
Hispanic culture and civilization.[71] on the one hand and the non-Christians on the other. Colonialism tended to
divide and rule an otherwise culturally and historically related populace
All lands lost by the old barangays in the process of pueblo through a colonial system that exploited both the virtues and vices of the
organization as well as all lands not assigned to them and the pueblos, were Filipinos.[79]
President McKinley, in his instructions to the Philippine Commission In 1957, the Philippine Congress passed R.A. No. 1888, an "Act to
of April 7, 1900, addressed the existence of the infieles: effectuate in a more rapid and complete manner the economic, social, moral
and political advancement of the non-Christian Filipinos or national cultural
"In dealing with the uncivilized tribes of the Islands, the Commission minorities and to render real, complete, and permanent the integration of all
should adopt the same course followed by Congress in permitting the said national cultural minorities into the body politic, creating the Commission
tribes of our North American Indians to maintain their tribal organization on National Integration charged with said functions." The law called for
and government, and under which many of those tribes are now living in a policy of integration of indigenous peoples into the Philippine mainstream
peace and contentment, surrounded by civilization to which they are unable or and for this purpose created the Commission on National
unwilling to conform. Such tribal government should, however, be subjected to Integration (CNI).[84] The CNI was given, more or less, the same task as the
wise and firm regulation; and, without undue or petty interference, constant and BNCT during the American regime. The post-independence policy of
active effort should be exercised to prevent barbarous practices and introduce integration was like the colonial policy of assimilation understood in the
civilized customs."[80] context of a guardian-ward relationship.[85]

Placed in an alternative of either letting the natives alone or guiding them in the The policy of assimilation and integration did not yield the desired
path of civilization, the American government chose "to adopt the latter result. Like the Spaniards and Americans, government attempts at
measure as one more in accord with humanity and with the national integration met with fierce resistance. Since World War II, a tidal wave of
conscience."[81] Christian settlers from the lowlands of Luzon and the Visayas swamped the
highlands and wide open spaces in Mindanao.[86]Knowledge by the settlers of
The Americans classified the Filipinos into two: the Christian the Public Land Acts and the Torrens system resulted in the titling of
Filipinos and the non-Christian Filipinos. The term "non-Christian" referred several ancestral lands in the settlers' names. With government initiative
not to religious belief, but to a geographical area, and more directly, "to natives and participation, this titling displaced several indigenous peoples from
of the Philippine Islands of a low grade of civilization, usually living in tribal their lands. Worse, these peoples were also displaced by projects undertaken
relationship apart from settled communities."[82] by the national government in the name of national development. [87]

Like the Spaniards, the Americans pursued a policy of It was in the 1973 Constitution that the State adopted the following
assimilation. In 1903, they passed Act No. 253 creating the Bureau of Non- provision:
Christian Tribes (BNCT). Under the Department of the Interior, the BNCT's
primary task was to conduct ethnographic research among unhispanized "The State shall consider the customs, traditions, beliefs, and interests of
Filipinos, including those in Muslim Mindanao, with a "special view to national cultural communities in the formulation and implementation of State
determining the most practicable means for bringing about their advancement policies."[88]
in civilization and prosperity." The BNCT was modeled after the bureau
dealing with American Indians. The agency took a keen anthropological For the first time in Philippine history, the "non-Christian tribes" or
interest in Philippine cultural minorities and produced a wealth of valuable the "cultural minorities" were addressed by the highest law of the
materials about them.[83] Republic, and they were referred to as "cultural communities." More
importantly this time, their "uncivilized" culture was given some recognition
The 1935 Constitution did not carry any policy on the non-Christian and their "customs, traditions, beliefs and interests" were to be considered by
Filipinos. The raging issue then was the conservation of the national the State in the formulation and implementation of State policies. President
patrimony for the Filipinos. Marcos abolished the CNI and transferred its functions to the Presidential
Adviser on National Minorities (PANAMIN). The PANAMIN was tasked to
integrate the ethnic groups that sought full integration into the larger
community, and at the same time "protect the rights of those who wish to 2. Their Concept of Land
preserve their original lifeways beside the larger community." [89] In short,
while still adopting the integration policy, the decree recognized the right Indigenous peoples share distinctive traits that set them apart from the
of tribal Filipinos to preserve their way of life.[90] Filipino mainstream. They are non-Christians. They live in less accessible,
marginal, mostly upland areas. They have a system of self-government not
In 1974, President Marcos promulgated P.D. No. 410, otherwise known dependent upon the laws of the central administration of the Republic of the
as the Ancestral Lands Decree. The decree provided for the issuance of land Philippines. They follow ways of life and customs that are perceived as
occupancy certificates to members of the national cultural communities who different from those of the rest of the population.[97] The kind of response the
were given up to 1984 to register their claims.[91] In 1979, the Commission on indigenous peoples chose to deal with colonial threat worked well to their
the Settlement of Land Problems was created under E.O. No. 561 which advantage by making it difficult for Western concepts and religion to erode
provided a mechanism for the expeditious resolution of land problems their customs and traditions. The "infieles societies" which had become
involving small settlers, landowners, and tribal Filipinos.[92] peripheral to colonial administration, represented, from a cultural perspective, a
much older base of archipelagic culture. The political systems were still
Despite the promulgation of these laws, from 1974 to the early 1980's, structured on the patriarchal and kinship oriented arrangement of power and
some 100,000 Kalingas and Bontoks of the Cordillera region were displaced by authority. The economic activities were governed by the concepts of an ancient
the Chico River dam project of the National Power Corporation (NPC). The communalism and mutual help. The social structure which emphasized division
Manobos of Bukidnon saw their land bulldozed by the Bukidnon Sugar of labor and distinction of functions, not status, was maintained. The cultural
Industries Company (BUSCO). In Agusan del Sur, the National Development styles and forms of life portraying the varieties of social courtesies and
Company was authorized by law in 1979 to take approximately 40,550 hectares ecological adjustments were kept constantly vibrant.[98]
of land that later became the NDC-Guthrie plantation in Agusan del Sur. Most
of the land was possessed by the Agusan natives.[93] Timber concessions, water Land is the central element of the indigenous peoples'
projects, plantations, mining, and cattle ranching and other projects of the existence. There is no traditional concept of permanent, individual, land
national government led not only to the eviction of the indigenous peoples from ownership. Among the Igorots, ownership of land more accurately applies to
their land but also to the reduction and destruction of their natural the tribal right to use the land or to territorial control. The people are the
environment.[94] secondary owners or stewards of the land and that if a member of the tribe
ceases to work, he loses his claim of ownership, and the land reverts to the
The Aquino government signified a total shift from the policy of beings of the spirit world who are its true and primary owners. Under the
integration to one of preservation. Invoking her powers under the Freedom concept of "trusteeship," the right to possess the land does not only belong to
Constitution, President Aquino created the Office of Muslim Affairs, Office the present generation but the future ones as well.[99]
for Northern Cultural Communities and the Office for Southern Cultural
Communities all under the Office of the President.[95] Customary law on land rests on the traditional belief that no one owns
the land except the gods and spirits, and that those who work the land are its
The 1987 Constitution carries at least six (6) provisions which insure mere stewards.[100]Customary law has a strong preference for communal
the right of tribal Filipinos to preserve their way of life. [96] This ownership, which could either be ownership by a group of individuals or
Constitution goes further than the 1973 Constitution by expressly families who are related by blood or by marriage,[101] or ownership by residents
guaranteeing the rights of tribal Filipinos to their ancestral domains and of the same locality who may not be related by blood or marriage. The system
ancestral lands. By recognizing their right to their ancestral lands and of communal ownership under customary laws draws its meaning from the
domains, the State has effectively upheld their right to live in a culture subsistence and highly collectivized mode of economic production. The
distinctly their own. Kalingas, for instance, who are engaged in team occupation like hunting,
foraging for forest products, and swidden farming found it natural that forest
areas, swidden farms, orchards, pasture and burial grounds should be Flavier, in his sponsorship speech, gave a background on the situation of
communally-owned.[102] For the Kalingas, everybody has a common right to a indigenous peoples in the Philippines, to wit:
common economic base. Thus, as a rule, rights and obligations to the land are
shared in common. "The Indigenous Cultural Communities, including the Bangsa Moro, have long
suffered from the dominance and neglect of government controlled by the
Although highly bent on communal ownership, customary law on majority. Massive migration of their Christian brothers to their homeland
land also sanctions individual ownership. The residential lots and terrace rice shrunk their territory and many of the tribal Filipinos were pushed to the
farms are governed by a limited system of individual ownership. It is limited hinterlands. Resisting the intrusion, dispossessed of their ancestral land and
because while the individual owner has the right to use and dispose of the with the massive exploitation of their natural resources by the elite among the
property, he does not possess all the rights of an exclusive and full owner as migrant population, they became marginalized. And the government has been
defined under our Civil Code.[103] Under Kalinga customary law, the alienation an indispensable party to this insidious conspiracy against the Indigenous
of individually-owned land is strongly discouraged except in marriage and Cultural Communities (ICCs). It organized and supported the resettlement of
succession and except to meet sudden financial needs due to sickness, death in people to their ancestral land, which was massive during the Commonwealth
the family, or loss of crops.[104] Moreover, and to be alienated should first be and early years of the Philippine Republic. Pursuant to the Regalian Doctrine
offered to a clan-member before any village-member can purchase it, and in no first introduced to our system by Spain through the Royal Decree of 13
case may land be sold to a non-member of the ili.[105] February 1894 or the Maura Law, the government passed laws to legitimize the
wholesale landgrabbing and provide for easy titling or grant of lands to migrant
Land titles do not exist in the indigenous peoples' economic and homesteaders within the traditional areas of the ICCs."[109]
social system. The concept of individual land ownership under the civil law
is alien to them.Inherently colonial in origin, our national land laws and Senator Flavier further declared:
governmental policies frown upon indigenous claims to ancestral
lands. Communal ownership is looked upon as inferior, if not inexistent.[106] "The IPs are the offsprings and heirs of the peoples who have first inhabited
and cared for the land long before any central government was
III. THE IPRA IS A NOVEL PIECE OF LEGISLATION. established. Their ancestors had territories over which they ruled themselves
and related with other tribes. These territories- the land- include people, their
A. The Legislative History of the IPRA dwelling, the mountains, the water, the air, plants, forest and the animals. This
is their environment in its totality. Their existence as indigenous peoples is
It was to address the centuries-old neglect of the Philippine manifested in their own lives through political, economic, socio-cultural and
indigenous peoples that the Tenth Congress of the Philippines, by their joint spiritual practices. The IPs culture is the living and irrefutable proof to this.
efforts, passed and approved R.A. No. 8371, the Indigenous Peoples Rights
Act (IPRA) of 1997. The law was a consolidation of two Bills-- Senate Bill No. Their survival depends on securing or acquiring land rights; asserting their
1728 and House Bill No. 9125. rights to it; and depending on it. Otherwise, IPs shall cease to exist as distinct
peoples."[110]
Principally sponsored by Senator Juan M. Flavier,[107] Senate Bill No.
1728 was a consolidation of four proposed measures referred to the Committees To recognize the rights of the indigenous peoples effectively, Senator Flavier
on Cultural Communities, Environment and Natural Resources, Ways and proposed a bill based on two postulates: (1) the concept of native title; and (2)
Means, as well as Finance. It adopted almost en toto the comprehensive version the principle of parens patriae.
of Senate Bill Nos. 1476 and 1486 which was a result of six regional
consultations and one national consultation with indigenous peoples According to Senator Flavier, "[w]hile our legal tradition subscribes to
nationwide.[108] At the Second Regular Session of the Tenth Congress, Senator the Regalian Doctrine reinstated in Section 2, Article XII of the 1987
Constitution," our "decisional laws" and jurisprudence passed by the State have Rep. Andolana's sponsorhip speech reads as follows:
"made exception to the doctrine." This exception was first laid down in the
case of Cario v. Insular Government where: "This Representation, as early as in the 8th Congress, filed a bill of similar
implications that would promote, recognize the rights of indigenous cultural
"x x x the court has recognized long occupancy of land by an indigenous communities within the framework of national unity and development.
member of the cultural communities as one of private ownership, which,
in legal concept, is termed "native title." This ruling has not been Apart from this, Mr. Speaker, is our obligation, the government's obligation to
overturned. In fact, it was affirmed in subsequent cases."[111] assure and ascertain that these rights shall be well-preserved and the cultural
traditions as well as the indigenous laws that remained long before this
Following Cario, the State passed Act No. 926, Act No. 2874, C.A. No. 141, Republic was established shall be preserved and promoted. There is a need, Mr.
P.D. 705, P.D. 410, P.D. 1529, R.A. 6734 (the Organic Act for the Autonomous Speaker, to look into these matters seriously and early approval of the substitute
Region of Muslim Mindanao). These laws, explicitly or implicitly, and liberally bill shall bring into reality the aspirations, the hope and the dreams of more
or restrictively, recognized "native title" or "private right" and the existence of than 12 million Filipinos that they be considered in the mainstream of the
ancestral lands and domains. Despite the passage of these laws, however, Philippine society as we fashion for the year 2000." [114]
Senator Flavier continued:
Rep. Andolana stressed that H.B. No. 9125 is based on the policy of
"x x x the executive department of government since the American preservation as mandated in the Constitution. He also emphasized that the
occupation has not implemented the policy. In fact, it was more honored rights of IPs to their land was enunciated in Cario v. Insular
in its breach than in its observance, its wanton disregard shown during Government which recognized the fact that they had vested rights prior to the
the period unto the Commonwealth and the early years of the Philippine establishment of the Spanish and American regimes.[115]
Republic when government organized and supported massive
resettlement of the people to the land of the ICCs." After exhaustive interpellation, House Bill No. 9125, and its
corresponding amendments, was approved on Second Reading with no
Senate Bill No. 1728 seeks to genuinely recognize the IPs right to own objections.
and possess their ancestral land. The bill was prepared also under the principle
of parens patriaeinherent in the supreme power of the State and deeply IV. THE PROVISIONS OF THE IPRA DO NOT CONTRAVENE
embedded in Philippine legal tradition. This principle mandates that persons THE CONSTITUTION.
suffering from serious disadvantage or handicap, which places them in a
position of actual inequality in their relation or transaction with others, are A. Ancestral Domains and Ancestral Lands are the Private Property
entitled to the protection of the State. of Indigenous Peoples and Do Not Constitute Part of the Land of the
Public Domain.
Senate Bill No. 1728 was passed on Third Reading by twenty-one (21)
Senators voting in favor and none against, with no abstention.[112] The IPRA grants to ICCs/IPs a distinct kind of ownership over
ancestral domains and ancestral lands. Ancestral lands are not the same as
House Bill No. 9125 was sponsored by Rep. Zapata, Chairman of the ancestral domains. These are defined in Section 3 [a] and [b] of the Indigenous
Committee on Cultural Communities. It was originally authored and Peoples Right Act, viz:
subsequently presented and defended on the floor by Rep. Gregorio
Andolana of North Cotabato.[113] "Sec. 3 a) Ancestral Domains. -- Subject to Section 56 hereof, refer to all areas
generally belonging to ICCs/IPs comprising lands, inland waters, coastal areas,
and natural resources therein, held under a claim of ownership, occupied or Ancestral lands are lands held by the ICCs/IPs under the same
possessed by ICCs/IPs by themselves or through their ancestors, communally conditions as ancestral domains except that these are limited to lands and that
or individually since time immemorial, continuously to the present except when these lands are not merely occupied and possessed but are also utilized by the
interrupted by war, force majeure or displacement by force, deceit, stealth or as ICCs/IPs under claims of individual or traditional group ownership. These
a consequence of government projects or any other voluntary dealings entered lands include but are not limited to residential lots, rice terraces or paddies,
into by government and private individuals/corporations, and which are private forests, swidden farms and tree lots.[117]
necessary to ensure their economic, social and cultural welfare. It shall include
ancestral lands, forests, pasture, residential, agricultural, and other lands The procedures for claiming ancestral domains and lands are similar to
individually owned whether alienable and disposable or otherwise, hunting the procedures embodied in Department Administrative Order (DAO) No. 2,
grounds, burial grounds, worship areas, bodies of water, mineral and other series of 1993, signed by then Secretary of the Department of Environment and
natural resources, and lands which may no longer be exclusively occupied by Natural Resources (DENR) Angel Alcala.[118] DAO No. 2 allowed the
ICCs/IPs but from which they traditionally had access to for their subsistence delineation of ancestral domains by special task forces and ensured the issuance
and traditional activities, particularly the home ranges of ICCs/IPs who are still of Certificates of Ancestral Land Claims (CALC's) and Certificates of
nomadic and/or shifting cultivators; Ancestral Domain Claims (CADC's) to IPs.

b) Ancestral Lands.-- Subject to Section 56 hereof, refers to land occupied, The identification and delineation of these ancestral domains and lands is
possessed and utilized by individuals, families and clans who are members of a power conferred by the IPRA on the National Commission on Indigenous
the ICCs/IPs since time immemorial, by themselves or through their Peoples (NCIP).[119] The guiding principle in identification and delineation is
predecessors-in-interest, under claims of individual or traditional group self-delineation.[120] This means that the ICCs/IPs have a decisive role in
ownership, continuously, to the present except when interrupted by war, force determining the boundaries of their domains and in all the activities pertinent
majeure or displacement by force, deceit, stealth, or as a consequence of thereto.[121]
government projects and other voluntary dealings entered into by government
and private individuals/corporations, including, but not limited to, residential The procedure for the delineation and recognition of ancestral
lots, rice terraces or paddies, private forests, swidden farms and tree lots." domains is set forth in Sections 51 and 52 of the IPRA. The identification,
delineation and certification of ancestral lands is in Section 53 of said law.
Ancestral domains are all areas belonging to ICCs/IPs held under a
claim of ownership, occupied or possessed by ICCs/IPs by themselves or Upon due application and compliance with the procedure provided under
through their ancestors, communally or individually since time immemorial, the law and upon finding by the NCIP that the application is meritorious, the
continuously until the present, except when interrupted by war, force majeure NCIP shall issue a Certificate of Ancestral Domain Title (CADT) in the name
or displacement by force, deceit, stealth or as a consequence of government of the community concerned.[122] The allocation of lands within the ancestral
projects or any other voluntary dealings with government and/or private domain to any individual or indigenous corporate (family or clan) claimants is
individuals or corporations. Ancestral domains comprise lands, inland left to the ICCs/IPs concerned to decide in accordance with customs and
waters, coastal areas, and natural resources therein and includes ancestral traditions.[123] With respect to ancestral lands outside the ancestral domain,
lands, forests, pasture, residential, agricultural, and other lands the NCIP issues a Certificate of Ancestral Land Title (CALT).[124]
individually owned whether alienable or not, hunting grounds, burial
grounds, worship areas, bodies of water, mineral and other natural
CADT's and CALT's issued under the IPRA shall be registered by the
resources. They also include lands which may no longer be exclusively
NCIP before the Register of Deeds in the place where the property is
occupied by ICCs/IPs but from which they traditionally had access to for their
situated.[125]
subsistence and traditional activities, particularly the home ranges of ICCs/IPs
who are still nomadic and/or shifting cultivators.[116]
(1) Right to Ancestral Domains and Ancestral Lands: How Acquired
The rights of the ICCs/IPs to their ancestral domains and ancestral lands In 1903, Don Mateo Cario, an Ibaloi, sought to register with the land
may be acquired in two modes: (1) by native title over both ancestral lands registration court 146 hectares of land in Baguio Municipality, Benguet
and domains; or (2) by torrens title under the Public Land Act and the Province. He claimed that this land had been possessed and occupied by his
Land Registration Act with respect to ancestral lands only. ancestors since time immemorial; that his grandfather built fences around the
property for the holding of cattle and that his father cultivated some parts of the
(2) The Concept of Native Title land. Cario inherited the land in accordance with Igorot custom. He tried to
have the land adjusted under the Spanish land laws, but no document issued
Native title is defined as: from the Spanish Crown.[131] In 1901, Cario obtained a possessory title to the
land under the Spanish Mortgage Law.[132] The North American colonial
"Sec. 3 [l]. Native Title-- refers to pre-conquest rights to lands and domains government, however, ignored his possessory title and built a public road on
which, as far back as memory reaches, have been held under a claim the land prompting him to seek a Torrens title to his property in the land
of private ownership by ICCs/IPs, have never been public lands and are registration court. While his petition was pending, a U.S. military
thus indisputably presumed to have been held that way since before the reservation[133] was proclaimed over his land and, shortly thereafter, a military
Spanish Conquest."[126] detachment was detailed on the property with orders to keep cattle and
trespassers, including Cario, off the land.[134]
Native title refers to ICCs/IPs' preconquest rights to lands and domains held
under a claim of private ownership as far back as memory reaches. These lands In 1904, the land registration court granted Cario's application for
are deemed never to have been public lands and are indisputably presumed to absolute ownership to the land. Both the Government of the Philippine Islands
have been held that way since before the Spanish Conquest. The rights of and the U.S. Government appealed to the C.F.I. of Benguet which reversed the
ICCs/IPs to their ancestral domains (which also include ancestral lands) by land registration court and dismissed Cario's application. The Philippine
virtue of native title shall be recognized and respected.[127] Formal recognition, Supreme Court[135] affirmed the C.F.I. by applying the Valenton ruling. Cario
when solicited by ICCs/IPs concerned, shall be embodied in a Certificate of took the case to the U.S. Supreme Court.[136] On one hand, the Philippine
Ancestral Domain Title (CADT), which shall recognize the title of the government invoked the Regalian doctrine and contended that Cario failed to
concerned ICCs/IPs over the territories identified and delineated.[128] comply with the provisions of the Royal Decree of June 25, 1880, which
required registration of land claims within a limited period of time. Cario, on
Like a torrens title, a CADT is evidence of private ownership of land by the other, asserted that he was the absolute owner of the land jure gentium, and
native title. Native title, however, is a right of private ownership peculiarly that the land never formed part of the public domain.
granted to ICCs/IPs over their ancestral lands and domains. The IPRA
categorically declares ancestral lands and domains held by native title as never In a unanimous decision written by Justice Oliver Wendell Holmes, the
to have been public land. Domains and lands held under native title are, U.S. Supreme Court held:
therefore, indisputably presumed to have never been public lands and are
private. "It is true that Spain, in its earlier decrees, embodied the universal feudal theory
that all lands were held from the Crown, and perhaps the general attitude of
(a) Cario v. Insular Government[129] conquering nations toward people not recognized as entitled to the treatment
accorded to those in the same zone of civilization with themselves. It is true,
The concept of native title in the IPRA was taken from the 1909 case also, that in legal theory, sovereignty is absolute, and that, as against foreign
of Cario v. Insular Government.[130] Cario firmly established a concept of nations, the United States may assert, as Spain asserted, absolute power. But it
private land title that existed irrespective of any royal grant from the State. does not follow that, as against the inhabitants of the Philippines, the United
States asserts that Spain had such power. When theory is left on one side,
sovereignty is a question of strength, and may vary in degree. How far a new
sovereign shall insist upon the theoretical relation of the subjects to the head in "[E]very presumption is and ought to be against the government in a case like the
the past, and how far it shall recognize actual facts, are matters for it to present. It might, perhaps, be proper and sufficient to say that when, as far back as
decide."[137] testimony or memory goes, the land has been held by individuals under a claim of
private ownership, it will be presumed to have been held in the same way from
The U.S. Supreme Court noted that it need not accept Spanish doctrines. The before the Spanish conquest, and never to have been public land. Certainly in a case
choice was with the new colonizer. Ultimately, the matter had to be decided under U.S. like this, if there is doubt or ambiguity in the Spanish law, we ought to give the applicant
law. the benefit of the doubt."[140]

The Cario decision largely rested on the North American constitutionalist's concept
The court thus laid down the presumption of a certain title held (1) as far back as
of "due process" as well as the pronounced policy "to do justice to the natives."[138] It was
testimony or memory went, and (2) under a claim of private ownership. Land held by this
based on the strong mandate extended to the Islands via the Philippine Bill of 1902 that
title is presumed to "never have been public land."
"No law shall be enacted in said islands which shall deprive any person of life, liberty, or
property without due process of law, or deny to any person therein the equal protection of Against this presumption, the U.S. Supreme Court analyzed the Spanish decrees
the laws." The court declared: upheld in the 1904 decision of Valenton v. Murciano. The U.S. Supreme Court
found no proof that the Spanish decrees did not honor native title. On the contrary, the
"The acquisition of the Philippines was not like the settlement of the white race in the decrees discussed in Valenton appeared to recognize that the natives owned some land,
United States. Whatever consideration may have been shown to the North American irrespective of any royal grant.The Regalian doctrine declared in the preamble of
Indians, the dominant purpose of the whites in America was to occupy land. It is obvious the Recopilacion was all "theory and discourse" and it was observed that titles were
that, however stated, the reason for our taking over the Philippines was different. No one, admitted to exist beyond the powers of the Crown, viz:
we suppose, would deny that, so far as consistent with paramount necessities, our first
object in the internal administration of the islands is to do justice to the natives, not to "If the applicant's case is to be tried by the law of Spain, we do not discover such
exploit their country for private gain. By the Organic Act of July 1, 1902, chapter 1369, clear proof that it was bad by that law as to satisfy us that he does not own the
section 12 (32 Statutes at Large, 691), all the property and rights acquired there by the land. To begin with, the older decrees and laws cited by the counsel for the plaintiff
United States are to be administered 'for the benefit of the inhabitants thereof.' It is in error seem to indicate pretty clearly that the natives were recognized as owning
reasonable to suppose that the attitude thus assumed by the United States with regard to some lands, irrespective of any royal grant. In other words, Spain did not assume to
what was unquestionably its own is also its attitude in deciding what it will claim for its convert all the native inhabitants of the Philippines into trespassers or even into tenants at
own. The same statute made a bill of rights, embodying the safeguards of the Constitution, will. For instance, Book 4, title 12, Law 14 of the the Recopilacion de Leyes de las Indias,
and, like the Constitution, extends those safeguards to all. It provides that 'no law shall be cited for a contrary conclusion in Valenton v. Murciano, 3 Philippine 537, while it
enacted in said islands which shall deprive any person of life, liberty, or property without commands viceroys and others, when it seems proper, to call for the exhibition of grants,
due process of law, or deny to any person therein the equal protection of the laws.' In the directs them to confirm those who hold by good grants or justa prescripcion. It is true
light of the declaration that we have quoted from section 12, it is hard to believe that the that it begins by the characteristic assertion of feudal overlordship and the origin of
United States was ready to declare in the next breath that "any person" did not embrace all titles in the King or his predecessors. That was theory and discourse. The fact
the inhabitants of Benguet, or that it meant by "property" only that which had become was that titles were admitted to exist that owed nothing to the powers of Spain
such by ceremonies of which presumably a large part of the inhabitants never had heard, beyond this recognition in their books." (Emphasis supplied).[141]
and that it proposed to treat as public land what they, by native custom and by long
association,-- of the profoundest factors in human thought,-- regarded as their own."[139]
The court further stated that the Spanish "adjustment" proceedings never held sway
over unconquered territories. The wording of the Spanish laws were not framed in a
The Court went further: manner as to convey to the natives that failure to register what to them has always been
their own would mean loss of such land. The registration requirement was "not to confer
title, but simply to establish it;" it was "not calculated to convey to the mind of an Igorot
chief the notion that ancient family possessions were in danger, if he had read every word This is the only instance when Justice Holmes used the term "native title" in the entire
of it." length of the Cario decision. It is observed that the widespread use of the term "native
title" may be traced to Professor Owen James Lynch, Jr., a Visiting Professor at the
By recognizing this kind of title, the court clearly repudiated the doctrine University of the Philippines College of Law from the Yale University Law School. In
of Valenton. It was frank enough, however, to admit the possibility that the applicant 1982, Prof. Lynch published an article in the Philippine Law Journal entitled Native
might have been deprived of his land under Spanish law because of the inherent Title, Private Right and Tribal Land Law.[146] This article was made after Professor
ambiguity of the decrees and concomitantly, the various interpretations which may be Lynch visited over thirty tribal communities throughout the country and studied the
given them. But precisely because of the ambiguity and of the strong "due process origin and development of Philippine land laws.[147] He discussed Cario extensively and
mandate" of the Constitution, the court validated this kind of title.[142] This title was used the term "native title" to refer to Cario's title as discussed and upheld by the U.S.
sufficient, even without government administrative action, and entitled the holder to a Supreme Court in said case.
Torrens certificate. Justice Holmes explained:

"It will be perceived that the rights of the applicant under the Spanish law present a
problem not without difficulties for courts of a legal tradition. We have deemed it proper (b) Indian Title
on that account to notice the possible effect of the change of sovereignty and the act of
Congress establishing the fundamental principles now to be observed. Upon a
consideration of the whole case we are of the opinion that law and justice require that the In a footnote in the same article, Professor Lynch stated that the concept of "native
applicant should be granted what he seeks, and should not be deprived of what, by the title" as defined by Justice Holmes in Cario "is conceptually similar to "aboriginal title"
practice and belief of those among whom he lived, was his property, through a refined of the American Indians.[148] This is not surprising, according to Prof. Lynch, considering
interpretation of an almost forgotten law of Spain."[143] that during the American regime, government policy towards ICCs/IPs was consistently
made in reference to native Americans.[149] This was clearly demonstrated in the case
of Rubi v. Provincial Board of Mindoro.[150]
Thus, the court ruled in favor of Cario and ordered the registration of the 148
hectares in Baguio Municipality in his name.[144] In Rubi, the Provincial Board of Mindoro adopted a Resolution authorizing the
provincial governor to remove the Mangyans from their domains and place them in a
Examining Cario closer, the U.S. Supreme Court did not categorically refer to the
permanent reservation in Sitio Tigbao, Lake Naujan. Any Mangyan who refused to
title it upheld as "native title." It simply said:
comply was to be imprisoned. Rubi and some Mangyans, including one who was
imprisoned for trying to escape from the reservation, filed for habeas corpus claiming
"The Province of Benguet was inhabited by a tribe that the Solicitor-General, in his deprivation of liberty under the Board Resolution. This Court denied the petition on the
argument, characterized as a savage tribe that never was brought under the civil or ground of police power. It upheld government policy promoting the idea that a permanent
military government of the Spanish Crown. It seems probable, if not certain, that settlement was the only successful method for educating the Mangyans, introducing
the Spanish officials would not have granted to anyone in that province the civilized customs, improving their health and morals, and protecting the public forests in
registration to which formerly the plaintiff was entitled by the Spanish Laws, and which they roamed.[151] Speaking through Justice Malcolm, the court said:
which would have made his title beyond question good. Whatever may have been the
technical position of Spain it does not follow that, in the view of the United States, he had
"Reference was made in the President's instructions to the Commission to the policy
lost all rights and was a mere trespasser when the present government seized his
adopted by the United States for the Indian Tribes. The methods followed by the
land. The argument to that effect seems to amount to a denial of native titles through an
Government of the Philippine Islands in its dealings with the so-called non-Christian
important part of the Island of Luzon, at least, for the want of ceremonies which the
people is said, on argument, to be practically identical with that followed by the United
Spaniards would not have permitted and had not the power to enforce."[145]
States Government in its dealings with the Indian tribes. Valuable lessons, it is insisted,
can be derived by an investigation of the American-Indian policy.
From the beginning of the United States, and even before, the Indians have been treated nature of aboriginal title was made in 1823 in Johnson & Graham's Lessee v.
as "in a state of pupilage." The recognized relation between the Government of the M'Intosh.[159]
United States and the Indians may be described as that of guardian and ward. It is for the
Congress to determine when and how the guardianship shall be terminated. The Indians In Johnson, the plaintiffs claimed the land in question under two (2) grants made
are always subject to the plenary authority of the United States. [152] by the chiefs of two (2) Indian tribes. The U.S. Supreme Court refused to recognize this
conveyance, the plaintiffs being private persons. The only conveyance that was
recognized was that made by the Indians to the government of the European
x x x. discoverer. Speaking for the court, Chief Justice Marshall pointed out that the potentates
of the old world believed that they had made ample compensation to the inhabitants of
As to the second point, the facts in the Standing Bear case and the Rubi case are not the new world by bestowing civilization and Christianity upon them; but in addition, said
exactly identical. But even admitting similarity of facts, yet it is known to all that Indian the court, they found it necessary, in order to avoid conflicting settlements and
reservations do exist in the United States, that Indians have been taken from different consequent war, to establish the principle that discovery gives title to the government
parts of the country and placed on these reservations, without any previous consultation by whose subjects, or by whose authority, the discovery was made, against all other
as to their own wishes, and that, when once so located, they have been made to remain on European governments, which title might be consummated by possession. [160] The
the reservation for their own good and for the general good of the country. If any lesson exclusion of all other Europeans gave to the nation making the discovery the sole right of
can be drawn from the Indian policy of the United States, it is that the determination of acquiring the soil from the natives and establishing settlements upon it. As regards the
this policy is for the legislative and executive branches of the government and that when natives, the court further stated that:
once so decided upon, the courts should not interfere to upset a carefully planned
governmental system. Perhaps, just as many forceful reasons exist for the segregation of "Those relations which were to exist between the discoverer and the natives were to be
the Manguianes in Mindoro as existed for the segregation of the different Indian tribes in regulated by themselves. The rights thus acquired being exclusive, no other power could
the United States."[153] interpose between them.

Rubi applied the concept of Indian land grants or reservations in the Philippines. An In the establishment of these relations, the rights of the original inhabitants were, in no
Indian reservation is a part of the public domain set apart by proper authority for the use instance, entirely disregarded; but were necessarily, to a considerable extent,
and occupation of a tribe or tribes of Indians.[154] It may be set apart by an act of Congress, impaired. They were admitted to be the rightful occupants of the soil, with a legal as
by treaty, or by executive order, but it cannot be established by custom and well as just claim to retain possession of it, and to use it according to their own
prescription.[155] discretion; but their rights to complete sovereignty, as independent nations, were
Indian title to land, however, is not limited to land grants or reservations. It necessarily diminished, and their power to dispose of the soil at their own will, to
also covers the "aboriginal right of possession or occupancy."[156] The aboriginal right whomsoever they pleased, was denied by the fundamental principle that discovery gave
of possession depends on the actual occupancy of the lands in question by the tribe or exclusive title to those who made it.
nation as their ancestral home, in the sense that such lands constitute definable territory
occupied exclusively by the particular tribe or nation.[157] It is a right which exists apart While the different nations of Europe respected the right of the natives as occupants,
from any treaty, statute, or other governmental action, although in numerous instances they asserted the ultimate dominion to be in themselves; and claimed and exercised,
treaties have been negotiated with Indian tribes, recognizing their aboriginal possession as a consequence of this ultimate dominion, a power to grant the soil, while yet in
and delimiting their occupancy rights or settling and adjusting their boundaries.[158] possession of the natives. These grants have been understood by all to convey a title
to the grantees, subject only to the Indian right of occupancy." [161]
American jurisprudence recognizes the Indians' or native Americans' rights to
land they have held and occupied before the "discovery" of the Americas by the
Europeans.The earliest definitive statement by the U.S. Supreme Court on the Thus, the discoverer of new territory was deemed to have obtained the exclusive
right to acquire Indian land and extinguish Indian titles. Only to the discoverer-- whether
to England, France, Spain or Holland-- did this right belong and not to any other nation or of the United States in its place, led naturally to the declaration, on the part of the
private person. The mere acquisition of the right nonetheless did not extinguish Indian Cherokees, that they were under the protection of the United States, and of no other
claims to land. Rather, until the discoverer, by purchase or conquest, exercised its right, power. They assumed the relation with the United States which had before subsisted with
the concerned Indians were recognized as the "rightful occupants of the soil, with a legal Great Britain.
as well as just claim to retain possession of it." Grants made by the discoverer to her
subjects of lands occupied by the Indians were held to convey a title to the grantees, This relation was that of a nation claiming and receiving the protection of one more
subject only to the Indian right of occupancy.Once the discoverer purchased the land powerful, not that of individuals abandoning their national character, and submitting as
from the Indians or conquered them, it was only then that the discoverer gained an subjects to the laws of a master."[166]
absolute title unrestricted by Indian rights.
The court concluded, in essence, that a grant of Indian lands by Indians could not It was the policy of the U.S. government to treat the Indians as nations with distinct
convey a title paramount to the title of the United States itself to other parties, saying: territorial boundaries and recognize their right of occupancy over all the lands within
their domains.Thus:
"It has never been contended that the Indian title amounted to nothing. Their right of
possession has never been questioned. The claim of government extends to the "From the commencement of our government Congress has passed acts to regulate trade
complete ultimate title, charged with this right of possession, and to the exclusive and intercourse with the Indians; which treat them as nations, respect their rights, and
power of acquiring that right."[162] manifest a firm purpose to afford that protection which treaties stipulate. All these acts,
and especially that of 1802, which is still in force, manifestly consider the several Indian
It has been said that the history of America, from its discovery to the present day, nations as distinct political communities, having territorial boundaries, within which
proves the universal recognition of this principle.[163] their authority is exclusive, and having a right to all the lands within those
boundaries, which is not only acknowledged, but guaranteed by the United States.
The Johnson doctrine was a compromise. It protected Indian rights and their native
lands without having to invalidate conveyances made by the government to many U.S. x x x.
citizens.[164]
Johnson was reiterated in the case of Worcester v. Georgia.[165] In this case, the "The Indian nations had always been considered as distinct, independent political
State of Georgia enacted a law requiring all white persons residing within the Cherokee communities, retaining their original natural rights, as the undisputed possessors of
nation to obtain a license or permit from the Governor of Georgia; and any violation of the soil from time immemorial, with the single exception of that imposed by irresistible
the law was deemed a high misdemeanor. The plaintiffs, who were white missionaries, power, which excluded them from intercourse with any other European potentate than the
did not obtain said license and were thus charged with a violation of the Act. first discoverer of the coast of the particular region claimed: and this was a restriction
which those European potentates imposed on themselves, as well as on the Indians. The
The U.S. Supreme Court declared the Act as unconstitutional for interfering with very term "nation," so generally applied to them, means "a people distinct from others."
the treaties established between the United States and the Cherokee nation as well as the x x x.[167]
Acts of Congress regulating intercourse with them. It characterized the relationship
between the United States government and the Indians as:
The Cherokee nation, then, is a distinct community, occupying its own territory, with
boundaries accurately described, in which the laws of Georgia can have no force, and
"The Indian nations were, from their situation, necessarily dependent on some foreign which the citizens of Georgia have no right to enter but with the assent of the Cherokees
potentate for the supply of their essential wants, and for their protection from lawless and themselves or in conformity with treaties and with the acts of Congress. The whole
injurious intrusions into their country. That power was naturally termed their intercourse between the United States and this nation is, by our Constitution and laws,
protector. They had been arranged under the protection of Great Britain; but the vested in the government of the United States."[168]
extinguishment of the British power in their neighborhood, and the establishment of that
The discovery of the American continent gave title to the government of the The American judiciary struggled for more than 200 years with the ancestral
discoverer as against all other European governments. Designated as the naked land claims of indigenous Americans.[182] And two things are clear. First, aboriginal
fee,[169] this title was to be consummated by possession and was subject to the Indian title title is recognized. Second, indigenous property systems are also recognized. From a
of occupancy. The discoverer acknowledged the Indians' legal and just claim to retain legal point of view, certain benefits can be drawn from a comparison of Philippine IPs to
possession of the land, the Indians being the original inhabitants of the land. The native Americans.[183] Despite the similarities between native title and aboriginal title,
discoverer nonetheless asserted the exclusive right to acquire the Indians' land-- either by however, there are at present some misgivings on whether jurisprudence on American
purchase, "defensive" conquest, or cession-- and in so doing, extinguish the Indian Indians may be cited authoritatively in the Philippines. The U.S. recognizes the
title. Only the discoverer could extinguish Indian title because it alone asserted ultimate possessory rights of the Indians over their land; title to the land, however, is deemed to
dominion in itself. Thus, while the different nations of Europe respected the rights of the have passed to the U.S. as successor of the discoverer. The aboriginal title of ownership
natives as occupants, they all asserted the ultimate dominion and title to be in is not specifically recognized as ownership by action authorized by Congress. [184] The
themselves.[170] protection of aboriginal title merely guards against encroachment by persons other than
the Federal Government.[185] Although there are criticisms against the refusal to recognize
As early as the 19th century, it became accepted doctrine that although fee the native Americans' ownership of these lands,[186] the power of the State to extinguish
title to the lands occupied by the Indians when the colonists arrived became vested these titles has remained firmly entrenched.[187]
in the sovereign-- first the discovering European nation and later the original 13
States and the United States-- a right of occupancy in the Indian tribes was Under the IPRA, the Philippine State is not barred form asserting sovereignty over
nevertheless recognized.The Federal Government continued the policy of respecting the the ancestral domains and ancestral lands.[188] The IPRA, however, is still in its infancy
Indian right of occupancy, sometimes called Indian title, which it accorded the protection and any similarities between its application in the Philippines vis--vis American
of complete ownership.[171] But this aboriginal Indian interest simply constitutes Jurisprudence on aboriginal title will depend on the peculiar facts of each case.
"permission" from the whites to occupy the land, and means mere possession not
specifically recognized as ownership by Congress.[172] It is clear that this right of
occupancy based upon aboriginal possession is not a property right.[173] It is vulnerable to
affirmative action by the federal government who, as sovereign, possessed exclusive (c) Why the Cario doctrine is unique
power to extinguish the right of occupancy at will.[174] Thus, aboriginal title is not the
same as legal title. Aboriginal title rests on actual, exclusive and continuous use and
occupancy for a long time.[175] It entails that land owned by Indian title must be used In the Philippines, the concept of native title first upheld in Cario and enshrined in
within the tribe, subject to its laws and customs, and cannot be sold to another sovereign the IPRA grants ownership, albeit in limited form, of the land to the ICCs/IPs. Native title
government nor to any citizen.[176] Such title as Indians have to possess and occupy land presumes that the land is private and was never public. Cario is the only case that
is in the tribe, and not in the individual Indian; the right of individual Indians to share in specifically and categorically recognizes native title. The long line of cases
the tribal property usually depends upon tribal membership, the property of the tribe citing Cario did not touch on native title and the private character of ancestral
generally being held in communal ownership.[177] domains and lands. Cario was cited by the succeeding cases to support the concept of
acquisitive prescription under the Public Land Act which is a different matter
As a rule, Indian lands are not included in the term "public lands," which is altogether. Under the Public Land Act, land sought to be registered must be public
ordinarily used to designate such lands as are subject to sale or other disposal under agricultural land. When the conditions specified in Section 48 [b] of the Public Land
general laws.[178] Indian land which has been abandoned is deemed to fall into the public Act are complied with, the possessor of the land is deemed to have acquired, by operation
domain.[179] On the other hand, an Indian reservation is a part of the public domain set of law, a right to a grant of the land.[189] The land ceases to be part of the public
apart for the use and occupation of a tribe of Indians.[180] Once set apart by proper domain,[190] ipso jure,[191] and is converted to private property by the mere lapse or
authority, the reservation ceases to be public land, and until the Indian title is completion of the prescribed statutory period.
extinguished, no one but Congress can initiate any preferential right on, or restrict the
nation's power to dispose of, them.[181] It was only in the case of Oh Cho v. Director of Lands[192] that the court declared
that the rule that all lands that were not acquired from the government, either by purchase
or grant, belong to the public domain has an exception. This exception would be any land The option granted under this section shall be exercised within twenty (20) years from the
that should have been in the possession of an occupant and of his predecessors-in-interest approval of this Act."[196]
since time immemorial. It is this kind of possession that would justify the presumption
that the land had never been part of the public domain or that it had been private property ICCs/IPs are given the option to secure a torrens certificate of title over their individually-
even before the Spanish conquest.[193] Oh Cho, however, was decided under the owned ancestral lands. This option is limited to ancestral lands only, not domains, and
provisions of the Public Land Act and Cario was cited to support the applicant's claim of such lands must be individually, not communally, owned.
acquisitive prescription under the said Act.
Ancestral lands that are owned by individual members of ICCs/IPs who, by
All these years, Cario had been quoted out of context simply to justify long, themselves or through their predecessors-in-interest, have been in continuous possession
continuous, open and adverse possession in the concept of owner of public agricultural and occupation of the same in the concept of owner since time immemorial[197] or for a
land. It is this long, continuous, open and adverse possession in the concept of owner of period of not less than 30 years, which claims are uncontested by the members of the
thirty years both for ordinary citizens[194] and members of the national cultural same ICCs/IPs, may be registered under C.A. 141, otherwise known as the Public Land
minorities[195] that converts the land from public into private and entitles the registrant to Act, or Act 496, the Land Registration Act. For purposes of registration, the individually-
a torrens certificate of title. owned ancestral lands are classified as alienable and disposable agricultural lands of the
(3) The Option of Securing a Torrens Title to the Ancestral Land Indicates public domain, provided, they are agricultural in character and are actually used for
that the Land is Private. agricultural, residential, pasture and tree farming purposes. These lands shall be classified
as public agricultural lands regardless of whether they have a slope of 18% or more.
The private character of ancestral lands and domains as laid down in the IPRA is
further strengthened by the option given to individual ICCs/IPs over their individually- The classification of ancestral land as public agricultural land is in compliance with
owned ancestrallands. For purposes of registration under the Public Land Act and the requirements of the Public Land Act and the Land Registration Act. C.A. 141, the
the Land Registration Act, the IPRA expressly converts ancestral land into public Public Land Act, deals specifically with lands of the public domain. [198] Its provisions
agricultural land which may be disposed of by the State. The necessary implication apply to those lands "declared open to disposition or concession" x x x "which have not
is that ancestral land is private. It, however, has to be first converted to public been reserved for public or quasi-public purposes, nor appropriated by the Government,
agricultural land simply for registration purposes. To wit: nor in any manner become private property, nor those on which a private right authorized
and recognized by this Act or any other valid law x x x or which having been reserved or
appropriated, have ceased to be so."[199] Act 496, the Land Registration Act, allows
"Sec. 12. Option to Secure Certificate of Title Under Commonwealth Act 141, as registration only of private lands and public agricultural lands. Since ancestral domains
amended, or the Land Registration Act 496-- Individual members of cultural and lands are private, if the ICC/IP wants to avail of the benefits of C.A. 141 and
communities, with respect to their individually-owned ancestral lands who, by Act 496, the IPRA itself converts his ancestral land, regardless of whether the land
themselves or through their predecessors-in-interest, have been in continuous possession has a slope of eighteen per cent (18%) or over,[200] from private to public
and occupation of the same in the concept of owner since time immemorial or for a agricultural land for proper disposition.
period of not less than thirty (30) years immediately preceding the approval of this Act
and uncontested by the members of the same ICCs/IPs shall have the option to secure title The option to register land under the Public Land Act and the Land Registration
to their ancestral lands under the provisions of Commonwealth Act 141, as amended, or Act has nonetheless a limited period. This option must be exercised within twenty (20)
the Land Registration Act 496. years from October 29, 1997, the date of approval of the IPRA.
Thus, ancestral lands and ancestral domains are not part of the lands of the
For this purpose, said individually-owned ancestral lands, which are agricultural in public domain. They are private and belong to the ICCs/IPs. Section 3 of Article
character and actually used for agricultural, residential, pasture, and tree farming XII on National Economy and Patrimony of the 1987 Constitution classifies lands of the
purposes, including those with a slope of eighteen percent (18%) or more, are hereby public domain into four categories: (a) agricultural, (b) forest or timber, (c) mineral lands,
classified as alienable and disposable agricultural lands. and (d) national parks.Section 5 of the same Article XII mentions ancestral lands and
ancestral domains but it does not classify them under any of the said four categories. To "Sec. 48. The following described citizens of the Philippines, occupying lands of the
classify them as public lands under any one of the four classes will render the entire public domain or claiming to own any such lands or an interest therein, but whose titles
IPRA law a nullity. The spirit of the IPRA lies in the distinct concept of ancestral have not been perfected or completed, may apply to the Court of First Instance of the
domains and ancestral lands. The IPRA addresses the major problem of the ICCs/IPs province where the land is located for confirmation of their claims and the issuance of a
which is loss of land. Land and space are of vital concern in terms of sheer survival of the certificate of title therefor, under the Land Registration Act, to wit:
ICCs/IPs.[201]
The 1987 Constitution mandates the State to "protect the rights of indigenous (a) [perfection of Spanish titles] xxx.
cultural communities to their ancestral lands" and that "Congress provide for the
applicability of customary laws x x x in determining the ownership and extent of (b) Those who by themselves or through their predecessors-in-interest have been in open,
ancestral domain."[202] It is the recognition of the ICCs/IPs distinct rights of continuous, exclusive, and notorious possession and occupation of agricultural lands of
ownership over their ancestral domains and lands that breathes life into this the public domain, under a bona fide claim of acquisition or ownership, for at least thirty
constitutional mandate. years immediately preceding the filing of the application for confirmation of title except
when prevented by war or force majeure. These shall be conclusively presumed to have
B. The right of ownership and possession by the ICCs/IPs of their ancestral performed all the conditions essential to a Government grant and shall be entitled to a
domains is a limited form of ownership and does not include the right to alienate the certificate of title under the provisions of this Chapter.
same.
Registration under the Public Land Act and Land Registration Act recognizes the (c) Members of the national cultural minorities who by themselves or through their
concept of ownership under the civil law. This ownership is based on adverse possession predecessors-in-interest have been in open, continuous, exclusive and notorious
for a specified period, and harkens to Section 44 of the Public Land Act on administrative possession and occupation of lands of the public domain suitable to agriculture,
legalization (free patent) of imperfect or incomplete titles and Section 48 (b) and (c) of whether disposable or not, under a bona fide claim of ownership for at least 30 years
the same Act on the judicial confirmation of imperfect or incomplete titles. Thus: shall be entitled to the rights granted in sub-section (b) hereof."[204]

"Sec. 44. Any natural-born citizen of the Philippines who is not the owner of more than Registration under the foregoing provisions presumes that the land was originally
twenty-four hectares and who since July fourth, 1926 or prior thereto, has continuously public agricultural land but because of adverse possession since July 4, 1955 (free patent)
occupied and cultivated, either by himself or through his predecessors-in-interest, a tract or at least thirty years (judicial confirmation), the land has become private. Open, adverse,
or tracts of agricultural public lands subject to disposition, or who shall have paid the real public and continuous possession is sufficient, provided, the possessor makes proper
estate tax thereon while the same has not been occupied by any person shall be entitled, application therefor.The possession has to be confirmed judicially or administratively
under the provisions of this chapter, to have a free patent issued to him for such tract or after which a torrens title is issued.
tracts of such land not to exceed twenty-four hectares.
A torrens title recognizes the owner whose name appears in the certificate as
entitled to all the rights of ownership under the civil law. The Civil Code of the
A member of the national cultural minorities who has continuously occupied and Philippines defines ownership in Articles 427, 428 and 429. This concept is based on
cultivated, either by himself or through his predecessors-in-interest, a tract or tracts Roman Law which the Spaniards introduced to the Philippines through the Civil Code of
of land, whether disposable or not since July 4, 1955, shall be entitled to the right 1889. Ownership, under Roman Law, may be exercised over things or rights. It primarily
granted in the preceding paragraph of this section: Provided, That at the time he includes the right of the owner to enjoy and dispose of the thing owned. And the right to
files his free patent application he is not the owner of any real property secured or enjoy and dispose of the thing includes the right to receive from the thing what it
disposable under the provision of the Public Land Law. [203] produces,[205] the right to consume the thing by its use,[206] the right to alienate, encumber,
transform or even destroy the thing owned,[207] and the right to exclude from the
x x x.
possession of the thing owned by any other person to whom the owner has not domain.This is the reason why the ancestral domain must be kept within the ICCs/IPs
transmitted such thing.[208] themselves. The domain cannot be transferred, sold or conveyed to other persons. It
belongs to the ICCs/IPs as a community.
1. The Indigenous Concept of Ownership and Customary Law.
Ancestral lands are also held under the indigenous concept of ownership. The
Ownership of ancestral domains by native title does not entitle the ICC/IP to a lands are communal. These lands, however, may be transferred subject to the following
torrens title but to a Certificate of Ancestral Domain Title (CADT). The CADT formally limitations:(a) only to the members of the same ICCs/IPs; (b) in accord with customary
recognizes the indigenous concept of ownership of the ICCs/IPs over their ancestral laws and traditions; and (c) subject to the right of redemption of the ICCs/IPs for a period
domain. Thus: of 15 years if the land was transferred to a non-member of the ICCs/IPs.

"Sec. 5. Indigenous concept of ownership.- Indigenous concept of ownership sustains the Following the constitutional mandate that "customary law govern property rights or
view that ancestral domains and all resources found therein shall serve as the material relations in determining the ownership and extent of ancestral domains,"[216] the IPRA,
bases of their cultural integrity. The indigenous concept of ownership generally holds that by legislative fiat, introduces a new concept of ownership. This is a concept that has
ancestral domains are the ICCs/IPs private but community property which belongs to all long existed under customary law.[217]
generations and therefore cannot be sold, disposed or destroyed. It likewise covers Custom, from which customary law is derived, is also recognized under the
sustainable traditional resource rights." Civil Code as a source of law.[218] Some articles of the Civil Code expressly provide that
custom should be applied in cases where no codal provision is applicable. [219] In other
The right of ownership and possession of the ICCs/IPs to their ancestral words, in the absence of any applicable provision in the Civil Code, custom, when duly
domains is held under the indigenous concept of ownership. This concept maintains proven, can define rights and liabilities.[220]
the view that ancestral domains are the ICCs/IPs private but community
property. It is private simply because it is not part of the public domain. But its Customary law is a primary, not secondary, source of rights under the IPRA and
private character ends there. The ancestral domain is owned in common by the uniquely applies to ICCs/IPs. Its recognition does not depend on the absence of a
ICCs/IPs and not by one particular person. The IPRA itself provides that areas within specific provision in the civil law. The indigenous concept of ownership under
the ancestral domains, whether delineated or not, are presumed to be communally customary law is specifically acknowledged and recognized, and coexists with the civil
held.[209] These communal rights, however, are not exactly the same as co-ownership law concept and the laws on land titling and land registration. [221]
rights under the Civil Code.[210] Co-ownership gives any co-owner the right to demand To be sure, the indigenous concept of ownership exists even without a paper
partition of the property held in common. The Civil Code expressly provides that "[n]o title. The CADT is merely a "formal recognition" of native title. This is clear from
co-owner shall be obliged to remain in the co-ownership." Each co-owner may demand at Section 11 of the IPRA, to wit:
any time the partition of the thing in common, insofar as his share is concerned. [211] To
allow such a right over ancestral domains may be destructive not only of customary law
of the community but of the very community itself.[212] "Sec. 11. Recognition of Ancestral Domain Rights.-- The rights of ICCs/IPs to their
ancestral domains by virtue of Native Title shall be recognized and respected. Formal
Communal rights over land are not the same as corporate rights over real recognition, when solicited by ICCs/IPs concerned shall be embodied in a Certificate of
property, much less corporate condominium rights. A corporation can exist only for a Ancestral Domain Title, which shall recognize the title of the concerned ICCs/IPs over
maximum of fifty (50) years subject to an extension of another fifty years in any single the territories identified and delineated."
instance.[213] Every stockholder has the right to disassociate himself from the
corporation.[214] Moreover, the corporation itself may be dissolved voluntarily or The moral import of ancestral domain, native land or being native is
involuntarily.[215] "belongingness" to the land, being people of the land-- by sheer force of having sprung
Communal rights to the land are held not only by the present possessors of the from the land since time beyond recall, and the faithful nurture of the land by the sweat of
land but extends to all generations of the ICCs/IPs, past, present and future, to the one's brow. This is fidelity of usufructuary relation to the land-- the possession of
stewardship through perduring, intimate tillage, and the mutuality of blessings between e) Right to Regulate the Entry of Migrants.-- Right to regulate the entry of migrant
man and land; from man, care for land; from the land, sustenance for man. [222] settlers and organizations into their domains;

C. Sections 7 (a), 7 (b) and 57 of the IPRA Do Not Violate the Regalian
Doctrine Enshrined in Section 2, Article XII of the 1987 Constitution. f) Right to Safe and Clean Air and Water.--For this purpose, the ICCs/IPs shall have
access to integrated systems for the management of their inland waters and air space;
1. The Rights of ICCs/IPs Over Their Ancestral Domains and Lands
The IPRA grants the ICCs/IPs several rights over their ancestral domains and g) Right to Claim Parts of Reservations.-- The right to claim parts of the ancestral
ancestral lands. Section 7 provides for the rights over ancestral domains: domains which have been reserved for various purposes, except those reserved and
intended for common and public welfare and service;

"Sec. 7. Rights to Ancestral Domains.-- The rights of ownership and possession of


ICCs/IPs to their ancestral domains shall be recognized and protected. Such rights include: h) Right to Resolve Conflict.-- Right to resolve land conflicts in accordance with
customary laws of the area where the land is located, and only in default thereof shall the
complaints be submitted to amicable settlement and to the Courts of Justice whenever
a) Right of Ownership.- The right to claim ownership over lands, bodies of water necessary."
traditionally and actually occupied by ICCs/IPs, sacred places, traditional hunting
and fishing grounds, and all improvements made by them at any time within the
domains; Section 8 provides for the rights over ancestral lands:

b) Right to Develop Lands and Natural Resources.-- Subject to Section 56 hereof, the "Sec. 8. Rights to Ancestral Lands.-- The right of ownership and possession of the
right to develop, control and use lands and territories traditionally occupied, owned, ICCs/IPs to their ancestral lands shall be recognized and protected.
or used; to manage and conserve natural resources within the territories and uphold
the responsibilities for future generations; to benefit and share the profits from a) Right to transfer land/property.-- Such right shall include the right to transfer land or
allocation and utilization of the natural resources found therein; the right to property rights to/among members of the same ICCs/IPs, subject to customary laws and
negotiate the terms and conditions for the exploration of natural resources in the traditions of the community concerned.
areas for the purpose of ensuring ecological, environmental protection and the
conservation measures, pursuant to national and customary laws; the right to an b) Right to Redemption.-- In cases where it is shown that the transfer of land/property
informed and intelligent participation in the formulation and implementation of any rights by virtue of any agreement or devise, to a non-member of the concerned ICCs/IPs
project, government or private, that will affect or impact upon the ancestral domains and is tainted by the vitiated consent of the ICCs/IPs, or is transferred for an unconscionable
to receive just and fair compensation for any damages which they may sustain as a result consideration or price, the transferor ICC/IP shall have the right to redeem the same
of the project; and the right to effective measures by the government to prevent any within a period not exceeding fifteen (15) years from the date of transfer."
interference with, alienation and encroachment upon these rights;"
Section 7 (a) defines the ICCs/IPs the right of ownership over their
c) Right to Stay in the Territories.-- The right to stay in the territory and not to be ancestral domains which covers (a) lands, (b) bodies of water traditionally and actually
removed therefrom. No ICCs/IPs will be relocated without their free and prior informed occupied by the ICCs/IPs, (c) sacred places, (d) traditional hunting and fishing grounds,
consent, nor through any means other than eminent domain. x x x; and (e) all improvements made by them at any time within the domains. The right of
ownership includes the following rights: (1) the right to develop lands and natural
d) Right in Case of Displacement.-- In case displacement occurs as a result of natural resources; (b) the right to stay in the territories; (c) the right to resettlement in case of
catastrophes, the State shall endeavor to resettle the displaced ICCs/IPs in suitable areas displacement; (d) the right to regulate the entry of migrants; (e) the right to safe and clean
where they can have temporary life support systems: x x x;
air and water; (f) the right to claim parts of the ancestral domains as reservations; and (g) utilization of minerals, petroleum, and other mineral oils according to the general
the right to resolve conflict in accordance with customary laws. terms and conditions provided by law, based on real contributions to the economic
growth and general welfare of the country. In such agreements, the state shall promote the
Section 8 governs their rights to ancestral lands. Unlike ownership over the development and use of local scientific and technical resources.
ancestral domains, Section 8 gives the ICCs/IPs also the right to transfer the land or
property rights to members of the same ICCs/IPs or non-members thereof. This is in
keeping with the option given to ICCs/IPs to secure a torrens title over the ancestral lands, The President shall notify the Congress of every contract entered into in accordance with
but not to domains. this provision, within thirty days from its execution."[223]

2. The Right of ICCs/IPs to Develop Lands and Natural Resources Within the All lands of the public domain and all natural resources-- waters, minerals, coal,
Ancestral Domains Does Not Deprive the State of Ownership Over the Natural Resources petroleum, and other mineral oils, all forces of potential energy, fisheries, forests or
and Control and Supervision in their Development and Exploitation. timber, wildlife, flora and fauna, and other natural resources-- are owned by the
The Regalian doctrine on the ownership, management and utilization of natural State. The Constitution provides that in the exploration, development and utilization of
resources is declared in Section 2, Article XII of the 1987 Constitution, viz: these natural resources, the State exercises full control and supervision, and may
undertake the same in four (4) modes:
"Sec. 2. All lands of the public domain, waters, minerals, coal, petroleum, and other 1. The State may directly undertake such activities; or
mineral oils, all forces of potential energy, fisheries, forests or timber, wildlife, flora
and fauna, and other natural resources are owned by the State. With the exception of 2. The State may enter into co-production, joint venture or production-sharing
agricultural lands, all other natural resources shall not be alienated. The exploration, agreements with Filipino citizens or qualified corporations;
development, and utilization of natural resources shall be under the full control and 3. Congress may, by law, allow small-scale utilization of natural resources by
supervision of the State. The State may directly undertake such activities, or, it may Filipino citizens;
enter into co-production, joint venture, or production-sharing agreements with
Filipino citizens, or corporations or associations at least sixty per centum of whose 4. For the large-scale exploration, development and utilization of minerals,
capital is owned by such citizens. Such agreements may be for a period not exceeding petroleum and other mineral oils, the President may enter into agreements
twenty-five years, renewable for not more than twenty-five years, and under such terms with foreign-owned corporations involving technical or financial
and conditions as may be provided by law. In cases of water rights for irrigation, water assistance.
supply, fisheries, water supply, fisheries, or industrial uses other than the development of
water power, beneficial use may be the measure and limit of the grant. As owner of the natural resources, the State is accorded primary power and
responsibility in the exploration, development and utilization of these natural
resources. The State may directly undertake the exploitation and development by itself,
The State shall protect the nation's marine wealth in its archipelagic waters, territorial sea, or, it may allow participation by the private sector through co-production,[224] joint
and exclusive economic zone, and reserve its use and enjoyment exclusively to Filipino venture,[225] or production-sharing agreements.[226] These agreements may be for a period
citizens. of 25 years, renewable for another 25 years. The State, through Congress, may allow the
small-scale utilization of natural resources by Filipino citizens. For the large-scale
The Congress may, by law, allow small-scale utilization of natural resources by exploration of these resources, specifically minerals, petroleum and other mineral oils, the
Filipino citizens, as well as cooperative fish farming, with priority to subsistence State, through the President, may enter into technical and financial assistance agreements
fishermen and fishworkers in rivers, lakes, bays, and lagoons. with foreign-owned corporations.
Under the Philippine Mining Act of 1995, (R.A. 7942) and the People's Small-
The President may enter into agreements with foreign-owned corporations involving Scale Mining Act of 1991 (R.A. 7076) the three types of agreements, i.e., co-production,
either technical or financial assistance for large-scale exploration, development, and joint venture or production-sharing, may apply to both large-scale[227] and small-scale
mining.[228] "Small-scale mining" refers to "mining activities which rely heavily on Section 1 of the Implementing Rules gives the ICCs/IPs rights of ownership over "lands,
manual labor using simple implements and methods and do not use explosives or heavy waters and natural resources." The term "natural resources" is not one of those expressly
mining equipment."[229] mentioned in Section 7 (a) of the law. Our Constitution and jurisprudence clearly declare
that the right to claim ownership over land does not necessarily include the right to claim
Examining the IPRA, there is nothing in the law that grants to the ICCs/IPs ownership over the natural resources found on or under the land. [231] The IPRA itself
ownership over the natural resources within their ancestral domains. The right of makes a distinction between land and natural resources. Section 7 (a) speaks of the
ICCs/IPs in their ancestral domains includes ownership, but this "ownership" is right of ownership only over the land within the ancestral domain. It is Sections 7 (b)
expressly defined and limited in Section 7 (a) as: and 57 of the law that speak of natural resources, and these provisions, as shall be
discussed later, do not give the ICCs/IPs the right of ownership over these resources.
"Sec. 7. a) Right of ownership-- The right to claim ownership over lands, bodies of water
traditionally and actually occupied by ICCs/IPs, sacred places, traditional hunting and The constitutionality of Section 1, Part II, Rule III of the Implementing Rules was
fishing grounds, and all improvements made by them at any time within the domains;" not specifically and categorically challenged by petitioners. Petitioners actually assail the
constitutionality of the Implementing Rules in general. [232] Nevertheless, to avoid any
confusion in the implementation of the law, it is necessary to declare that the inclusion of
The ICCs/IPs are given the right to claim ownership over "lands, bodies of water "natural resources" in Section 1, Part II, Rule III of the Implementing Rules goes beyond
traditionally and actually occupied by ICCs/IPs, sacred places, traditional hunting and the parameters of Section 7 (b) of the law and is contrary to Section 2, Article XII of
fishing grounds, and all improvements made by them at any time within the domains." It the 1987 Constitution.
will be noted that this enumeration does not mention bodies of water not occupied by the
ICCs/IPs, minerals, coal, wildlife, flora and fauna in the traditional hunting (b) The Small-Scale Utilization of Natural Resources In Sec. 7 (b) of the IPRA
grounds, fish in the traditional fishing grounds, forests or timber in the sacred places, etc. Is Allowed Under Paragraph 3, Section 2 of Article XII of the Constitution.
and all other natural resources found within the ancestral domains. Indeed, the right of
ownership under Section 7 (a) does not cover Ownership over natural resources remain with the State and the IPRA in Section 7
"waters, minerals, coal, petroleum and other mineral oils, all forces of potential (b) merely grants the ICCs/IPs the right to manage them, viz:
energy, fisheries, forests or timber, wildlife, flora and fauna and all other natural
resources" enumerated in Section 2, Article XII of the 1987 Constitution as "Sec. 7 (b) Right to Develop Lands and Natural Resources.-- Subject to Section 56
belonging to the State. hereof, right to develop, control and use lands and territories traditionally occupied,
owned, or used; to manage and conserve natural resources within the territories and
The non-inclusion of ownership by the ICCs/IPs over the natural resources in uphold the responsibilities for future generations; to benefit and share the profits from
Section 7(a) complies with the Regalian doctrine. allocation and utilization of the natural resources found therein; the right to negotiate the
(a) Section 1, Part II, Rule III of the Implementing Rules Goes Beyond the terms and conditions for the exploration of natural resources in the areas for the purpose
Parameters of Sec. 7 (a) of the IPRA And is Unconstitutional. of ensuring ecological, environmental protection and the conservation measures, pursuant
to national and customary laws; the right to an informed and intelligent participation in
The Rules Implementing the IPRA[230] in Section 1, Part II, Rule III reads: the formulation and implementation of any project, government or private, that will affect
or impact upon the ancestral domains and to receive just and fair compensation for any
"Section 1. Rights of Ownership. ICCs/IPs have rights of ownership over lands, waters, damages which they may sustain as a result of the project; and the right to effective
and natural resources and all improvements made by them at any time within the measures by the government to prevent any interference with, alienation and
ancestral domains/ lands. These rights shall include, but not limited to, the right over the encroachment upon these rights;"
fruits, the right to possess, the right to use, right to consume, right to exclude and right to
recover ownership, and the rights or interests over land and natural resources. The right to The right to develop lands and natural resources under Section 7 (b) of the IPRA
recover shall be particularly applied to lands lost through fraud or any form or vitiated enumerates the following rights:
consent or transferred for an unconscionable price."
a) the right to develop, control and use lands and territories traditionally occupied; scale.Small-scale utilization of natural resources is expressly allowed in the third
paragraph of Section 2, Article XII of the Constitution "in recognition of the plight of
b) the right to manage and conserve natural resources within the territories and uphold the forest dwellers, gold panners, marginal fishermen and others similarly situated who
responsibilities for future generations; exploit our natural resources for their daily sustenance and survival."[235] Section 7 (b)
also expressly mandates the ICCs/IPs to manage and conserve these resources and ensure
environmental and ecological protection within the domains, which duties, by their very
c) the right to benefit and share the profits from the allocation and utilization of nature, necessarily reject utilization in a large-scale.
the natural resources found therein;
(c) The Large-Scale Utilization of Natural Resources In Section 57 of the IPRA
d) the right to negotiate the terms and conditions for the exploration of natural Is Allowed Under Paragraphs 1 and 4, Section 2, Article XII of the 1987
resources for the purpose of ensuring ecological, environmental protection and the Constitution.
conservation measures, pursuant to national and customary laws; Section 57 of the IPRA provides:

e) the right to an informed and intelligent participation in the formulation and "Sec. 57. Natural Resources within Ancestral Domains.-- The ICCs/IPs shall
implementation of any project, government or private, that will affect or impact upon the have priority rights in the harvesting, extraction, development or exploitation of any
ancestral domains and to receive just and fair compensation for any damages which they natural resources within the ancestral domains. A non-member of the ICCs/IPs
may sustain as a result of the project; concerned may be allowed to take part in the development and utilization of the natural
resources for a period of not exceeding twenty-five (25) years renewable for not more
f) the right to effective measures by the government to prevent any interference with, than twenty-five (25) years: Provided, That a formal and written agreement is entered
alienation and encroachment upon these rights.[233] into with the ICCs/IPs concerned or that the community, pursuant to its own decision-
making process, has agreed to allow such operation: Provided finally, That the NCIP may
Ownership over the natural resources in the ancestral domains remains with the exercise visitorial powers and take appropriate action to safeguard the rights of the
State and the ICCs/IPs are merely granted the right to "manage and conserve" ICCs/IPs under the same contract."
them for future generations, "benefit and share" the profits from their allocation
and utilization, and "negotiate the terms and conditions for their exploration" for Section 57 speaks of the "harvesting, extraction, development or exploitation of
the purpose of "ensuring ecological and environmental protection and conservation natural resources within ancestral domains" and "gives the ICCs/IPs 'priority rights'
measures." It must be noted that the right to negotiate the terms and conditions over the therein." The terms "harvesting, extraction, development or exploitation" of any
natural resources covers only their exploration which must be for the purpose of ensuring natural resources within the ancestral domains obviously refer to large-scale
ecological and environmental protection of, and conservation measures in the ancestral utilization. It is utilization not merely for subsistence but for commercial or other
domain. It does not extend to the exploitation and development of natural resources. extensive use that require technology other than manual labor.[236] The law recognizes the
probability of requiring a non-member of the ICCs/IPs to participate in the development
Simply stated, the ICCs/IPs' rights over the natural resources take the form of and utilization of the natural resources and thereby allows such participation for a period
management or stewardship. For the ICCs/IPs may use these resources and share in the of not more than 25 years, renewable for another 25 years. This may be done on condition
profits of their utilization or negotiate the terms for their exploration. At the same time, that a formal written agreement be entered into by the non-member and members of the
however, the ICCs/IPs must ensure that the natural resources within their ancestral ICCs/IPs.
domains are conserved for future generations and that the "utilization" of these resources
must not harm the ecology and environment pursuant to national and customary laws. [234] Section 57 of the IPRA does not give the ICCs/IPs the right to "manage and
conserve" the natural resources. Instead, the law only grants the ICCs/IPs "priority rights"
The limited rights of "management and use" in Section 7 (b) must be taken to in the development or exploitation thereof. Priority means giving preference. Having
contemplate small-scale utilization of natural resources as distinguished from large- priority rights over the natural resources does not necessarily mean ownership rights. The
grant of priority rights implies that there is a superior entity that owns these resources and supervision or control. This indicates that the State does not lose control and ownership
this entity has the power to grant preferential rights over the resources to whosoever itself over the resources even in their exploitation.Sections 7 (b) and 57 of the law simply give
chooses. due respect to the ICCs/IPs who, as actual occupants of the land where the natural
resources lie, have traditionally utilized these resources for their subsistence and survival.
Section 57 is not a repudiation of the Regalian doctrine. Rather, it is an affirmation
of the said doctrine that all natural resources found within the ancestral domains belong Neither is the State stripped of ownership and control of the natural resources by
to the State.It incorporates by implication the Regalian doctrine, hence, requires that the the following provision:
provision be read in the light of Section 2, Article XII of the 1987
Constitution. Interpreting Section 2, Article XII of the 1987 Constitution[237] in "Section 59. Certification Precondition.-- All departments and other governmental
relation to Section 57 of IPRA, the State, as owner of these natural resources, may agencies shall henceforth be strictly enjoined from issuing, renewing or granting any
directly undertake the development and exploitation of the natural resources by concession, license or lease, or entering into any production-sharing agreement. without
itself, or in the alternative, it may recognize the priority rights of the ICCs/IPs as prior certification from the NCIP that the area affected does not overlap with any
owners of the land on which the natural resources are found by entering into a co- ancestral domain. Such certification shall only be issued after a field-based investigation
production, joint venture, or production-sharing agreement with them. The State is conducted by the Ancestral Domains Office of the area concerned: Provided, That no
may likewise enter into any of said agreements with a non-member of the ICCs/IPs, certification shall be issued by the NCIP without the free and prior informed and written
whether natural or juridical, or enter into agreements with foreign-owned consent of the ICCs/IPs concerned: Provided, further, That no department, government
corporations involving either technical or financial assistance for the large-scale agency or government-owned or -controlled corporation may issue new concession,
exploration, development and utilization of minerals, petroleum, and other mineral license, lease, or production sharing agreement while there is a pending application for a
oils, or allow such non-member to participate in its agreement with the ICCs/IPs. If CADT: Provided, finally, That the ICCs/IPs shall have the right to stop or suspend, in
the State decides to enter into an agreement with a non-ICC/IP member, the National accordance with this Act, any project that has not satisfied the requirement of this
Commission on Indigenous Peoples (NCIP) shall ensure that the rights of the ICCs/IPs consultation process."
under the agreement shall be protected. The agreement shall be for a period of 25 years,
renewable for another 25 years.
Concessions, licenses, lease or production-sharing agreements for the exploitation of
To reiterate, in the large-scale utilization of natural resources within the ancestral natural resources shall not be issued, renewed or granted by all departments and
domains, the State, as owner of these resources, has four (4) options: (1) it may, of and by government agencies without prior certification from the NCIP that the area subject of the
itself, directly undertake the development and exploitation of the natural resources; or (2) agreement does not overlap with any ancestral domain. The NCIP certification shall be
it may recognize the priority rights of the ICCs/IPs by entering into an agreement with issued only after a field-based investigation shall have been conducted and the free and
them for such development and exploitation; or (3) it may enter into an agreement with a prior informed written consent of the ICCs/IPs obtained. Non-compliance with the
non-member of the ICCs/IPs, whether natural or juridical, local or foreign; or (4) it may consultation requirement gives the ICCs/IPs the right to stop or suspend any project
allow such non-member to participate in the agreement with the ICCs/IPs. granted by any department or government agency.
The rights granted by the IPRA to the ICCs/IPs over the natural resources in As its subtitle suggests, this provision requires as a precondition for the issuance of
their ancestral domains merely gives the ICCs/IPs, as owners and occupants of the any concession, license or agreement over natural resources, that a certification be issued
land on which the resources are found, the right to the small-scale utilization of by the NCIP that the area subject of the agreement does not lie within any ancestral
these resources, and at the same time, a priority in their large-scale development domain. The provision does not vest the NCIP with power over the other agencies of the
and exploitation.Section 57 does not mandate the State to automatically give priority State as to determine whether to grant or deny any concession or license or agreement. It
to the ICCs/IPs. The State has several options and it is within its discretion to choose merely gives the NCIP the authority to ensure that the ICCs/IPs have been informed of
which option to pursue. Moreover, there is nothing in the law that gives the ICCs/IPs the agreement and that their consent thereto has been obtained. Note that the certification
the right to solely undertake the large-scale development of the natural resources within applies to agreements over natural resources that do not necessarily lie within the
their domains. The ICCs/IPs must undertake such endeavour always under State
ancestral domains. For those that are found within the said domains, Sections 7(b) and 57 ILO Convention No. 169 is entitled the "Convention Concerning Indigenous and
of the IPRA apply. Tribal Peoples in Independent Countries"[248] and was adopted on June 27, 1989. It is
based on the Universal Declaration of Human Rights, the International Covenant on
V. THE IPRA IS A RECOGNITION OF OUR ACTIVE PARTICIPATION Economic, Social and Cultural Rights, the International Covenant on Civil and Political
IN THE INDIGENOUS INTERNATIONAL MOVEMENT. Rights, and many other international instruments on the prevention of
The indigenous movement can be seen as the heir to a history of anti-imperialism discrimination.[249] ILO Convention No. 169 revised the "Convention Concerning the
stretching back to prehistoric times. The movement received a massive impetus during Protection and Integration of Indigenous and Other Tribal and Semi-Tribal Populations in
the 1960's from two sources. First, the decolonization of Asia and Africa brought into the Independent Countries" (ILO No. 107) passed on June 26, 1957. Developments in
limelight the possibility of peoples controlling their own destinies. Second, the right of international law made it appropriate to adopt new international standards on indigenous
self-determination was enshrined in the UN Declaration on Human Rights.[238] The rise of peoples "with a view to removing the assimilationist orientation of the earlier standards,"
the civil rights movement and anti-racism brought to the attention of North American and recognizing the aspirations of these peoples to exercise control over their own
Indians, Aborigines in Australia, and Maori in New Zealand the possibility of fighting for institutions, ways of life and economic development."[250]
fundamental rights and freedoms.
In 1974 and 1975, international indigenous organizations were founded,[239] and
during the 1980's, indigenous affairs were on the international agenda. The people of the CONCLUSION
Philippine Cordillera were the first Asians to take part in the international indigenous
movement. It was the Cordillera People's Alliance that carried out successful campaigns
against the building of the Chico River Dam in 1981-82 and they have since become one The struggle of the Filipinos throughout colonial history had been plagued by
of the best-organized indigenous bodies in the world.[240] ethnic and religious differences. These differences were carried over and magnified by
the Philippine government through the imposition of a national legal order that is mostly
Presently, there is a growing concern for indigenous rights in the international foreign in origin or derivation.[251] Largely unpopulist, the present legal system has
scene. This came as a result of the increased publicity focused on the continuing resulted in the alienation of a large sector of society, specifically, the indigenous
disrespect for indigenous human rights and the destruction of the indigenous peoples' peoples. The histories and cultures of the indigenes are relevant to the evolution of
environment, together with the national governments' inability to deal with the Philippine culture and are vital to the understanding of contemporary problems.[252] It is
situation.[241] Indigenous rights came as a result of both human rights and environmental through the IPRA that an attempt was made by our legislators to understand Filipino
protection, and have become a part of today's priorities for the international agenda. [242] society not in terms of myths and biases but through common experiences in the course
of history. The Philippines became a democracy a centennial ago and the decolonization
International institutions and bodies have realized the necessity of applying policies, process still continues. If the evolution of the Filipino people into a democratic society is
programs and specific rules concerning IPs in some nations. The World Bank, for to truly proceed democratically, i.e., if the Filipinos as a whole are to participate fully in
example, first adopted a policy on IPs as a result of the dismal experience of projects in the task of continuing democratization,[253] it is this Court's duty to acknowledge the
Latin America.[243] The World Bank now seeks to apply its current policy on IPs to some presence of indigenous and customary laws in the country and affirm their co-existence
of its projects in Asia. This policy has provided an influential model for the projects of with the land laws in our national legal system.
the Asian Development Bank.[244]
With the foregoing disquisitions, I vote to uphold the constitutionality of the
The 1987 Philippine Constitution formally recognizes the existence of ICCs/IPs Indigenous Peoples Rights Act of 1997.
and declares as a State policy the promotion of their rights within the framework of
national unity and development.[245] The IPRA amalgamates the Philippine category of
ICCs with the international category of IPs,[246] and is heavily influenced by both the
International Labor Organization (ILO) Convention 169 and the United Nations (UN)
Draft Declaration on the Rights of Indigenous Peoples.[247]
Republic of the Philippines SITUATED NAMED IN A LIST,
Supreme Court ANNEX A OF THIS PETITION,
Manila Petitioners,

EN BANC - versus -

THE SECRETARY OF THE G.R. No. 167707 THE SECRETARY OF THE


DEPARTMENT OF ENVIRONMENT DEPARTMENT OF ENVIRONMENT
AND NATURAL RESOURCES, THE AND NATURAL RESOURCES, THE
REGIONAL EXECUTIVE Present: REGIONAL TECHNICAL
DIRECTOR, DENR-REGION VI, DIRECTOR FOR LANDS, LANDS
REGIONAL TECHNICAL PUNO, C.J., MANAGEMENT BUREAU,
DIRECTOR FOR LANDS, QUISUMBING, REGION VI, PROVINCIAL
LANDS MANAGEMENT BUREAU, YNARES-SANTIAGO, ENVIRONMENT AND NATURAL
REGION VI PROVINCIAL CARPIO, RESOURCES OFFICER, KALIBO,
ENVIRONMENT AND NATURAL AUSTRIA-MARTINEZ, AKLAN,
RESOURCES OFFICER OF KALIBO, CORONA,* Respondents.
AKLAN, REGISTER OF DEEDS, CARPIO MORALES,
DIRECTOR OF LAND AZCUNA, x--------------------------------------------------x
REGISTRATION AUTHORITY, TINGA,
DEPARTMENT OF TOURISM CHICO-NAZARIO, DECISION
SECRETARY, DIRECTOR OF VELASCO, JR.,
PHILIPPINE TOURISM NACHURA,** REYES, R.T., J.:
AUTHORITY, REYES,
Petitioners, LEONARDO-DE CASTRO, and AT stake in these consolidated cases is the right of the present occupants
BRION, JJ. of Boracay Island to secure titles over their occupied lands.
- versus -
There are two consolidated petitions. The first is G.R. No. 167707, a petition
MAYOR JOSE S. YAP, LIBERTAD for review on certiorari of the Decision[1] of the Court of Appeals (CA) affirming
TALAPIAN, MILA Y. SUMNDAD, and that[2] of the Regional Trial Court (RTC) in Kalibo, Aklan, which granted the petition for
ANICETO YAP, in their behalf and Promulgated: declaratory relief filed by respondents-claimants Mayor Jose Yap, et al. and ordered the
in behalf of all those similarly situated, survey of Boracay for titling purposes. The second is G.R. No. 173775, a petition for
Respondents. October 8, 2008 prohibition, mandamus, and nullification of Proclamation No. 1064 [3] issued by President
Gloria Macapagal-Arroyo classifying Boracay into reserved forest and agricultural land.
x--------------------------------------------------x
The Antecedents
DR. ORLANDO SACAY and G.R. No. 173775
WILFREDO GELITO, joined by G.R. No. 167707
THE LANDOWNERS OF
BORACAY SIMILARLY
Boracay Island in the Municipality of Malay, Aklan, with its powdery white The OSG maintained that respondents-claimants reliance on PD No. 1801
sand beaches and warm crystalline waters, is reputedly a premier Philippine tourist and PTA Circular No. 3-82 was misplaced. Their right to judicial confirmation of title
destination.The island is also home to 12,003 inhabitants[4] who live in the bone-shaped was governed by CA No. 141 and PD No. 705. Since Boracay Island had not been
islands three barangays.[5] classified as alienable and disposable, whatever possession they had cannot ripen into
ownership.
On April 14, 1976, the Department of Environment and Natural
Resources (DENR) approved the National Reservation Survey of Boracay During pre-trial, respondents-claimants and the OSG stipulated on the
Island,[6] which identified several lots as being occupied or claimed by named persons. [7] following facts: (1) respondents-claimants were presently in possession of parcels of land
in Boracay Island; (2) these parcels of land were planted with coconut trees and other
On November 10, 1978, then President Ferdinand Marcos issued Proclamation natural growing trees; (3) the coconut trees had heights of more or less twenty (20)
No. 1801[8] declaring Boracay Island, among other islands, caves and peninsulas in meters and were planted more or less fifty (50) years ago; and (4) respondents-claimants
the Philippines, as tourist zones and marine reserves under the administration of the declared the land they were occupying for tax purposes.[12]
Philippine Tourism Authority (PTA). President Marcos later approved the issuance
of PTA Circular 3-82[9] dated September 3, 1982, to implement Proclamation No. 1801. The parties also agreed that the principal issue for resolution was purely legal:
whether Proclamation No. 1801 posed any legal hindrance or impediment to the titling of
Claiming that Proclamation No. 1801 and PTA Circular No 3-82 precluded the lands in Boracay. They decided to forego with the trial and to submit the case for
them from filing an application for judicial confirmation of imperfect title or survey of resolution upon submission of their respective memoranda.[13]
land for titling purposes, respondents-claimants
Mayor Jose S. Yap, Jr., Libertad Talapian, Mila Y. Sumndad, and Aniceto Yap filed a The RTC took judicial notice[14] that certain parcels of land in Boracay Island,
petition for declaratory relief with the RTC in Kalibo, Aklan. more particularly Lots 1 and 30, Plan PSU-5344, were covered by Original Certificate of
Title No. 19502 (RO 2222) in the name of the Heirs of Ciriaco S. Tirol. These lots were
In their petition, respondents-claimants alleged that Proclamation No. 1801 involved in Civil Case Nos. 5222 and 5262 filed before
and PTA Circular No. 3-82 raised doubts on their right to secure titles over their occupied the RTC of Kalibo, Aklan.[15]The titles were issued on
lands. They declared that they themselves, or through their predecessors-in-interest, had August 7, 1933.[16]
been in open, continuous, exclusive, and notorious possession and occupation in Boracay
since June 12, 1945, or earlier since time immemorial. They declared their lands for tax RTC and CA Dispositions
purposes and paid realty taxes on them.[10]
On July 14, 1999, the RTC rendered a decision in favor of respondents-
Respondents-claimants posited that Proclamation No. 1801 and its claimants, with a fallo reading:
implementing Circular did not place Boracay beyond the commerce of man. Since
the Island was classified as a tourist zone, it was susceptible of private ownership. Under WHEREFORE, in view of the foregoing, the Court
Section 48(b) of Commonwealth Act (CA) No. 141, otherwise known as the Public Land declares that Proclamation No. 1801 and PTA Circular No. 3-82 pose
Act, they had the right to have the lots registered in their names through judicial no legal obstacle to the petitioners and those similarly situated to
confirmation of imperfect titles. acquire title to their lands in Boracay, in accordance with the
applicable laws and in the manner prescribed therein; and to have
The Republic, through the Office of the Solicitor General (OSG), opposed the their lands surveyed and approved by respondent Regional Technical
petition for declaratory relief. The OSG countered that Boracay Island was Director of Lands as the approved survey does not in itself constitute
an unclassified land of the public domain. It formed part of the mass of lands classified a title to the land.
as public forest, which was not available for disposition pursuant to Section 3(a) of
Presidential Decree (PD) No. 705 or the Revised Forestry Code, [11] as amended. SO ORDERED.[17]
time immemorial. They have also invested billions of pesos in developing their lands and
The RTC upheld respondents-claimants right to have their occupied lands titled building internationally renowned first class resorts on their lots.[31]
in their name. It ruled that neither Proclamation No. 1801 nor PTA Circular No. 3-82
mentioned that lands in Boracay were inalienable or could not be the subject of
disposition.[18] The Circular itself recognized private ownership of lands.[19] The trial Petitioners-claimants contended that there is no need for a proclamation
court cited Sections 87[20] and 53[21] of the Public Land Act as basis for acknowledging reclassifying Boracay into agricultural land. Being classified as neither mineral nor
private ownership of lands in Boracay and that only those forested areas in public lands timber land, the island is deemed agricultural pursuant to the Philippine Bill of 1902 and
were declared as part of the forest reserve.[22] Act No. 926, known as the first Public Land Act.[32] Thus, their possession in the concept
of owner for the required period entitled them to judicial confirmation of imperfect title.
The OSG moved for reconsideration but its motion was denied.[23] The
Republic then appealed to the CA. Opposing the petition, the OSG argued that petitioners-claimants do not have a
vested right over their occupied portions in the island. Boracay is an unclassified public
On December 9, 2004, the appellate court affirmed in toto the RTC decision, forest land pursuant to Section 3(a) of PD No. 705. Being public forest, the claimed
disposing as follows: portions of the island are inalienable and cannot be the subject of judicial confirmation of
imperfect title. It is only the executive department, not the courts, which has authority to
WHEREFORE, in view of the foregoing premises, reclassify lands of the public domain into alienable and disposable lands. There is a need
judgment is hereby rendered by us DENYING the appeal filed in this for a positive government act in order to release the lots for disposition.
case and AFFIRMING the decision of the lower court.[24]
On November 21, 2006, this Court ordered the consolidation of the two
The CA held that respondents-claimants could not be prejudiced by a petitions as they principally involve the same issues on the land classification
declaration that the lands they occupied since time immemorial were part of a forest of Boracay Island.[33]
reserve.
Again, the OSG sought reconsideration but it was similarly denied.[25] Hence, Issues
the present petition under Rule 45.
G.R. No. 167707
G.R. No. 173775 The OSG raises the lone issue of whether Proclamation No. 1801
and PTA Circular No. 3-82 pose any legal obstacle for respondents, and all those
On May 22, 2006, during the pendency of G.R. No. 167707, President Gloria Macapagal- similarly situated, to acquire title to their occupied lands in Boracay Island.[34]
Arroyo issued Proclamation No. 1064[26] classifying Boracay Island into four hundred
(400) hectares of reserved forest land (protection purposes) and six hundred twenty-eight G.R. No. 173775
and 96/100 (628.96) hectares of agricultural land (alienable and disposable). The Petitioners-claimants hoist five (5) issues, namely:
Proclamation likewise provided for a fifteen-meter buffer zone on each side of the I.
centerline of roads and trails, reserved for right-of-way and which shall form part of the AT THE TIME OF THE ESTABLISHED POSSESSION OF
area reserved for forest land protection purposes. PETITIONERS IN CONCEPT OF OWNER OVER THEIR
RESPECTIVE AREAS IN BORACAY, SINCE TIME
On August 10, 2006, petitioners-claimants Dr. Orlando Sacay,[27] Wilfredo IMMEMORIAL OR AT THE LATEST SINCE 30 YRS. PRIOR TO
Gelito,[28]and other landowners[29] in Boracay filed with this Court an original petition THE FILING OF THE PETITION FOR DECLARATORY RELIEF
for prohibition, mandamus, and nullification of Proclamation No. 1064.[30] They ON NOV. 19, 1997, WERE THE AREAS OCCUPIED BY THEM
allege that the Proclamation infringed on their prior vested rights over portions of PUBLIC AGRICULTURAL LANDS AS DEFINED
Boracay. They have been in continued possession of their respective lots in Boracay since BY LAWS THEN ON JUDICIAL CONFIRMATION OF
IMPERFECT TITLES OR PUBLIC FOREST AS DEFINED to reclassify lands of the public domain
BY SEC. 3a, PD 705?
Private claimants rely on three (3) laws and executive acts in their bid for
II. judicial confirmation of imperfect title, namely: (a) Philippine Bill of 1902 [36] in relation
HAVE PETITIONERS OCCUPANTS ACQUIRED PRIOR to Act No. 926, later amended and/or superseded by Act No. 2874 and CA No. 141;[37] (b)
VESTED RIGHT OF PRIVATE OWNERSHIP OVER THEIR Proclamation No. 1801[38] issued by then President Marcos; and (c) Proclamation No.
OCCUPIED PORTIONS OF BORACAY LAND, DESPITE 1064[39] issued by President Gloria Macapagal-Arroyo. We shall proceed to determine
THE FACT THAT THEY HAVE NOT APPLIED YET FOR their rights to apply for judicial confirmation of imperfect title under these laws and
JUDICIAL CONFIRMATION OF IMPERFECT TITLE? executive acts.

III. But first, a peek at the Regalian principle and the power of the executive to
IS THE EXECUTIVE DECLARATION OF THEIR AREAS AS reclassify lands of the public domain.
ALIENABLE AND DISPOSABLE UNDER SEC 6, CA 141 [AN]
INDISPENSABLE PRE-REQUISITE FOR PETITIONERS TO The 1935 Constitution classified lands of the public domain into agricultural,
OBTAIN TITLE UNDER THE TORRENS SYSTEM? forest or timber.[40] Meanwhile, the 1973 Constitution provided the following divisions:
agricultural, industrial or commercial, residential, resettlement, mineral, timber or forest
IV. and grazing lands, and such other classes as may be provided by law, [41] giving the
IS THE ISSUANCE OF PROCLAMATION 1064 ON MAY 22, government great leeway for classification.[42] Then the 1987 Constitution reverted to the
2006, VIOLATIVE OF THE PRIOR VESTED RIGHTS TO 1935 Constitution classification with one addition: national parks.[43] Of
PRIVATE OWNERSHIP OF PETITIONERS OVER THEIR these, onlyagricultural lands may be alienated.[44] Prior to Proclamation No. 1064 of May
LANDS IN BORACAY, PROTECTED BY THE DUE PROCESS 22, 2006, Boracay Island had never been expressly and administratively classified under
CLAUSE OF THE CONSTITUTION OR IS PROCLAMATION any of these grand divisions. Boracay was an unclassified land of the public domain.
1064 CONTRARY TO SEC. 8, CA 141, OR SEC. 4(a) OF RA 6657.
The Regalian Doctrine dictates that all lands of the public domain belong to the
V. State, that the State is the source of any asserted right to ownership of land and charged
CAN RESPONDENTS BE COMPELLED BY MANDAMUS TO with the conservation of such patrimony.[45] The doctrine has been consistently adopted
ALLOW THE SURVEY AND TO APPROVE THE SURVEY under the 1935, 1973, and 1987 Constitutions.[46]
PLANS FOR PURPOSES OF THE APPLICATION FOR TITLING
OF THE LANDS OF PETITIONERS IN All lands not otherwise appearing to be clearly within private ownership are
BORACAY?[35] (Underscoring supplied) presumed to belong to the State.[47] Thus, all lands that have not been acquired from the
government, either by purchase or by grant, belong to the State as part of the inalienable
In capsule, the main issue is whether private claimants (respondents-claimants public domain.[48] Necessarily, it is up to the State to determine if lands of the public
in G.R. No. 167707 and petitioners-claimants in G.R. No. 173775) have a right to secure domain will be disposed of for private ownership. The government, as the agent of the
titles over their occupied portions in Boracay. The twin petitions pertain to their right, if state, is possessed of the plenary power as the persona in law to determine who shall be
any, to judicial confirmation of imperfect title under CA No. 141, as amended. They do the favored recipients of public lands, as well as under what terms they may be granted
not involve their right to secure title under other pertinent laws. such privilege, not excluding the placing of obstacles in the way of their exercise of what
otherwise would be ordinary acts of ownership.[49]
Our Ruling
Our present land law traces its roots to the Regalian Doctrine. Upon the
Regalian Doctrine and power of the executive Spanish conquest of the Philippines, ownership of all lands, territories and possessions in
the Philippines passed to the Spanish Crown.[50] The Regalian doctrine was first
introduced in the Philippines through the Laws of the Indies and the Royal Cedulas, On February 1, 1903, the Philippine Legislature passed Act No. 496, otherwise
which laid the foundation that all lands that were not acquired from the Government, known as the Land Registration Act. The act established a system of registration by
either by purchase or by grant, belong to the public domain. [51] which recorded title becomes absolute, indefeasible, and imprescriptible. This is known
as the Torrens system.[66]
The Laws of the Indies was followed by the Ley Hipotecaria or the Mortgage
Law of 1893. The Spanish Mortgage Law provided for the systematic registration of titles Concurrently, on October 7, 1903, the Philippine Commission passed Act
and deeds as well as possessory claims.[52] No. 926, which was the first Public Land Act. The Act introduced the homestead system
and made provisions for judicial and administrative confirmation of imperfect titles and
The Royal Decree of 1894 or the Maura Law[53] partly amended the Spanish for the sale or lease of public lands. It permitted corporations regardless of the nationality
Mortgage Law and the Laws of the Indies. It established possessory information as the of persons owning the controlling stock to lease or purchase lands of the public
method of legalizing possession of vacant Crown land, under certain conditions which domain.[67] Under the Act, open, continuous, exclusive, and notorious possession and
were set forth in said decree.[54] Under Section 393 of the Maura Law, an informacion occupation of agricultural lands for the next ten (10) years preceding July 26, 1904 was
posesoria or possessory information title,[55] when duly inscribed in the Registry of sufficient for judicial confirmation of imperfect title.[68]
Property, is converted into a title of ownership only after the lapse of twenty (20) years of
uninterrupted possession which must be actual, public, and adverse, [56] from the date of On November 29, 1919, Act No. 926 was superseded by Act No. 2874,
its inscription.[57] However, possessory information title had to be perfected one year after otherwise known as the second Public Land Act. This new, more comprehensive law
the promulgation of the Maura Law, or until April 17, 1895. Otherwise, the lands would limited the exploitation of agricultural lands to Filipinos and Americans and citizens of
revert to the State.[58] other countries which gave Filipinos the same privileges. For judicial confirmation of title,
possession and occupation en concepto dueo since time immemorial, or since July 26,
In sum, private ownership of land under the Spanish regime could only be 1894, was required.[69]
founded on royal concessions which took various forms, namely: (1) titulo real or royal
grant; (2) concesion especial or special grant; (3) composicion con el estado or After the passage of the 1935 Constitution, CA No. 141 amended Act No. 2874
adjustment title; (4) titulo de compra or title by purchase; and (5) informacion on December 1, 1936. To this day, CA No. 141, as amended, remains as the existing
posesoria or possessory information title.[59] general law governing the classification and disposition of lands of the public domain
other than timber and mineral lands,[70] and privately owned lands which reverted to the
The first law governing the disposition of public lands in the Philippines under State.[71]
American rule was embodied in the Philippine Bill of 1902.[60] By this law, lands of the
public domain in the Philippine Islands were classified into three (3) grand divisions, to Section 48(b) of CA No. 141 retained the requirement under Act No. 2874 of
wit: agricultural, mineral, and timber or forest lands.[61] The act provided for, among possession and occupation of lands of the public domain since time immemorial or
others, the disposal of mineral lands by means of absolute grant (freehold system) and by since July 26, 1894. However, this provision was superseded by Republic Act (RA) No.
lease (leasehold system).[62] It also provided the definition by exclusion of agricultural 1942,[72] which provided for a simple thirty-year prescriptive period for judicial
public lands.[63] Interpreting the meaning of agricultural lands under the Philippine Bill of confirmation of imperfect title. The provision was last amended by PD No.
1902, the Court declared in Mapa v. Insular Government:[64] 1073,[73] which now provides for possession and occupation of the land applied
for since June 12, 1945, or earlier.[74]

x x x In other words, that the phrase agricultural land as The issuance of PD No. 892[75] on February 16, 1976 discontinued the use of
used in Act No. 926 means those public lands acquired Spanish titles as evidence in land registration proceedings.[76] Under the decree, all
from Spain which are not timber or mineral lands. x x holders of Spanish titles or grants should apply for registration of their lands under Act
x[65](Emphasis Ours) No. 496 within six (6) months from the effectivity of the decree on February 16,
1976. Thereafter, the recording of all unregistered lands[77] shall be governed by Section Ankron and De Aldecoa did not make the whole of Boracay Island, or
194 of the Revised Administrative Code, as amended by Act No. 3344. portions of it, agricultural lands. Private claimants posit that Boracay was already an
agricultural land pursuant to the old cases Ankron v. Government of the
On June 11, 1978, Act No. 496 was amended and updated by PD No. 1529, Philippine Islands (1919)[88] and De Aldecoa v. The Insular Government (1909).[89] These
known as the Property Registration Decree. It was enacted to codify the various laws cases were decided under the provisions of the Philippine Bill of 1902 and Act No.
relative to registration of property.[78] It governs registration of lands under 926. There is a statement in these old cases that in the absence of evidence to the contrary,
the Torrens system as well as unregistered lands, including chattel mortgages.[79] that in each case the lands are agricultural lands until the contrary is shown.[90]

A positive act declaring land as alienable and disposable is required. In Private claimants reliance on Ankron and De Aldecoa is misplaced. These cases
keeping with the presumption of State ownership, the Court has time and again did not have the effect of converting the whole of Boracay Island or portions of it into
emphasized that there must be a positive act of the government, such as an official agricultural lands. It should be stressed that the Philippine Bill of 1902 and Act No. 926
proclamation,[80] declassifying inalienable public land into disposable land for merely provided the manner through which land registration courts would classify lands
agricultural or other purposes.[81] In fact, Section 8 of CA No. 141 limits alienable or of the public domain. Whether the land would be classified as timber, mineral, or
disposable lands only to those lands which have been officially delimited and agricultural depended on proof presented in each case.
classified.[82]
Ankron and De Aldecoa were decided at a time when the President of the
The burden of proof in overcoming the presumption of State ownership of the Philippines had no power to classify lands of the public domain into mineral, timber, and
lands of the public domain is on the person applying for registration (or claiming agricultural. At that time, the courts were free to make corresponding classifications in
ownership), who must prove that the land subject of the application is alienable or justiciable cases, or were vested with implicit power to do so, depending upon the
disposable.[83] To overcome this presumption, incontrovertible evidence must be preponderance of the evidence.[91] This was the Courts ruling in Heirs of the Late Spouses
established that the land subject of the application (or claim) is alienable or Pedro S. Palanca and Soterranea Rafols Vda. De Palanca v. Republic,[92] in which it
disposable.[84] There must still be a positive act declaring land of the public domain as stated, through Justice Adolfo Azcuna, viz.:
alienable and disposable. To prove that the land subject of an application for registration
is alienable, the applicant must establish the existence of a positive act of the government x x x Petitioners furthermore insist that a particular land
such as a presidential proclamation or an executive order; an administrative action; need not be formally released by an act of the Executive before it can
investigation reports of Bureau of Lands investigators; and a legislative act or a be deemed open to private ownership, citing the cases of Ramos v.
statute.[85] The applicant may also secure a certification from the government that the land Director of Lands and Ankron v. Government of the
claimed to have been possessed for the required number of years is alienable and Philippine Islands.
disposable.[86]
xxxx
In the case at bar, no such proclamation, executive order, administrative action,
report, statute, or certification was presented to the Court. The records are bereft of Petitioners reliance upon Ramos v. Director of
evidence showing that, prior to 2006, the portions of Boracay occupied by private Lands and Ankron v. Government is misplaced. These cases were
claimants were subject of a government proclamation that the land is alienable and decided under the Philippine Bill of 1902 and the first Public Land
disposable.Absent such well-nigh incontrovertible evidence, the Court cannot accept the Act No. 926 enacted by the Philippine Commission on October 7,
submission that lands occupied by private claimants were already open to disposition 1926, under which there was no legal provision vesting in the Chief
before 2006. Matters of land classification or reclassification cannot be assumed. They Executive or President of the Philippines the power to classify lands
call for proof.[87] of the public domain into mineral, timber and agricultural so that the
courts then were free to make corresponding classifications in
justiciable cases, or were vested with implicit power to do so, we have just said, many definitions have been given for agriculture,
depending upon the preponderance of the evidence.[93] forestry, and mineral lands, and that in each case it is a question of
fact, we think it is safe to say that in order to be forestry or mineral
To aid the courts in resolving land registration cases under Act No. 926, it was land the proof must show that it is more valuable for the forestry or
then necessary to devise a presumption on land classification. Thus evolved the dictum the mineral which it contains than it is for agricultural purposes. (Sec.
in Ankron that the courts have a right to presume, in the absence of evidence to the 7, Act No. 1148.) It is not sufficient to show that there exists some
contrary, that in each case the lands are agricultural lands until the contrary is shown. [94] trees upon the land or that it bears some mineral. Land may be
classified as forestry or mineral today, and, by reason of the
But We cannot unduly expand the presumption in Ankron and De Aldecoa to an exhaustion of the timber or mineral, be classified as agricultural land
argument that all lands of the public domain had been automatically reclassified as tomorrow. And vice-versa, by reason of the rapid growth of timber or
disposable and alienable agricultural lands. By no stretch of imagination did the the discovery of valuable minerals, lands classified as agricultural
presumption convert all lands of the public domain into agricultural lands. today may be differently classified tomorrow. Each case must be
decided upon the proof in that particular case, having regard for
If We accept the position of private claimants, the Philippine Bill of 1902 and its present or future value for one or the other purposes. We
Act No. 926 would have automatically made all lands in the Philippines, except those believe, however, considering the fact that it is a matter of public
already classified as timber or mineral land, alienable and disposable lands. That would knowledge that a majority of the lands in the Philippine Islands are
take these lands out of State ownership and worse, would be utterly inconsistent with and agricultural lands that the courts have a right to presume, in the
totally repugnant to the long-entrenched Regalian doctrine. absence of evidence to the contrary, that in each case the lands are
agricultural lands until the contrary is shown. Whatever the land
The presumption in Ankron and De Aldecoa attaches only to land registration involved in a particular land registration case is forestry or
cases brought under the provisions of Act No. 926, or more specifically those cases mineral land must, therefore, be a matter of proof. Its superior
dealing with judicial and administrative confirmation of imperfect titles. The presumption value for one purpose or the other is a question of fact to be
applies to an applicant for judicial or administrative conformation of imperfect title under settled by the proof in each particular case. The fact that the land
Act No. 926. It certainly cannot apply to landowners, such as private claimants or their is a manglar [mangrove swamp] is not sufficient for the courts to
predecessors-in-interest, who failed to avail themselves of the benefits of Act No. 926. As decide whether it is agricultural, forestry, or mineral land. It may
to them, their land remained unclassified and, by virtue of the Regalian doctrine, perchance belong to one or the other of said classes of land. The
continued to be owned by the State. Government, in the first instance, under the provisions of Act No.
1148, may, by reservation, decide for itself what portions of public
In any case, the assumption in Ankron and De Aldecoa was not absolute. Land land shall be considered forestry land, unless private interests have
classification was, in the end, dependent on proof. If there was proof that the land was intervened before such reservation is made. In the latter case, whether
better suited for non-agricultural uses, the courts could adjudge it as a mineral or timber the land is agricultural, forestry, or mineral, is a question of proof.
land despite the presumption. In Ankron, this Court stated: Until private interests have intervened, the Government, by virtue of
the terms of said Act (No. 1148), may decide for itself what portions
In the case of Jocson vs. Director of Forestry (supra), the of the public domain shall be set aside and reserved as forestry or
Attorney-General admitted in effect that whether the particular land mineral land. (Ramos vs. Director of Lands, 39 Phil. 175; Jocson vs.
in question belongs to one class or another is a question of fact. The Director of Forestry, supra)[95] (Emphasis ours)
mere fact that a tract of land has trees upon it or has mineral within it
is not of itself sufficient to declare that one is forestry land and the Since 1919, courts were no longer free to determine the classification of lands
other, mineral land. There must be some proof of the extent and from the facts of each case, except those that have already became private lands.[96] Act
present or future value of the forestry and of the minerals. While, as No. 2874, promulgated in 1919 and reproduced in Section 6 of CA No. 141, gave the
Executive Department, through the President, the exclusive prerogative to classify or
reclassify public lands into alienable or disposable, mineral or forest. 96-a Since then, A similar argument was squarely rejected by the Court in Collado v. Court of
courts no longer had the authority, whether express or implied, to determine the Appeals.[107] Collado, citing the separate opinion of now Chief Justice Reynato S. Puno
classification of lands of the public domain.[97] in Cruz v. Secretary of Environment and Natural Resources,107-a ruled:

Here, private claimants, unlike the Heirs of Ciriaco Tirol who were issued their Act No. 926, the first Public Land Act,
title in 1933,[98] did not present a justiciable case for determination by the land was passed in pursuance of the provisions of the
registration court of the propertys land classification. Simply put, there was no Philippine Bill of 1902. The law governed the
opportunity for the courts then to resolve if the land the Boracay occupants are now disposition of lands of the public domain. It
claiming were agricultural lands. When Act No. 926 was supplanted by Act No. 2874 in prescribed rules and regulations for the
1919, without an application for judicial confirmation having been filed by private homesteading, selling and leasing of portions of
claimants or their predecessors-in-interest, the courts were no longer authorized to the public domain of the Philippine Islands, and
determine the propertys land classification. Hence, private claimants cannot bank on Act prescribed the terms and conditions to enable
No. 926. persons to perfect their titles to public lands in
the Islands. It also provided for the issuance of
We note that the RTC decision[99] in G.R. No. 167707 mentioned Krivenko v. patents to certain native settlers upon public lands,
Register of Deeds of Manila,[100] which was decided in 1947 when CA No. 141, vesting for the establishment of town sites and sale of lots
the Executive with the sole power to classify lands of the public domain was already in therein, for the completion of imperfect titles, and
effect. Krivenko cited the old cases Mapa v. Insular Government,[101] De Aldecoa v. The for the cancellation or confirmation of Spanish
Insular Government,[102] and Ankron v. Government of the Philippine Islands.[103] concessions and grants in the Islands. In short,
the Public Land Act operated on the assumption
Krivenko, however, is not controlling here because it involved a totally that title to public lands in the Philippine Islands
different issue. The pertinent issue in Krivenko was whether residential lots were included remained in the government; and that the
in the general classification of agricultural lands; and if so, whether an alien could acquire governments title to public land sprung from the
a residential lot. This Court ruled that as an alien, Krivenko was prohibited by the 1935 Treaty of Paris and other subsequent treaties
Constitution[104] from acquiring agricultural land, which included residential lots. Here, between Spain and the United States. The term
the issue is whether unclassified lands of the public domain are automatically deemed public land referred to all lands of the public
agricultural. domain whose title still remained in the
government and are thrown open to private
Notably, the definition of agricultural public lands mentioned appropriation and settlement, and excluded the
in Krivenko relied on the old cases decided prior to the enactment of Act No. 2874, patrimonial property of the government and the
including Ankron and De Aldecoa.[105] As We have already stated, those cases cannot friar lands.
apply here, since they were decided when the Executive did not have the authority to
classify lands as agricultural, timber, or mineral. Thus, it is plain error for petitioners to argue that under the
Philippine Bill of 1902 and Public Land Act No. 926, mere
Private claimants continued possession under Act No. 926 does not create a possession by private individuals of lands creates the legal
presumption that the land is alienable. Private claimants also contend that their presumption that the lands are alienable and
continued possession of portions of Boracay Island for the requisite period of ten (10) disposable.[108] (Emphasis Ours)
years under Act No. 926[106] ipso facto converted the island into private ownership. Hence,
they may apply for a title in their name.
Except for lands already covered by existing titles, Boracay was an not have to be descriptive of what the land actually looks
unclassified land of the public domain prior to Proclamation No. 1064. Such like. Unless and until the land classified as forest is released in an
unclassified lands are considered public forest under PD No. 705. The DENR[109] and official proclamation to that effect so that it may form part of the
the National Mapping and Resource Information Authority[110] certify disposable agricultural lands of the public domain, the rules on
that Boracay Island is an unclassified land of the public domain. confirmation of imperfect title do not apply.[115] (Emphasis supplied)

PD No. 705 issued by President Marcos categorized all unclassified lands of the There is a big difference between forest as defined in a dictionary and forest or timber
public domain as public forest. Section 3(a) of PD No. 705 defines a public forest as land as a classification of lands of the public domain as appearing in our statutes. One is
a mass of lands of the public domain which has not been the subject of the present system descriptive of what appears on the land while the other is a legal status, a classification
of classification for the determination of which lands are needed for forest purpose and for legal purposes.[116] At any rate, the Court is tasked to determine the legal status
which are not. Applying PD No. 705, all unclassified lands, including those of Boracay Island, and not look into its physical layout. Hence, even if its forest cover has
in Boracay Island, are ipso facto considered public forests. PD No. 705, however, been replaced by beach resorts, restaurants and other commercial establishments, it has
respects titles already existing prior to its effectivity. not been automatically converted from public forest to alienable agricultural land.

The Court notes that the classification of Boracay as a forest land under PD No. Private claimants cannot rely on Proclamation No. 1801 as basis for judicial
705 may seem to be out of touch with the present realities in the island. Boracay, no confirmation of imperfect title. The proclamation did not convert Boracay into an
doubt, has been partly stripped of its forest cover to pave the way for commercial agricultural land. However, private claimants argue that Proclamation No. 1801 issued
developments. As a premier tourist destination for local and foreign tourists, Boracay by then President Marcos in 1978 entitles them to judicial confirmation of imperfect
appears more of a commercial island resort, rather than a forest land. title. The Proclamation classified Boracay, among other islands, as a tourist zone. Private
claimants assert that, as a tourist spot, the island is susceptible of private ownership.
Nevertheless, that the occupants of Boracay have built multi-million peso beach
resorts on the island;[111] that the island has already been stripped of its forest cover; or Proclamation No. 1801 or PTA Circular No. 3-82 did not convert the whole of
that the implementation of Proclamation No. 1064 will destroy the islands tourism Boracay into an agricultural land. There is nothing in the law or the Circular which
industry, do not negate its character as public forest. made Boracay Island an agricultural land. The reference in Circular No. 3-82 to private
lands[117] and areas declared as alienable and disposable[118] does not by itself classify the
Forests, in the context of both the Public Land Act and the entire island as agricultural. Notably, Circular No. 3-82 makes reference not only to
Constitution[112] classifying lands of the public domain into agricultural, forest or timber, private lands and areas but also to public forested lands. Rule VIII, Section 3 provides:
mineral lands, and national parks, do not necessarily refer to large tracts of wooded land
or expanses covered by dense growths of trees and underbrushes.[113] The discussion No trees in forested private lands may be cut without prior
in Heirs of Amunategui v. Director of Forestry[114] is particularly instructive: authority from the PTA. All forested areas in public lands are
declared forest reserves. (Emphasis supplied)
A forested area classified as forest land of the public
domain does not lose such classification simply because loggers or Clearly, the reference in the Circular to both private and public lands merely
settlers may have stripped it of its forest cover. Parcels of land recognizes that the island can be classified by the Executive department pursuant to its
classified as forest land may actually be covered with grass or planted powers under CA No. 141. In fact, Section 5 of the Circular recognizes the then Bureau
to crops by kaingin cultivators or other farmers. Forest lands do not of Forest Developments authority to declare areas in the island as alienable and
have to be on mountains or in out of the way places. Swampy areas disposable when it provides:
covered by mangrove trees, nipa palms, and other trees growing in
brackish or sea water may also be classified as forest land. The Subsistence farming, in areas declared as alienable and
classification is descriptive of its legal nature or status and does disposable by the Bureau of Forest Development.
15-meter buffer zone on each side of the center line of roads and trails, which are
Therefore, Proclamation No. 1801 cannot be deemed the positive act needed to reserved for right of way and which shall form part of the area reserved for forest land
classify Boracay Island as alienable and disposable land. If President Marcos intended to protection purposes.
classify the island as alienable and disposable or forest, or both, he would have identified Contrary to private claimants argument, there was nothing invalid or irregular,
the specific limits of each, as President Arroyo did in Proclamation No. 1064. This was much less unconstitutional, about the classification of Boracay Island made by the
not done in Proclamation No. 1801. President through Proclamation No. 1064. It was within her authority to make such
classification, subject to existing vested rights.
The Whereas clauses of Proclamation No. 1801 also explain the rationale
behind the declaration of Boracay Island, together with other islands, caves and Proclamation No. 1064 does not violate the Comprehensive Agrarian Reform
peninsulas in the Philippines, as a tourist zone and marine reserve to be administered by Law. Private claimants further assert that Proclamation No. 1064 violates the provision of
the PTA to ensure the concentrated efforts of the public and private sectors in the the Comprehensive Agrarian Reform Law (CARL) or RA No. 6657 barring conversion of
development of the areas tourism potential with due regard for ecological balance in the public forests into agricultural lands. They claim that since Boracay is a public forest
marine environment. Simply put, the proclamation is aimed at administering the islands under PD No. 705, President Arroyo can no longer convert it into an agricultural land
for tourism and ecological purposes. It does not address the areas alienability.[119] without running afoul of Section 4(a) of RA No. 6657, thus:

More importantly, Proclamation No. 1801 covers not only Boracay Island, but SEC. 4. Scope. The Comprehensive Agrarian Reform Law
sixty-four (64) other islands, coves, and peninsulas in the Philippines, such as Fortune of 1988 shall cover, regardless of tenurial arrangement and
and Verde Islands in Batangas, Port Galera in Oriental Mindoro, Panglao and Balicasag commodity produced, all public and private agricultural lands as
Islands in Bohol, Coron Island, Puerto Princesa and surrounding areas in Palawan, provided in Proclamation No. 131 and Executive Order No. 229,
Camiguin Island in Cagayan de Oro, and Misamis Oriental, to name a few. If the including other lands of the public domain suitable for agriculture.
designation of Boracay Island as tourist zone makes it alienable and disposable by virtue
of Proclamation No. 1801, all the other areas mentioned would likewise be declared wide More specifically, the following lands are covered by the
open for private disposition. That could not have been, and is clearly beyond, the intent of Comprehensive Agrarian Reform Program:
the proclamation.
(a) All alienable and disposable lands of the
It was Proclamation No. 1064 of 2006 which positively declared part of public domain devoted to or suitable for
Boracay as alienable and opened the same to private ownership. Sections 6 and 7 of CA agriculture. No reclassification of forest or
No. 141[120] provide that it is only the President, upon the recommendation of the proper mineral lands to agricultural lands shall be
department head, who has the authority to classify the lands of the public domain into undertaken after the approval of this Act
alienable or disposable, timber and mineral lands.[121] until Congress, taking into account
ecological, developmental and equity
In issuing Proclamation No. 1064, President Gloria Macapagal-Arroyo merely considerations, shall have determined by
exercised the authority granted to her to classify lands of the public domain, presumably law, the specific limits of the public domain.
subject to existing vested rights. Classification of public lands is the exclusive prerogative
of the Executive Department, through the Office of the President. Courts have no That Boracay Island was classified as a public forest under PD No. 705 did not
authority to do so.[122] Absent such classification, the land remains unclassified until bar the Executive from later converting it into agricultural land. Boracay Island still
released and rendered open to disposition.[123] remained an unclassified land of the public domain despite PD No. 705.

Proclamation No. 1064 classifies Boracay into 400 hectares of reserved forest In Heirs of the Late Spouses Pedro S. Palanca and Soterranea Rafols v.
land and 628.96 hectares of agricultural land. The Proclamation likewise provides for a Republic,[124] the Court stated that unclassified lands are public forests.
While it is true that the land classification map does not As discussed, the Philippine Bill of 1902, Act No. 926, and Proclamation No.
categorically state that the islands are public forests, the fact that 1801 did not convert portions of Boracay Island into an agricultural land. The island
they were unclassified lands leads to the same result.In the remained an unclassified land of the public domain and, applying the Regalian doctrine,
absence of the classification as mineral or timber land, the land is considered State property.
remains unclassified land until released and rendered open to
disposition.[125] (Emphasis supplied) Private claimants bid for judicial confirmation of imperfect title, relying on the
Philippine Bill of 1902, Act No. 926, and Proclamation No. 1801, must fail because of
Moreover, the prohibition under the CARL applies only to a reclassification of the absence of the second element of alienable and disposable land. Their entitlement to a
land. If the land had never been previously classified, as in the case of Boracay, there can government grant under our present Public Land Act presupposes that the land possessed
be no prohibited reclassification under the agrarian law. We agree with the opinion of the and applied for is already alienable and disposable. This is clear from the wording of the
Department of Justice[126] on this point: law itself.[129] Where the land is not alienable and disposable, possession of the land, no
matter how long, cannot confer ownership or possessory rights. [130]
Indeed, the key word to the correct application of the
prohibition in Section 4(a) is the word reclassification. Where there Neither may private claimants apply for judicial confirmation of imperfect title
has been no previous classification of public forest [referring, we under Proclamation No. 1064, with respect to those lands which were classified as
repeat, to the mass of the public domain which has not been the agricultural lands. Private claimants failed to prove the first element of open, continuous,
subject of the present system of classification for purposes of exclusive, and notorious possession of their lands in Boracay since June 12, 1945.
determining which are needed for forest purposes and which are not]
into permanent forest or forest reserves or some other forest uses We cannot sustain the CA and RTC conclusion in the petition for declaratory
under the Revised Forestry Code, there can be no reclassification of relief that private claimants complied with the requisite period of possession.
forest lands to speak of within the meaning of Section 4(a).
The tax declarations in the name of private claimants are insufficient to prove
Thus, obviously, the prohibition in Section 4(a) of the the first element of possession. We note that the earliest of the tax declarations in the
CARL against the reclassification of forest lands to agricultural lands name of private claimants were issued in 1993. Being of recent dates, the tax declarations
without a prior law delimiting the limits of the public domain, does are not sufficient to convince this Court that the period of possession and occupation
not, and cannot, apply to those lands of the public domain, commenced on June 12, 1945.
denominated as public forest under the Revised Forestry Code, which
have not been previously determined, or classified, as needed for Private claimants insist that they have a vested right in Boracay, having been in
forest purposes in accordance with the provisions of the Revised possession of the island for a long time. They have invested millions of pesos in
Forestry Code.[127] developing the island into a tourist spot. They say their continued possession and
investments give them a vested right which cannot be unilaterally rescinded by
Private claimants are not entitled to apply for judicial confirmation of Proclamation No. 1064.
imperfect title under CA No. 141. Neither do they have vested rights over the occupied
lands under the said law. There are two requisites for judicial confirmation of imperfect The continued possession and considerable investment of private claimants do
or incomplete title under CA No. 141, namely: (1) open, continuous, exclusive, and not automatically give them a vested right in Boracay. Nor do these give them a right to
notorious possession and occupation of the subject land by himself or through his apply for a title to the land they are presently occupying. This Court is constitutionally
predecessors-in-interest under a bona fide claim of ownership since time immemorial or bound to decide cases based on the evidence presented and the laws applicable. As the
from June 12, 1945; and (2) the classification of the land as alienable and disposable land law and jurisprudence stand, private claimants are ineligible to apply for a judicial
of the public domain.[128] confirmation of title over their occupied portions in Boracay even with their continued
possession and considerable investment in the island.
One Last Note The view this Court takes of the cases at bar is but in
adherence to public policy that should be followed with respect to
The Court is aware that millions of pesos have been invested for the forest lands. Many have written much, and many more have spoken,
development of Boracay Island, making it a by-word in the local and international and quite often, about the pressing need for forest preservation,
tourism industry. The Court also notes that for a number of years, thousands of people conservation, protection, development and reforestation. Not without
have called the island their home. While the Court commiserates with private claimants justification. For, forests constitute a vital segment of any country's
plight, We are bound to apply the law strictly and judiciously. This is the law and it natural resources. It is of common knowledge by now that absence of
should prevail. Ito ang batas at ito ang dapat umiral. the necessary green cover on our lands produces a number of adverse
or ill effects of serious proportions. Without the trees, watersheds dry
All is not lost, however, for private claimants. While they may not be eligible up; rivers and lakes which they supply are emptied of their contents.
to apply for judicial confirmation of imperfect title under Section 48(b) of CA No. 141, as The fish disappear. Denuded areas become dust bowls. As waterfalls
amended, this does not denote their automatic ouster from the residential, commercial, cease to function, so will hydroelectric plants. With the rains, the
and other areas they possess now classified as agricultural. Neither will this mean the loss fertile topsoil is washed away; geological erosion results. With
of their substantial investments on their occupied alienable lands. Lack of title does not erosion come the dreaded floods that wreak havoc and destruction to
necessarily mean lack of right to possess. property crops, livestock, houses, and highways not to mention
precious human lives. Indeed, the foregoing observations should be
For one thing, those with lawful possession may claim good faith as builders of written down in a lumbermans decalogue.[135]
improvements. They can take steps to preserve or protect their possession. For another,
they may look into other modes of applying for original registration of title, such as by WHEREFORE, judgment is rendered as follows:
homestead[131] or sales patent,[132] subject to the conditions imposed by law.
1. The petition for certiorari in G.R. No. 167707 is GRANTED and the Court
More realistically, Congress may enact a law to entitle private claimants to of Appeals Decision in CA-G.R. CV No. 71118 REVERSED AND SET ASIDE.
acquire title to their occupied lots or to exempt them from certain requirements under the
present land laws. There is one such bill[133] now pending in the House of 2. The petition for certiorari in G.R. No. 173775 is DISMISSED for lack of
Representatives. Whether that bill or a similar bill will become a law is for Congress to merit.
decide.
SO ORDERED.
In issuing Proclamation No. 1064, the government has taken the step necessary
to open up the island to private ownership. This gesture may not be sufficient to appease
some sectors which view the classification of the island partially into a forest reserve as
absurd. That the island is no longer overrun by trees, however, does not becloud the
vision to protect its remaining forest cover and to strike a healthy balance between
progress and ecology. Ecological conservation is as important as economic progress.

To be sure, forest lands are fundamental to our nations survival. Their


promotion and protection are not just fancy rhetoric for politicians and activists. These
are needs that become more urgent as destruction of our environment gets prevalent and
difficult to control. As aptly observed by Justice Conrado Sanchez in 1968 in Director of
Forestry v. Munoz:[134]
FIRST DIVISION predecessors-in-interest of private respondents Heirs of Antonio Carag and
Victoria Turingan (private respondents), covering a parcel of land identified as Lot No.
REPUBLIC OF THE PHILIPPINES G.R. No. 155450 2472, Cad. 151, containing an area of 7,047,673 square meters (subject property),
represented by the Regional situated in Tuguegarao, Cagayan. On 19 July 1938, pursuant to said Decree, the Register
Executive Director, Department of of Deeds of Cagayan issued Original Certificate of Title No. 11585[5] (OCT No. 11585)
Environment and Natural Resources, Present: in the name of spouses Carag.
Regional Office No. 2,
Petitioner, PUNO, C.J., Chairperson, On 2 July 1952, OCT No. 11585 was cancelled to discharge the encumbrance expressly
CARPIO, stated in Decree No. 381928. Two transfer certificates of title were issued: Transfer
AUSTRIA-MARTINEZ,* Certificate of Title No. T-1277,[6] issued in the name of the Province of Cagayan,
- versus - CORONA, and covering Lot 2472-B consisting of 100,000 square meters and Transfer Certificate of
LEONARDO-DE CASTRO, JJ. Title No. T-1278,[7] issued in the name of the private respondents, covering Lot 2472-A
consisting of 6,997,921 square meters.

COURT OF APPEALS, On 19 May 1994, Bienvenida Taguiam Vda. De Dayag and others filed with the Regional
HEIRS OF ANTONIO CARAG AND Office No. 2 of the Department of Environment and Natural Resources
VICTORIA TURINGAN, (DENR), Tuguegarao, Cagayan, a letter-petition requesting the DENR to initiate the
THE REGISTER OF DEEDS OF filing of an action for the annulment of Decree No. 381928 on the ground that the trial
CAGAYAN, and the COURT OF Promulgated: court did not have jurisdiction to adjudicate a portion of the subject property which was
FIRST INSTANCE OF CAGAYAN, allegedly still classified as timber land at the time of the issuance of Decree No. 381928.
Respondents. August 6, 2008
The Regional Executive Director of the DENR created an investigating team to conduct
x- - - - - - - - - - - - - - - - - - - - - - - - - - - - - - - - - - - - - - - - - - - - - - - - - - - x ground verification and ocular inspection of the subject property.

D E C I S IO N The investigating team reported that:

CARPIO, J.: A) The portion of Lot 2472 Cad-151 as shown in the Plan prepared
The Case for spouses Carag, and covered under LC Project 3-L
of Tuguegarao, Cagayan, was found to be still within the timberland
This is a petition for review[1] of the 21 May 2001[2] and 25 September area at the time of the issuance of the Decree and O.C.T. of the
2002[3] Resolutions of the Court of Appeals in CA-G.R. SP No. 47965. The spouses Antonio Carag and Victoria Turingan, and the same was only
21 May 2001 Resolution dismissed petitioner Republic of the Philippines (petitioner) released as alienable and disposable on February 22, 1982, as
amended complaint for reversion, annulment of decree, cancellation and declaration of certified by USEC Jose G. Solis of the NAMRIA on 27 May 1994.
nullity of titles. The 25 September 2002 Resolution denied petitioners motion for
reconsideration. B) Petitioner Bienvenida Taguiam Vda. De Dayag and others have
possessed and occupied by themselves and thru their predecessors-in-
The Facts interest the portion of Lot 2472 Cad-151, covered by LC Project 3-L
of LC Map 2999, since time immemorial.[8]
On 2 June 1930, the then Court of First Instance of Cagayan (trial court) issued Decree
No. 381928[4] in favor of spouses Antonio Carag and Victoria Turingan (spouses Carag),
Thus, the investigating team claimed that a portion of Lot 2472 Cad-151 was only The rule is clear that such judgments, final orders and resolutions in
released as alienable and disposable on 22 February 1982. civil actions which this court may annul are those which the ordinary
remedies of new trial, appeal, petition for relief or other appropriate
In a Memorandum dated 9 September 1996, the Legal Division of the Land Management remedies are no longer available. The Amended Complaint contains
Bureau recommended to the Director of Lands that an action for the cancellation of OCT no such allegations which are jurisdictional neither can such
No. 11585, as well as its derivative titles, be filed with the proper court. The Director of circumstances be divined from its allegations.Furthermore, such
Lands approved the recommendation. actions for Annulment may be based only on two (2) grounds:
extrinsic fraud and lack of jurisdiction. Neither ground is alleged in
the Amended Complaint which is for Reversion/Annulment of Decree,
On 10 June 1998, or 68 years after the issuance of Decree No. 381928, petitioner filed Cancellation and Declaration of Nullity of Titles. It merely alleges
with the Court of Appeals a complaint for annulment of judgment, cancellation and that around 2,640,000 square meters of timberland area
declaration of nullity of titles[9] on the ground that in 1930 the trial court had no within Lot 2472 Cad. 151, had been erroneously included in the title
jurisdiction to adjudicate a portion of the subject property, which portion consists of of the Spouses Antonio Carag and Victoria Turingan under Decree
2,640,000 square meters (disputed portion). The disputed portion was allegedly still No. 381928 and O.C.T. No. 11585 issued on June 2, 1930 and July 19,
classified as timber land at the time of issuance of Decree No. 381928 and, therefore, was 1938, respectively; that hence, such adjudication and/or Decree and
not alienable and disposable until 22 February 1982 when the disputed portion was Title covering a timberland area is null and void ab initio under the
classified as alienable and disposable. provisions of the 1935, 1973 and 1987 Constitutions.

On 19 October 1998, private respondents filed a motion to dismiss.[10] Private Finally, it is clear that the issues raised in the Amended Complaint as
respondents alleged that petitioner failed to comply with Rule 47 of the Rules of Court well as those in the Motion to dismiss are factual in nature and should
because the real ground for the complaint was mistake, not lack of jurisdiction, and that be threshed out in the proper trial court in accordance with Section
petitioner, as a party in the original proceedings, could have availed of the ordinary 101 of the Public Land Act.[14] (Citations omitted)
remedies of new trial, appeal, petition for relief or other appropriate remedies but failed to
do so. Private respondents added that petitioner did not attach to the complaint a certified Petitioner filed a motion for reconsideration. In its 25 September 2002 Resolution, the
true copy of the decision sought to be annulled. Private respondents also maintained that Court of Appeals denied the motion for reconsideration.
the complaint was barred by the doctrines of res judicata and law of the case and by
Section 38 of Act No. 496.[11] Private respondents also stated that not all the heirs of Hence, this petition.
spouses Carag were brought before the Court of Appeals for an effective resolution of the
case. Finally, private respondents claimed that the real party in interest was not petitioner The Issues
but a certain Alfonso Bassig, who had an ax to grind against private respondents.[12]
Petitioner raises the following issues:
On 3 March 1999, petitioner filed an amended complaint for reversion, annulment of
decree, cancellation and declaration of nullity of titles.[13] 1.Whether the allegations of the complaint clearly stated that the ordinary
remedies of new trial, appeal, petition for relief and other appropriate
The Ruling of the Court of Appeals remedies are no longer available;
2. Whether the amended complaint clearly alleged the ground of lack of
On 21 May 2001, the Court of Appeals dismissed the complaint because of lack of jurisdiction;
jurisdiction over the subject matter of the case. The Court of Appeals declared: 3. Whether the Court of Appeals may try the factual issues raised in the
amended complaint and in the motion to dismiss;
4. Whether the then Court of First Instance of Cagayan had jurisdiction to erroneous for the reason that said Court and/or the Register of
adjudicate a tract of timberland in favor of respondent spouses Deeds of Cagayan did not have any authority or jurisdiction to
Antonio Carag and Victoria Turingan; decree or adjudicate the said timberland area of Lot 2472 Cad-
5. Whether the fact that the Director of Lands was a party to the original 151, consequently, the same are null and void ab initio, and of no
proceedings changed the nature of the land and granted jurisdiction to the force and effect whatsoever.[16] (Emphasis supplied; citations omitted)
then Court of First Instance over the land; Petitioner clearly alleged in the complaint and amended complaint that it was seeking to
6. Whether the doctrine of res judicata applies in this case; and annul Decree No. 381928 on the ground of the trial courts lack of jurisdiction over the
7. Whether Section 38 of Act No. 496 is applicable in this case. subject land, specifically over the disputed portion, which petitioner maintained was
classified as timber land and was not alienable and disposable.
The Ruling of the Court
Second, the Court of Appeals also dismissed the complaint on the ground of petitioners
While the Court of Appeals erred in dismissing the complaint on procedural failure to allege that the ordinary remedies of new trial, appeal, petition for relief or other
grounds, we will still deny the petition because the complaint for annulment of decree has appropriate remedies are no longer available.
no merit.
In Ancheta v. Ancheta,[17] we ruled:
Petitioner Complied with Rule 47 of the Rules of Court
First, the Court of Appeals ruled that petitioner failed to allege either of the grounds of In a case where a petition for annulment of judgment or final order of
extrinsic fraud or lack of jurisdiction in the complaint for annulment of decree.[15] the RTC filed under Rule 47 of the Rules of Court is grounded on
lack of jurisdiction over the person of the defendant/respondent or
We find otherwise. In its complaint and amended complaint, petitioner stated: over the nature or subject of the action, the petitioner need not allege
in the petition that the ordinary remedy of new trial or
11. In view of the fact that in 1930 or in 1938, only the Executive reconsideration of the final order or judgment or appeal therefrom are
Branch of the Government had the authority and power to declassify no longer available through no fault of her own. This is so because a
or reclassify land of the public domain, the Court did not, therefore, judgment rendered or final order issued by the RTC without
have the power and authority to adjudicate in favor of the jurisdiction is null and void and may be assailed any time either
spouses Antonio Carag and Victoria Turingan the said tract of collaterally or in a direct action or by resisting such judgment or final
timberland, portion of the Lot 2472 Cad-151, at the time of the order in any action or proceeding whenever it is invoked, unless
issuance of the Decree and the Original Certificate of Title of the barred by laches.[18]
said spouses; and such adjudication and/or Decree and Title issued
covering the timberland area is null and void ab initio considering the Since petitioners complaint is grounded on lack of jurisdiction over the subject of the
provisions of the 1935, 1973 and 1987 Philippine constitution. action, petitioner need not allege that the ordinary remedies of new trial, appeal, petition
for relief or other appropriate remedies are no longer available through no fault of
xxxx petitioner.

15. The issuance of Decree No. 381928 and O.C.T. No. 11585 in the Third, the Court of Appeals ruled that the issues raised in petitioners complaint were
name of spouses Antonio Carag and Victoria Turingan, and all the factual in nature and should be threshed out in the proper trial court in accordance with
derivative titles thereto in the name of the Heirs and said spouses, Section 101 of the Public Land Act.[19]
specifically with respect to the inclusion thereto of timberland area,
by the then Court of First Instance (now the Regional Trial Court), Section 6, Rule 47 of the Rules of Court provides:
and the Register of Deeds of Cagayan is patently illegal and
SEC. 6. Procedure. - The procedure in ordinary civil cases shall be In this case, petitioner has not alleged that the disputed portion had been declared as
observed. Should a trial be necessary, the reception of evidence may mineral or forest zone, or reserved for some public purpose in accordance with law,
be referred to a member of the court or a judge of a Regional during the Spanish regime or thereafter. The land classification maps[24] petitioner
Trial Court. attached to the complaint also do not show that in 1930 the disputed portion was part of
the forest zone or reserved for some public purpose. The certification of the National
Therefore, the Court of Appeals may try the factual issues raised in the complaint for the Mapping and Resources Information Authority, dated 27 May 1994, contained no
complete and proper determination of the case. statement that the disputed portion was declared and classified as timber land. [25]
The law prevailing when Decree No. 381928 was issued in 1930 was Act No.
However, instead of remanding the complaint to the Court of Appeals for further 2874,[26] which provides:
proceedings, we shall decide the case on the merits.
SECTION 6. The Governor-General, upon the recommendation of
Complaint for Annulment of Decree Has No Merit the Secretary of Agriculture and Natural Resources, shall from time
Petitioner contends that the trial court had no jurisdiction to adjudicate to to time classify the lands of the public domain into -
spouses Carag the disputed portion of the subject property. Petitioner claims that the (a) Alienable or disposable
disputed portion was still classified as timber land, and thus not alienable and disposable, (b) Timber and
when Decree No. 381928 was issued in 1930. In effect, petitioner admits that the adjacent (c) Mineral lands
4,407,673 square meters of the subject property, outside of the disputed portion, were and may at any time and in a like manner transfer such lands from
alienable and disposable in 1930. Petitioner argues that in 1930 or in 1938, only the one class to another, for the purposes of their government and
Executive Branch of the Government, not the trial courts, had the power to declassify or disposition.
reclassify lands of the public domain.
Petitioner has not alleged that the Governor-General had declared the disputed portion of
Lack of jurisdiction, as a ground for annulment of judgment, refers to either lack of the subject property timber or mineral land pursuant to Section 6 of Act No. 2874.
jurisdiction over the person of the defending party or over the subject matter of the
claim.[20]Jurisdiction over the subject matter is conferred by law and is determined by the It is true that Section 8 of Act No. 2874 opens to disposition only those lands which have
statute in force at the time of the filing of the action.[21] been declared alienable or disposable. Section 8 provides:

Under the Spanish regime, all Crown lands were per se alienable. In Aldecoa v. Insular SECTION 8. Only those lands shall be declared open to disposition
Government,[22] we ruled: or concession which have been officially delimited and classified and,
when practicable, surveyed, and which have not been reserved for
From the language of the foregoing provisions of law, it is deduced public or quasi-public uses, not appropriated by the Government, nor
that, with the exception of those comprised within the mineral and in any manner become private property, nor those on which a
timber zone, all lands owned by the State or by the sovereign private right authorized and recognized by this Act or any other
nation are public in character, and per se alienable and, provided valid law may be claimed, or which, having been reserved or
they are not destined to the use of the public in general or reserved by appropriated, have ceased to be so. However, the Governor-General
the Government in accordance with law, they may be acquired by may, for reasons of public interest, declare lands of the public domain
any private or juridical person x x x[23] (Emphasis supplied) open to disposition before the same have had their boundaries
established or been surveyed, or may, for the same reasons, suspend
Thus, unless specifically declared as mineral or forest zone, or reserved by the State for their concession or disposition by proclamation duly published or by
some public purpose in accordance with law, all Crown lands were deemed alienable. Act of the Legislature. (Emphasis supplied)
However, Section 8 provides that lands which are already private lands, as well as lands years. It was the land registration court which had the
on which a private claim may be made under any law, are not covered by the jurisdiction to determine whether the land applied for was
classification requirement in Section 8 for purposes of disposition. This exclusion in agricultural, forest or timber taking into account the proof or
Section 8 recognizes that during the Spanish regime, Crown lands were per se evidence in each particular case. (Emphasis supplied)
alienableunless falling under timber or mineral zones, or otherwise reserved for some
public purpose in accordance with law. As with this case, when the trial court issued the decision for the issuance of Decree No.
381928 in 1930, the trial court had jurisdiction to determine whether the subject property,
Clearly, with respect to lands excluded from the classification requirement in Section including the disputed portion, applied for was agricultural, timber or mineral land. The
8, trial courts had jurisdiction to adjudicate these lands to private parties. Petitioner has trial court determined that the land was agricultural and that spouses Carag proved that
not alleged that the disputed portion had not become private property prior to the they were entitled to the decree and a certificate of title. The government, which was a
enactment of Act No. 2874. Neither has petitioner alleged that the disputed portion was party in the original proceedings in the trial court as required by law, did not appeal the
not land on which a private right may be claimed under any existing law at that time. decision of the trial court declaring the subject land as agricultural. Since the trial court
had jurisdiction over the subject matter of the action, its decision rendered in 1930, or 78
In Republic of the Philippines v. Court of Appeals,[27] the Republic sought to annul the years ago, is now final and beyond review.
judgment of the Court of First Instance (CFI) of Rizal, sitting as a land
registration court,because when the application for land registration was filed in 1927 the The finality of the trial courts decision is further recognized in Section 1, Article XII of
land was alleged to be unclassified forest land. The Republic also alleged that the CFI the 1935 Constitution which provides:
of Rizal had no jurisdiction to determine whether the land applied for was forest or
agricultural land since the authority to classify lands was then vested in the Director of SECTION 1. All agricultural, timber, and mineral lands of the public
Lands as provided in Act Nos. 926[28] and 2874. The Court ruled: domain, waters, minerals, coal, petroleum, and other mineral oils, all
forces of potential energy, and other natural resources of the
We are inclined to agree with the respondent that it is legally doubtful Philippines belong to the State, and their disposition, exploitation,
if the authority of the Governor General to declare lands as alienable development, or utilization shall be limited to citizens of the
and disposable would apply to lands that have become private Philippines, or to corporations or associations at least sixty per
property or lands that have been impressed with a private right centum of the capital of which is owned by such citizens, subject to
authorized and recognized by Act 2874 or any valid law. By express any existing right, grant, lease, or concession at the time of the
declaration of Section 45 (b) of Act 2874 which is quoted above, inauguration of the Government established under
those who have been in open, continuous, exclusive and notorious this Constitution. (Emphasis supplied)
possession and occupation of agricultural lands of the public domain
under a bona fide claim of acquisition of ownership since July 26, Thus, even as the 1935 Constitution declared that all agricultural, timber and mineral
1894 may file an application with the Court of First Instance of the lands of the public domain belong to the State, it recognized that these lands were subject
province where the land is located for confirmation of their claims to any existing right, grant, lease or concession at the time of the inauguration of the
and these applicants shall be conclusively presumed to have Government established under this Constitution.[29] When the Commonwealth
performed all the conditions essential to a government grant and shall Government was established under the 1935 Constitution, spouses Carag had already an
be entitled to a certificate of title. When the land registration court existing right to the subject land, including the disputed portion, pursuant to Decree No.
issued a decision for the issuance of a decree which was the basis 381928 issued in 1930 by the trial court.
of an original certificate of title to the land, the court had already WHEREFORE, we DENY the petition. We DISMISS petitioner Republic of
made a determination that the land was agricultural and that the the Philippines complaint for reversion, annulment of decree, cancellation and declaration
applicant had proven that he was in open and exclusive of nullity of titles for lack of merit.
possession of the subject land for the prescribed number of SO ORDERED.ANTONIO T. CARPIO. Associate Justice
Republic of the Philippines Fifth. Several months later (the 13th day of December, 1912) the plaintiffs discovered
SUPREME COURT that the wall which had been included in the certificate granted to them had also been
Manila included in the certificate granted to the defendant .They immediately presented a petition
in the Court of Land Registration for an adjustment and correction of the error committed
EN BANC by including said wall in the registered title of each of said parties. The lower court
however, without notice to the defendant, denied said petition upon the theory that,
during the pendency of the petition for the registration of the defendant's land, they failed
G.R. No. L-8936 October 2, 1915 to make any objection to the registration of said lot, including the wall, in the name of the
defendant.
CONSUELO LEGARDA, with her husband MAURO PRIETO, plaintiffs-
appellants, Sixth. That the land occupied by t he wall is registered in the name of each of the owners
vs. of the adjoining lots. The wall is not a joint wall.
N.M. SALEEBY, defendant-appellee.
Under these facts, who is the owner of the wall and the land occupied by it?
Singson, Ledesma and Lim for appellants.
D.R. Williams for appellee.
The decision of the lower court is based upon the theory that the action for the
registration of the lot of the defendant was a judicial proceeding and that the judgment or
JOHNSON, J.: decree was binding upon all parties who did not appear and oppose it. In other words, by
reason of the fact that the plaintiffs had not opposed the registration of that part of the lot
From the record the following facts appear: on which the wall was situate they had lost it, even though it had been theretofore
registered in their name. Granting that theory to be correct one, and granting even that the
First. That the plaintiffs and the defendant occupy, as owners, adjoining lots in the district wall and the land occupied by it, in fact, belonged to the defendant and his predecessors,
of Ermita in the city of Manila. then the same theory should be applied to the defendant himself. Applying that theory to
him, he had already lost whatever right he had therein, by permitting the plaintiffs to have
the same registered in their name, more than six years before. Having thus lost hid right,
Second. That there exists and has existed a number of years a stone wall between the said may he be permitted to regain it by simply including it in a petition for registration? The
lots. Said wall is located on the lot of the plaintiffs. plaintiffs having secured the registration of their lot, including the wall, were they obliged
to constantly be on the alert and to watch all the proceedings in the land court to see that
Third. That the plaintiffs, on the 2d day of March, 1906, presented a petition in the Court some one else was not having all, or a portion of the same, registered? If that question is
of Land Registration for the registration of their lot. After a consideration of said petition to be answered in the affirmative, then the whole scheme and purpose of the torrens
the court, on the 25th day of October, 1906, decreed that the title of the plaintiffs should system of land registration must fail. The real purpose of that system is to quiet title to
be registered and issued to them the original certificate provided for under the torrens land; to put a stop forever to any question of the legality of the title, except claims which
system. Said registration and certificate included the wall. were noted at the time of registration, in the certificate, or which may arise subsequent
thereto. That being the purpose of the law, it would seem that once a title is registered the
owner may rest secure, without the necessity of waiting in the portals of the court, or
Fourth. Later the predecessor of the defendant presented a petition in the Court of Land
sitting in the "mirador de su casa," to avoid the possibility of losing his land. Of course, it
Registration for the registration of the lot now occupied by him. On the 25th day of
can not be denied that the proceeding for the registration of land under the torrens system
March, 1912, the court decreed the registration of said title and issued the original
is judicial (Escueta vs. .Director of Lands, 16 Phil. Rep., 482). It is clothed with all the
certificate provided for under the torrens system. The description of the lot given in the
forms of an action and the result is final and binding upon all the world. It is an action in
petition of the defendant also included said wall.
rem. (Escueta vs. Director of Lands (supra); Grey Alba vs. De la Cruz, 17 Phil. rep., 49 The question, who is the owner of land registered in the name of two different persons,
Roxas vs. Enriquez, 29 Phil. Rep., 31; Tyler vs. Judges, 175 Mass., 51 American Land has been presented to the courts in other jurisdictions. In some jurisdictions, where the
Co. vs. Zeiss, 219 U.S., 47.) "torrens" system has been adopted, the difficulty has been settled by express statutory
provision. In others it has been settled by the courts. Hogg, in his excellent discussion of
While the proceeding is judicial, it involves more in its consequences than does an the "Australian Torrens System," at page 823, says: "The general rule is that in the case
ordinary action. All the world are parties, including the government. After the registration of two certificates of title, purporting to include the same land, the earlier in date prevails,
is complete and final and there exists no fraud, there are no innocent third parties who whether the land comprised in the latter certificate be wholly, or only in part, comprised
may claim an interest. The rights of all the world are foreclosed by the decree of in the earlier certificate. (Oelkers vs. Merry, 2 Q.S.C.R., 193; Miller vs. Davy, 7 N.Z.R.,
registration. The government itself assumes the burden of giving notice to all parties. To 155; Lloyd vs. Myfield, 7 A.L.T. (V.) 48; Stevens vs. Williams, 12 V.L. R., 152; Register
permit persons who are parties in the registration proceeding (and they are all the world) of Titles, vs. Esperance Land Co., 1 W.A.R., 118.)" Hogg adds however that, "if it can be
to again litigate the same questions, and to again cast doubt upon the validity of the very clearly ascertained by the ordinary rules of construction relating to written
registered title, would destroy the very purpose and intent of the law. The registration, documents, that the inclusion of the land in the certificate of title of prior date is a mistake,
under the torrens system, does not give the owner any better title than he had. If he does the mistake may be rectified by holding the latter of the two certificates of title to be
not already have a perfect title, he can not have it registered. Fee simple titles only may conclusive." (See Hogg on the "Australian torrens System," supra, and cases cited. See
be registered. The certificate of registration accumulates in open document a precise and also the excellent work of Niblack in his "Analysis of the Torrens System," page 99.)
correct statement of the exact status of the fee held by its owner. The certificate, in the Niblack, in discussing the general question, said: "Where two certificates purport to
absence of fraud, is the evidence of title and shows exactly the real interest of its owner. include the same land the earlier in date prevails. ... In successive registrations, where
The title once registered, with very few exceptions, should not thereafter be impugned, more than one certificate is issued in respect of a particular estate or interest in land, the
altered, changed, modified, enlarged, or diminished, except in some direct proceeding person claiming under the prior certificates is entitled to the estate or interest; and that
permitted by law. Otherwise all security in registered titles would be lost. A registered person is deemed to hold under the prior certificate who is the holder of, or whose claim
title can not be altered, modified, enlarged, or diminished in a collateral proceeding and is derived directly or indirectly from the person who was the holder of the earliest
not even by a direct proceeding, after the lapse of the period prescribed by law. certificate issued in respect thereof. While the acts in this country do not expressly cover
the case of the issue of two certificates for the same land, they provide that a registered
owner shall hold the title, and the effect of this undoubtedly is that where two certificates
For the difficulty involved in the present case the Act (No. 496) providing for the purport to include the same registered land, the holder of the earlier one continues to hold
registration of titles under the torrens system affords us no remedy. There is no provision the title" (p. 237).
in said Act giving the parties relief under conditions like the present. There is nothing in
the Act which indicates who should be the owner of land which has been registered in the
name of two different persons. Section 38 of Act No. 496, provides that; "It (the decree of registration) shall be
conclusive upon and against all persons, including the Insular Government and all the
branches thereof, whether mentioned by name in the application, notice, or citation, or
The rule, we think, is well settled that the decree ordering the registration of a particular included in the general description "To all whom it may concern." Such decree shall not
parcel of land is a bar to future litigation over the same between the same parties .In view be opened by reason of the absence, infancy, or other disability of any person affected
of the fact that all the world are parties, it must follow that future litigation over the title thereby, nor by any proceeding in any court for reversing judgments or decrees; subject,
is forever barred; there can be no persons who are not parties to the action. This, we think, however, to the right of any person deprived of land or of any estate or interest therein by
is the rule, except as to rights which are noted in the certificate or which arise decree of registration obtained by fraud to file in the Court of Land Registration a petition
subsequently, and with certain other exceptions which need not be dismissed at present. for review within one year after entry of the decree (of registration), provided no innocent
A title once registered can not be defeated, even by an adverse, open, and notorious purchaser for value has acquired an interest.
possession. Registered title under the torrens system can not be defeated by prescription
(section 46, Act No. 496). The title, once registered, is notice to the world. All persons
must take notice. No one can plead ignorance of the registration. It will be noted, from said section, that the "decree of registration" shall not be opened,
for any reason, in any court, except for fraud, and not even for fraud, after the lapse
of one year. If then the decree of registration can not be opened for any reason, except for elapsed after the plaintiff had secured their title, they had mortgaged or sold their right,
fraud, in a direct proceeding for that purpose, may such decree be opened or set aside in a what would be the position or right of the mortgagee or vendee? That mistakes are bound
collateral proceeding by including a portion of the land in a subsequent certificate or to occur cannot be denied, and sometimes the damage done thereby is irreparable. It is the
decree of registration? We do not believe the law contemplated that a person could be duty of the courts to adjust the rights of the parties under such circumstances so as to
deprived of his registered title in that way. minimize such damages, taking into consideration al of the conditions and the diligence
of the respective parties to avoid them. In the present case, the appellee was the first
We have in this jurisdiction a general statutory provision which governs the right of the negligent (granting that he was the real owner, and if he was not the real owner he can not
ownership of land when the same is registered in the ordinary registry in the name of two complain) in not opposing the registration in the name of the appellants. He was a party-
persons. Article 1473 of the Civil Code provides, among other things, that when one defendant in an action for the registration of the lot in question, in the name of the
piece of real property had been sold to two different persons it shall belong to the person appellants, in 1906. "Through his failure to appear and to oppose such registration, and
acquiring it, who first inscribes it in the registry. This rule, of course, presupposes that the subsequent entry of a default judgment against him, he became irrevocably bound by
each of the vendees or purchasers has acquired title to the land. The real ownership in the decree adjudicating such land to the appellants. He had his day in court and should
such a case depends upon priority of registration. While we do not now decide that the not be permitted to set up his own omissions as the ground for impugning the validity of a
general provisions of the Civil Code are applicable to the Land Registration Act, even judgment duly entered by a court of competent jurisdiction." Granting that he was the
though we see no objection thereto, yet we think, in the absence of other express owner of the land upon which the wall is located, his failure to oppose the registration of
provisions, they should have a persuasive influence in adopting a rule for governing the the same in the name of the appellants, in the absence of fraud, forever closes his mouth
effect of a double registration under said Act. Adopting the rule which we believe to be against impugning the validity of that judgment. There is no more reason why the
more in consonance with the purposes and the real intent of the torrens system, we are of doctrine invoked by the appellee should be applied to the appellants than to him.
the opinion and so decree that in case land has been registered under the Land
Registration Act in the name of two different persons, the earlier in date shall prevail. We have decided, in case of double registration under the Land Registration Act, that the
owner of the earliest certificate is the owner of the land. That is the rule between original
In reaching the above conclusion, we have not overlooked the forceful argument of the parties. May this rule be applied to successive vendees of the owners of such certificates?
appellee. He says, among other things; "When Prieto et al. were served with notice of the Suppose that one or the other of the parties, before the error is discovered, transfers his
application of Teus (the predecessor of the defendant) they became defendants in a original certificate to an "innocent purchaser." The general rule is that the vendee of land
proceeding wherein he, Teus, was seeking to foreclose their right, and that of orders, to has no greater right, title, or interest than his vendor; that he acquires the right which his
the parcel of land described in his application. Through their failure to appear and contest vendor had, only. Under that rule the vendee of the earlier certificate would be the owner
his right thereto, and the subsequent entry of a default judgment against them, they as against the vendee of the owner of the later certificate.
became irrevocably bound by the decree adjudicating such land to Teus. They had their
day in court and can not set up their own omission as ground for impugning the validity We find statutory provisions which, upon first reading, seem to cast some doubt upon the
of a judgment duly entered by a court of competent jurisdiction. To decide otherwise rule that the vendee acquires the interest of the vendor only. Sections 38, 55, and 112 of
would be to hold that lands with torrens titles are above the law and beyond the Act No. 496 indicate that the vendee may acquire rights and be protected against defenses
jurisdiction of the courts". which the vendor would not. Said sections speak of available rights in favor of third
parties which are cut off by virtue of the sale of the land to an "innocent purchaser." That
As was said above, the primary and fundamental purpose of the torrens system is to quiet is to say, persons who had had a right or interest in land wrongfully included in an
title. If the holder of a certificate cannot rest secure in this registered title then the purpose original certificate would be unable to enforce such rights against an "innocent
of the law is defeated. If those dealing with registered land cannot rely upon the purchaser," by virtue of the provisions of said sections. In the present case Teus had his
certificate, then nothing has been gained by the registration and the expense incurred land, including the wall, registered in his name. He subsequently sold the same to the
thereby has been in vain. If the holder may lose a strip of his registered land by the appellee. Is the appellee an "innocent purchaser," as that phrase is used in said sections?
method adopted in the present case, he may lose it all. Suppose within the six years which May those who have been deprived of their land by reason of a mistake in the original
certificate in favor of Teus be deprived of their right to the same, by virtue of the sale by
him to the appellee? Suppose the appellants had sold their lot, including the wall, to an While there is no statutory provision in force here requiring that original deeds of
"innocent purchaser," would such purchaser be included in the phrase "innocent conveyance of real property be recorded, yet there is a rule requiring mortgages to be
purchaser," as the same is used in said sections? Under these examples there would be recorded. (Arts. 1875 and 606 of the Civil Code.) The record of a mortgage is
two innocent purchasers of the same land, is said sections are to be applied .Which of the indispensable to its validity. (Art .1875.) In the face of that statute would the courts allow
two innocent purchasers, if they are both to be regarded as innocent purchasers, should be a mortgage to be valid which had not been recorded, upon the plea of ignorance of the
protected under the provisions of said sections? These questions indicate the difficulty statutory provision, when third parties were interested? May a purchaser of land,
with which we are met in giving meaning and effect to the phrase "innocent purchaser," subsequent to the recorded mortgage, plead ignorance of its existence, and by reason of
in said sections. such ignorance have the land released from such lien? Could a purchaser of land, after the
recorded mortgage, be relieved from the mortgage lien by the plea that he was a bona
May the purchaser of land which has been included in a "second original certificate" ever fide purchaser? May there be a bona fide purchaser of said land, bona fide in the sense
be regarded as an "innocent purchaser," as against the rights or interest of the owner of that he had no knowledge of the existence of the mortgage? We believe the rule that all
the first original certificate, his heirs, assigns, or vendee? The first original certificate is persons must take notice of what the public record contains in just as obligatory upon all
recorded in the public registry. It is never issued until it is recorded. The record notice to persons as the rule that all men must know the law; that no one can plead ignorance of the
all the world. All persons are charged with the knowledge of what it contains. All persons law. The fact that all men know the law is contrary to the presumption. The conduct of
dealing with the land so recorded, or any portion of it, must be charged with notice of men, at times, shows clearly that they do not know the law. The rule, however, is
whatever it contains. The purchaser is charged with notice of every fact shown by the mandatory and obligatory, notwithstanding. It would be just as logical to allow the
record and is presumed to know every fact which the record discloses .This rule is so well defense of ignorance of the existence and contents of a public record.
established that it is scarcely necessary to cite authorities in its support (Northwestern
National Bank vs. Freeman, 171 U.S., 620, 629; Delvin on Real Estate, sections 710, 710 In view, therefore, of the foregoing rules of law, may the purchaser of land from the
[a]). owner of the second original certificate be an "innocent purchaser," when a part or all of
such land had theretofore been registered in the name of another, not the vendor? We are
When a conveyance has been properly recorded such record is constructive notice of its of the opinion that said sections 38, 55, and 112 should not be applied to such purchasers.
contents and all interests, legal and equitable, included therein. (Grandin vs. Anderson, 15 We do not believe that the phrase "innocent purchaser should be applied to such a
Ohio State, 286, 289; Orvis vs. Newell, 17 Conn., 97; Buchanan vs. Intentional Bank, 78 purchaser. He cannot be regarded as an "innocent purchaser" because of the facts
Ill., 500; Youngs vs. Wilson, 27 N.Y., 351; McCabe vs. Grey, 20 Cal., 509; contained in the record of the first original certificate. The rule should not be applied to
Montefiore vs. Browne, 7 House of Lords Cases, 341.) the purchaser of a parcel of land the vendor of which is not the owner of the original
certificate, or his successors. He, in nonsense, can be an "innocent purchaser" of the
portion of the land included in another earlier original certificate. The rule of notice of
Under the rule of notice, it is presumed that the purchaser has examined every instrument what the record contains precludes the idea of innocence. By reason of the prior registry
of record affecting the title. Such presumption is irrebutable. He is charged with notice of there cannot be an innocent purchaser of land included in a prior original certificate and
every fact shown by the record and is presumed to know every fact which an examination in a name other than that of the vendor, or his successors. In order to minimize the
of the record would have disclosed. This presumption cannot be overcome by proof of difficulties we think this is the safe rule to establish. We believe the phrase "innocent
innocence or good faith. Otherwise the very purpose and object of the law requiring a purchaser," used in said sections, should be limited only to cases where unregistered land
record would be destroyed. Such presumption cannot be defeated by proof of want of has been wrongfully included in a certificate under the torrens system. When land is once
knowledge of what the record contains any more than one may be permitted to show that brought under the torrens system, the record of the original certificate and all subsequent
he was ignorant of the provisions of the law. The rule that all persons must take notice of transfers thereof is notice to all the world. That being the rule, could Teus even regarded
the facts which the public record contains is a rule of law. The rule must be absolute. Any as the holder in good fifth of that part of the land included in his certificate of the
variation would lead to endless confusion and useless litigation. appellants? We think not. Suppose, for example, that Teus had never had his lot
registered under the torrens system. Suppose he had sold his lot to the appellee and had
included in his deed of transfer the very strip of land now in question. Could his vendee
be regarded as an "innocent purchaser" of said strip? Would his vendee be an "innocent Without any findings as to costs, it is so ordered.
purchaser" of said strip? Certainly not. The record of the original certificate of the
appellants precludes the possibility. Has the appellee gained any right by reason of the Arellano, C.J., Torrens, and Araullo, JJ., concur.
registration of the strip of land in the name of his vendor? Applying the rule of notice
resulting from the record of the title of the appellants, the question must be answered in
the negative. We are of the opinion that these rules are more in harmony with the purpose Separate Opinion
of Act No. 496 than the rule contended for by the appellee. We believe that the purchaser
from the owner of the later certificate, and his successors, should be required to resort to TRENT, J., dissenting:
his vendor for damages, in case of a mistake like the present, rather than to molest the
holder of the first certificate who has been guilty of no negligence. The holder of the first I dissent.
original certificate and his successors should be permitted to rest secure in their title,
against one who had acquired rights in conflict therewith and who had full and complete
knowledge of their rights. The purchaser of land included in the second original In cases of double or overlapping registration, I am inclined to agree with the reasoning
certificate, by reason of the facts contained in the public record and the knowledge with and authority on which it is held in the majority opinion (first) that the original holder of
which he is charged and by reason of his negligence, should suffer the loss, if any, the prior certificate is entitled to the land as against the original holder of the later
resulting from such purchase, rather than he who has obtained the first certificate and certificate, where there has been no transfer of title by either party to an innocent
who was innocent of any act of negligence. purchaser; both, as is shown in the majority opinion, being at fault in permitting the
double registration to take place; (second) that an innocent purchaser claiming under the
prior certificate is entitled to the land as against the original holder of the later certificate,
The foregoing decision does not solve, nor pretend to solve, all the difficulties resulting and also as against innocent purchasers from the holder of the later certificate; the
from double registration under the torrens system and the subsequent transfer of the land. innocent purchaser being in no wise at fault in connection with the issuance of the later
Neither do we now attempt to decide the effect of the former registration in the ordinary certificate.
registry upon the registration under the torrens system. We are inclined to the view,
without deciding it, that the record under the torrens system, supersede all other registries.
If that view is correct then it will be sufficient, in dealing with land registered and But I am of opinion that neither the authorities cited, nor the reasoning of the majority
recorded alone. Once land is registered and recorded under the torrens system, that record opinion sustains the proposition that the original holder of the prior certificate is entitled
alone can be examined for the purpose of ascertaining the real status of the title to the to the land as against an innocent purchaser from the holder of the later certificate.
land.
As to the text-book authorities cited in the majority opinion, it is sufficient to say that the
It would be seen to a just and equitable rule, when two persons have acquired equal rights rules laid down by both Hogg and Niblack are mere general rules, admittedly subject to
in the same thing, to hold that the one who acquired it first and who has complied with all exception, and of course of no binding force or authority where the reasoning upon which
the requirements of the law should be protected. these rules are based is applicable to the facts developed in a particular case.

In view of our conclusions, above stated, the judgment of the lower court should be and is In its last analysis the general rule laid down in the majority opinion rests upon the
hereby revoked. The record is hereby returned to the court now having and exercising the proposition set forth in the last page of the opinion wherein it is said that "it would seem
jurisdiction heretofore exercised by the land court, with direction to make such orders and to be a just and equitable rule, when two persons have acquired equal rights in the same
decrees in the premises as may correct the error heretofore made in including the land in thing, to hold that the one who acquired it first and who has complied with all the
the second original certificate issued in favor of the predecessor of the appellee, as well as requirements of the law should be protected." The rule, as applied to the matter in hand,
in all other duplicate certificates issued. may be stated as follows: It would seem to be a just and equitable rule when two persons
have acquired separate and independent registered titles to the same land, under the Land
Registration Act, to hold that the one who first acquired registered title and who has
complied with all the requirements of the law in that regard should be protected, in the the general rule should prevail, because both such original parties must held to have been
absence of any express statutory provision to the contrary. fault and, their equities being equal, preference should be given to the earlier title.

Thus stated I have no quarrel with the doctrine as a statement of the general rule to be The majority opinion further recognizes the soundness of my contention by the reasoning
applied in cases of double or overlapping registration under the Land Registration Act; whereby it undertakes to sustain the application of the general rule in favor of the original
for it is true as stated in the majority opinion that in the adjudication and registration of holder of the earlier certificate against purchasers from the original holder of the later
titles by the Courts of Land Registration "mistakes are bound to occur, and sometimes the certificate, by an attempt to demonstrate that such purchasers can in no event be held to
damage done thereby is irreparable;" and that in the absence of statutory provisions be innocent purchasers; because, as it is said, negligence may and should always be
covering such cases, "it is the duty of the courts to adjust the rights of the parties, under imputed to such a purchaser, so that in no event can he claim to be without fault when it
such circumstances, so as to minimize such damages, taking into consideration all of the appears that the lands purchased by him from the holder of a duly registered certificate of
conditions, and the diligence of the respective parties to avoid them." title are included within the bounds of the lands described in a certificate of title of an
earlier date.
But like most such general rules, it has its exceptions and should not be applied in a case
wherein the reasons on which it is based do not exist, or in cases wherein still more At considerable length the majority opinion (in reliance upon the general rule laid down
forceful reasons demand the application of a contrary rule. The general rule relied upon in under the various systems of land registration, other than those based on the torrens
the majority opinion is a mere application of a well settled equity rule that: "Where system) insists that a purchaser of land land duly registered in the Land Registration
conflicting equities are otherwise equal in merit, that which first occurred will be given Court, is charged with notice of the contents of each and every one of the thousands and
the preference." But it is universally laid down by all the courts which have had occasion tens of thousands of certificates of registry on file in the land registry office, so
to apply this equity rule that "it should be the last test resorted to," and that "it never that negligence may be imputed to him if he does not ascertain that all or any part of the
prevails when any other equitable ground for preference exists." (See 19 Cent. Dig., tit. land purchased by him is included within the boundary lines of any one of the thousands
Equity, par. 181; and may cases cited in 16 Cyc., 139, note 57.) It follows that the general or tens of thousands of tracts of land whose original registry bears an earlier date than the
rules, that in cases of double or overlapping registration the earlier certificate should be date of the original registry of the land purchased by him. It is contended that he cannot
protected, ought not to prevail so as to deprive an innocent purchaser under the later claim to be without fault should he buy such land because, as it is said, it was possible for
certificate of his title of the earlier certificate contributed to the issuance of the later him to discover that the land purchased by him had been made the subject of double or
certificate. Hence the holder of the earlier certificate of title should not be heard to invoke overlapping registration by a comparison of the description and boundary lines of the
the "just and equitable rule" as laid down in the majority opinion, in order to have his thousands of tracts and parcels of land to be found in the land registry office.
own title protected and the title of an innocent purchaser of a later certificate cancelled or
annulled, in any case wherein it appears that the holder of the later certificate was wholly But such ruling goes far to defeat one of the principal objects sought to be attained by the
without fault, while the holder of the issuance of the later certificate, in that he might introduction and adoption of the so-called torrens system for the registration of land. The
have prevented its issuance by merely entering his appearance in court in response to avowed intent of that system of land registration is to relieve the purchase of registered
lawful summons personally served upon him in the course of the proceedings for the lands from the necessity of looking farther than the certificate of title of the vendor in
issuance of the second certificate, and pleading his superior rights under the earlier order that he may rest secure as to the validity of the title to the lands conveyed to him.
certificate, instead of keeping silent and by his silence permitting a default judgment to be And yet it is said in the majority opinion that he is charged with notice of the contents of
entered against him adjudicating title in favor of the second applicant. every other certificate of title in the office of the registrar so that his failure to acquaint
himself with its contents may be imputed to him as negligence.
The majority opinion clearly recognizes the soundness of the principles I am contending
for by reasoning (with which I am inclined to agree) whereby it undertakes to If the rule announced in the majority opinion is to prevail, the new system of land
demonstrate that as between the original holders of the double or overlapping registration registration, instead of making transfers of real estate simple, expenditious and secure,
and instead of avoiding the necessity for expensive and oftimes uncertain searches of the
land record and registries, in order to ascertain the true condition of the title before actual notice that proceedings are pending in that court wherein another applicant,
purchase, will, in many instances, add to the labor, expense and uncertainty of any claiming the land as his own, is seeking to secure its registry in his name. All that is
attempt by a purchaser to satisfy himself as to the validity of the title to lands purchased necessary for him to do is to enter his appearance in those proceedings, invite the court's
by him. As I have said before, one of the principal objects, if not the principal object, of attention to the certificate of title registered in his name, and thus, at the cost of the
the torrens system of land registration upon which our Land Registration Act is avowedly applicant, avoid all the damage and inconvenience flowing from the double or
modelled is to facilitate the transfer of real estate. To that end the Legislature undertakes overlapping registration of the land in question. There is nothing in the new system of
to relieve prospective purchasers and all others dealing in registered lands from the land registration which seems to render it either expedient or necessary to relieve a holder
necessity of looking farther than the certificate of title to such lands furnished by the of a registered title of the duty of appearing and defending that title, when he has actual
Court of Land Registration, and I cannot, therefore, give my consent to a ruling which notice that it is being attacked in a court of competent jurisdiction, and if, as a result of
charges a purchaser or mortgage of registered lands with notice of the contents of every his neglect or failure so to do, his lands become subject to double or overlapping
other certificate of title in the land registry, so that negligence and fault may be imputed registration, he should not be permitted to subject an innocent purchaser, holding under
to him should he be exposed to loss or damages as a result of the lack of such knowledge. the later certificate, to all the loss and damage resulting from the double or overlapping
registration, while he goes scot free and holds the land under a manifest misapplication of
Suppose a prospective purchaser of lands registered under the Land Registration Act the equitable rule that "where conflicting equities are otherwise equal in merit, that which
desires to avoid the imputation of negligence in the event that, unknown to him, such first accrued will be given the preference." It is only where both or neither of the parties
lands have been made the subject of double or overlapping registration, what course are at fault that the rule is properly applicable as between opposing claimants under an
should he pursue? What measures should he adopt in order to search out the information earlier and a later certificate of registry to the same land.
with notice of which he is charged? There are no indexes to guide him nor is there
anything in the record or the certificate of title of the land he proposes to buy which Of course all that is said in the briefs of counsel and the majority opinion as to the right of
necessarily or even with reasonable probability will furnish him a clue as to the fact of the the holder of a certificate to rest secure in his registered title so that those dealing with
existence of such double or overlapping registration. Indeed the only course open to him, registered lands can confidently rely upon registry certificates thereto is equally forceful
if he desires to assure himself against the possibility of double or overlapping registration, by way of argument in favor of the holder of one or the other certificate in case of double
would even seem to be a careful, laborious and extensive comparison of the registered or overlapping registration. The problem is to determine which of the certificate holders
boundary lines contained in the certificate of title of the tract of land he proposes to buy is entitled to the land. The decision of that question in favor of either one must
with those contained in all the earlier certificates of title to be found in the land registry. necessarily have the effect of destroying the value of the registered title of the other and
Assuredly it was never the intention of the author of the new Land Registration Act to to that extent shaking the public confidence in the value of the whole system for the
impose such a burden on a purchaser of duly registered real estate, under penalty that a registration of lands. But, in the language of the majority opinion, "that mistakes are
lack of the knowledge which might thus be acquired may be imputed to him by this court bound to occur cannot be denied and sometimes the damage done thereby is irreparable.
as negligence in ruling upon the respective equities of the holders of lands which have It is the duty of the courts to adjust the rights of the parties under such circumstances so
been the subject of double or overlapping registration. as to minimize the damages, taking into consideration all the conditions and the diligence
of the respective parties to avoid them."lawphil.net
On the other hand, I think that negligence and fault may fairly be imputed to a holder of a
registered certificate of title who stood supinely by and let a default judgment be entered It will be observed that I limit the exception to the general equitable rule, as laid down in
against him, adjudicating all or any part of his registered lands to another applicant, if it the majority opinion, to case wherein the holder of the earlier certificate of title has actual
appears that he was served with notice or had actual notice of the pendency of the notice of the pendency of the proceedings in the course of which the latter certificate of
proceedings in the Court of Land Registration wherein such default judgment was entered. title was issued, or to cases in which he has received personal notice of the pendency of
those proceedings. Unless he has actual notice of the pendency of such proceedings I
The owner of land who enjoys the benefits secured to him by its registry in the Court of readily agree with the reasoning of the majority opinion so far as it holds that negligence,
Land Registration may reasonably be required to appear and defend his title when he has culpable negligence, should not be imputed to him for failure to appear and defend his
title so as to defeat his right to the benefit of the equitable rule. It is true that the order of
publication in such cases having been duly complied with, all the world is charged with of registered titles, negligently or fraudulently and conclusively, to permit default
notice thereof, but it does not necessarily follow that, in the absence of actual notice, judgments to be entered against them adjudicating title to all or a part of their registered
culpable negligence in permitting a default judgment to be entered against him may be lands in favor of other applicants, despite actual notice of the pendency of judicial
imputed to the holder of the earlier certificate so as to defeat his right to the land under proceedings had for that purpose, and this, without adding in any appreciable degree to
the equitable rule favoring the earlier certificate. Such a holding would have the effect (to the security of thir titles, and merely to save them the very slight trouble or inconvenience
quote the language of the majority opinion) of requiring the holder of a certificate of title incident to an entry of appearance in the court in which their own titles were secured, and
to wait indefinitely "in the portals of the court" and to sit in the "mirador de su casa" in inviting attention to the fact that their right, title and ownership in the lands in questions
order to avoid the possibility of losing his lands; and I agree with the writer of the has already been conclusively adjudicated.
majority opinion that to do so would place an unreasonable burden on the holders of such
certificate, which was not contemplated by the authors of the Land Registration Act. But The cases wherein there is a practical possibility of double or overlapping registration
no unreasonable burden is placed upon the holder of a registered title by a rule which without actual notice to the holder of the earlier certificate must in the very nature of
imputes culpable negligence to him when he sits supinely by and lets a judgment in things to be so rare as to be practically negligible. Double or overlapping registration
default be entered against him adjudicating title to his lands in favor of another applicant, almost invariably occurs in relation to lands held by adjoining occupants or claimants. It
despite the fact that he has actual knowledge of the pendency of the proceedings in which is difficult to conceive of a case wherein double registration can take place, in the absence
such judgment is entered and despite the fact that he has been personally served with of fraud, without personal service of notice of the pendency of the proceedings upon the
summons to appear and default his title. holder of the earlier certificate, the statute requiring such notice to be served upon the
owner or occupant of all lands adjoining those for which application for registration is
Taking into consideration all of the conditions and the diligence of the respective parties," made; and the cases wherein an adjoining land owner can, even by the use of fraud,
it seems to me that there is no "equality in merit" between the conflicting equities set up conduct proceedings for the registration of his land to a successful conclusion without
by an innocent purchaser who acquires title to the land under a registered certificate, and actual notice to the adjoining property owners must be rare indeed.
the holder of an earlier certificate who permitted a default judgment to be entered against
him, despite actual notice of the pendency of the proceedings in the course of which the In the case at bar the defendant purchased the land in question from the original holder of
later certificate was issued. a certificate of title issued by the Court of Land Registration, relying upon the records of
the Court of Land Registration with reference thereto and with no knowledge that any
I am convinced, furthermore, that aside from the superior equities of the innocent part of the land thus purchased was included in an earlier certificate of title issued to the
purchaser in cases such as that now under discussion, there are strong reasons of plaintiff. The plaintiff, the holder of the earlier certificate of title, negligently permitted a
convenience and public policy which militate in favor of the recognition of his title rather default judgment to be entered against him in the Court of Land Registration,
than that of the holder of the earlier title. One ruling exposes all persons purchasing or adjudicating part of the lands included in his own certificate of title in favor of another
dealing in registered lands to unknown, unspecified and uncertain dangers, to guard applicant, from whom the defendant in this action acquired title, and this despite the fact
against which all such persons will be put to additional cost, annoyance and labor on that he was an adjoining land owner, had actual notice of the pendency of the proceedings
every occasion when any transaction is had with regard to such lands; while the other and was personally served with summons to appear and defends his rights in the premises.
ruling tends to eliminate consequences so directly adverse to the purpose and object for It seems to me that there can be no reason for doubt as to the respective merits of the
which the land registration law was enacted, and imposes no burden upon any holder of a equities of the parties, and further that the judgment of the majority in favor of the
certificate of registered lands other than that of defending his title on those rare, definite plaintiff will inevitably tend to increase the number of cases wherein registered land
and specific occasions wherein he has actual notice that his title is being challenged in a owners in the future will fail to appear and defend their titles when challenged in other
Court of Land Registration, a proceeding in which the cost and expense is reduced to the proceedings in the Courts of Land Registration, thereby enormously increasing the
minimum by the conclusive character of his certificate of title in support of his claim of possibility and probability of loss and damage to innocent third parties and dealers in
ownership. Furthermore, judgment against the innocent purchaser and in favor of the registered lands generally, arising out of erroneous, double or overlapping registration of
holder of the earlier certificate in a case such as that under consideration must inevitably lands by the Courts of Land Registration. Carson, J., concurs.
tend to increase the danger of double or overlapping registrations by encouraging holders
Republic of the Philippines The plaintiffs, being the registered owners in fee simple of the land in
SUPREME COURT question, necessarily have the lawful right to the physical possession
Manila of the land. The owner of a land has a right to enjoy and possess it,
and he has also the right to recover and repossess the same from any
FIRST DIVISION person occupying it unlawfully.

G.R. No. 86787 May 8, 1992 Art. 428 New Civil Code

MILAGROS TUMULAK BISHOP, JUANA PANGILINAN, EMILIO MAXIMO, The owner has the right to enjoy and dispose of a
ANITA PANGILINAN, MAGDALENA ROSETE, MANUEL DACUT, RECTO thing, without other limitations than those
DIESTA, VIRGINIA NOVICIO, and LINDA BONILLA, petitioners, established by law.
vs.
HONORABLE COURT OF APPEALS and SPOUSES MANUEL AND JESUSA The owner has also a right of action against the
SALANG, respondents. holder and possessor of the thing in order to
recover it.
Saturnino Bactad for petitioners.
There is, therefore, no doubt in law, that the plaintiffs being the
CRUZ, J.: registered owners of the land in question have also the corresponding
right to the recovery and possession of the same. The defendants who
are in physical occupancy of the land belonging to the plaintiffs have
The question presented in this case is not novel. As in previous cases resolving the same no right whatsoever to unjustly withhold the possession of the said
issue, the answer will not change. land from the plaintiffs. The defendants' occupancy of the land in
question is unlawful and in violation of plaintiffs right to the recovery
In dispute are certain portions of a parcel of land situated in Calapacuan, Subic, Zambales, and possession of the land they owned. The evidence presented by
with a total area of 1,652 square meters. These portions are in the possession of the the defendants claiming as per certifications of the Bureau of Forestry
petitioners. The entire parcel is registered in the name of the private respondents under that the land occupied by them is within the alienable and disposable
Transfer Certificate of Title No. T-29018. public land, deserves scant consideration as the said certification are
without basis in law. The moment the land in question was titled in
On January 22, 1985, the private respondents sued the petitioners for recovery of the name of the plaintiffs, it ceased to become a part of the public
possession of the lots in question. The plaintiffs invoked their rights as registered owners domain as the same became the private property of the registered
of the land. In their answer, the defendants claimed that the lots were part of the public owner, the herein plaintiffs. Tax declarations of the land made in the
domain and could not have been registered under the Torrens system. All alleged long names of the defendants are not evidence of title, it appearing that the
and continuous possession of the lots and produced tax declarations in their names. Two land is already titled to the plaintiffs. The registration of the land in
of them maintained that they had acquired their respective lots by virtue of valid contracts the names of the defendants with the Assessor's Office for taxation
of sale. Another based her claim on inheritance. purposes and the payments of real property taxes by the defendants
can not and does not defeat the title of the plaintiffs to the land. The
fact that the defendants have been in occupancy of the land in
After trial, Judge Nicias O. Mendoza of the Regional Trial Court of Olongapo City question for quite a period of time is of no moment as prescription
rendered judgment in favor of the plaintiffs. 1 He held in part as follows: will not ripen into ownership because the land is covered by a torrens
title. Acquisitive prescription will not be available to land titled under that there was actual fraud in securing the title. 3Neither of these requirements has been
Art. 496. established by the petitioners. All they submitted was the certification of the Bureau of
Forestry that the land in question was alienable and disposable public land. The trial court
PREMISED THEREFORE on the foregoing consideration, the Court was correct in ruling that this deserved scant consideration for lack of legal basis. To be
finds and so holds that the plaintiffs being the registered owners of sure, a certification from an administrative body cannot prevail against court decision
the land in question are entitled to the possession of the same, and declaring the land to be registrable.
that the defendants who are occupying the land belonging to the
plaintiffs in violation of the right of the latter, are duty-bound to Significantly, it does not appear in the record that the Director of Forestry, or any other
restore possession of the same to the titled owners, the herein representative of the Government for that matter, entered any opposition to the land
plaintiffs. registration proceedings that led to the issuance of the Original Certificate of Title. No
less importantly, an action to invalidate a certificate of title on the ground of fraud
On appeal, this decision was affirmed by the respondent court on August 22, prescribes after the expiration of one (1) year from the entry of the decree of
1988. 2 Their motion for reconsideration having been denied, the petitioners then came to registration 4 and cannot now be resorted to by the petitioners at this late hour. And
this Court, urging reversal of the courts below. collaterally at that.

They allege that: The strange theory submitted by the petitioners that the owner of registered land must
also possess it does not merit serious attention. The non-presentation by the private
respondents of their tax declarations on the land is no indication that they have never
1. The land in question is part of the public domain and could not acquired ownership thereof or have lost it by such omission.
have been validly registered under the Torrens system.
The second ground must also be rejected.
2. The petitioners have acquired title to their respective lots by laches.
As registered owners of the lots in question, the private respondents have a right to eject
3. In the alternative, they should be considered builders in good faith any person illegally occupying their property. This right is imprescriptible. Even if it be
entitled to the rights granted by Articles 448, 546, 547 and 548 of the supposed that they were aware of the petitioners' occupation of the property, and
Civil Code. regardless of the length of that possession, the lawful owners have a right to demand the
return of their property at any time as long as the possession was unauthorized or merely
The petition has no merit. tolerated, if at all. This right is never barred by laches.

On the first ground, the Court notes that the private respondents' title is traceable to an In urging laches against the private respondents for not protesting their long and
Original Certificate of Title issued way back in 1910 or eighty-two years ago. That continuous occupancy of the lots in question, the petitioners are in effect contending that
certificate is now incontrovertible and conclusive against the whole world. The they have acquired the said lots by acquisitive prescription. It is an elementary principle
resumption of regularity applies to the issuance of that certificate. This presumption that the owner of a land registered under the Torrens system cannot lose it by
covers the finding that the land subject of the certificate was private in nature and prescription. 5
therefore registrable under the Torrens system.
As the Court observed in the early case Legarda v. Saleeby: 6
To sustain an action for annulment of a Torrens certificate for being void ab initio, it
must be shown that the registration court had not acquired jurisdiction over the case and
The real purpose of the Torrens system of land registration is to quite
title to land; to put a stop forever to any question of the legality of the
title, except claims which were noted at the time of registration in the
certificate, or which may arise subsequent thereto. That being the
purpose of the law, it would seem that once the title was registered,
the owner may rest secure, without the necessity of waiting in the
portals of the court, or sitting in the "mirador de su casa," to avoid the
possibility of losing his land.

Applied consistently these many years, this doctrine has been burnished bright with use
and has long become a settled rule of law.

In light of the observations already made, it is obvious that the petitioners cannot invoke
the status of builders in good faith to preserve their claimed rights to the constructions
they have made on the lots in dispute.

A builder in good faith is one who is unaware of any flaw in his title to the land at the
time he builds on it. 7 This definition cannot apply to the petitioners because they knew at
the very outset that they had no right at all to occupy the subject lots.

The petitioners have consistently insisted that the lots were part of the public domain and
even submitted a certification to that effect from the Bureau of Forestry. The land was in
fact registered under the Torrens system and such registration was constructive notice to
the whole world, including the petitioners. Apparently, the petitioners did not take the
trouble of checking such registration. At any rate, the point is that, whether the land be
public or private, the petitioners knew they had no right to occupy it and build on it. The
Court of Appeals was correct in calling them squatters for having entered, without
permission or authority, land that did not belong to them.

In urging reversal of the trial court and the respondent court, the petitioners are asking us
to overturn long established doctrines guaranteeing the integrity of the Torrens system
and the indefeasibility of titles issued thereunder for the protection and peace of mind of
the registered owner against illegal encroachments upon his property. We are not
disposed to take this drastic step on the basis alone of their feeble arguments.
WHEREFORE, the petition is DENIED, with costs against the petitioners. It is so
ordered. Narvasa, C.J., Grio-Aquino, Medialdea and Bellosillo, JJ., concur.
Republic of the Philippines On February 26, 1975, an Original Certificate of Title No. T-1728 covering the property
SUPREME COURT in question was issued to and in the name of the spouses Vivas and Lizardo without the
Manila knowledge of the private respondents and on April 30, 1975, said Spouses executed a
Special Power of Attorney in favor of Irenea Ramirez authorizing the latter to mortgage
FIRST DIVISION the property with the petitioner, National Grains Authority.

G.R. No. L-68741 January 28, 1988 On May 2, 1974, the counsel for the petitioner wrote the Provincial Sheriff in Sta. Cruz,
Laguna, requesting for the extrajudicial foreclosure of the mortgage executed by Irenea
Ramirez on May 18, 1975, covering, among others, the property involved in this case
NATIONAL GRAINS AUTHORITY, plaintiff-appellee, covered by OCT No. T-1728, for unpaid indebtedness in the amount of P63,948.80 in
vs. favor of the petitioner.
INTERMEDIATE APPELLATE COURT, MELECIO MAGCAMIT, NENA
COSICO and EMELITA MAGCAMIT, defendants-appellants.
On May 31, 1974, the Provincial Sheriff caused the issuance of the notice of sale of the
property in question, scheduling the public auction sale on June 28, 1974. The petitioner
PARAS, J.: was the highest and successful bidder so that a Certificate of Sale was issued in its favor
on the same date by the Provincial Sheriff.
This is a petition for review of the decision of the then Intermediate Appellate
Court * (now Court of Appeals) dated January 31, 1984, reversing the decision of the On July 10, 1974, the petitioner in its capacity as attorney-in-fact of the mortgagor sold
Court of First Instance of Laguna and San Pablo City, 8th Judicial District, Branch III, the subject real property in favor of itself. By virtue of the deed of absolute sale, TCT No.
and of the resolution dated August 28, 1984 denying the motion for reconsideration filed T-75171 of the Register of Deeds for the Province of Laguna was issued in the name of
thereof. the petitioner on July 16, 1974. It was only in July 1974, that private respondents learned
that a title in the name of the Vivas spouses had been issued covering the property in
The undisputed facts of this case as found by the Trial Court and the Intermediate question and that the same property had been mortgaged in favor of the petitioner. Private
Appellate Court are as follows: respondent Nena Magcamit offered to pay the petitioner NGA the amount of P40,000.00
which is the balance of the amount due the Vivas spouses under the terms of the absolute
On December 2,1971, the spouses Paulino Vivas and Engracia Lizards, as owners of a deed of sale but the petitioner refused to accept the payment. On July 31, 1974, counsel
parcel of land situated in Bo. San Francisco, Victoria, Laguna, comprising more or less for private respondents made a formal demand on the spouses Vivas and Lizardo to
105,710 square meters, sold for P30,000.00 said property in favor of spouses Melencio comply with their obligation under the terms of the absolute deed of sale; and soon after
Magcamit and Nena Cosico, and Amelita Magcamit (herein private respondents) as reiterated to the NGA, the offer to pay the balance of P40,000.00 due under the absolute
evidenced by "Kasulatan Ng Bilihang Mabiling Muli." This sale with right to repurchase deed of sale. On August 13, 1974 petitioner in its reply informed counsel of private
was recorded in the Office of the Register of Deeds of Laguna on December 6,1971 under respondents that petitioner is now the owner of the property in question and has no
Act No. 3344. On January 31,1972 the sale was made absolute by the spouses Vivas and intention of disposing of the same.
Lizardo in favor of the private respondents for the sum of P90,000.00; P50,000.00 of
which was paid upon the execution of the instrument, entitled "Kasulatan Ng Bilihan The private respondents, who as previously stated, are in possession of subject property
Tuluyan," after being credited with the P30,000.00 consideration of the "Kasulatan Ng were asked by petitioner to vacate it but the former refused. Petitioner filed a suit for
Mabibiling Muli," and the balance of P40,000.00 was to be paid the moment that the ejectment against private respondents in the Municipal Court of Victoria, Laguna, but the
certificate of title is issued. From the execution of said Kasulatan, private respondent case was dismissed.
have remained in peaceful, adverse and open possession of subject property.
On June 4, 1975, private respondents filed a complaint before the then Court of First After proper proceedings, the appellate court rendered its decision on January 31, 1984,
Instance of Laguna and San Pablo City, Branch III, San Pablo City, against the petitioner reversing and setting aside the decision of the trial court as follows:
and the spouses Vivas and Lizardo, praying, among others, that they be declared the
owners of the property in question and entitled to continue in possession of the same, and WHEREFORE, the decision of the lower court is hereby reversed and
if the petitioner is declared the owner of the said property, then, to order it to reconvey or set aside and another one is rendered ordering the National Grains
transfer the ownership to them under such terms and conditions as the court may find just, Authority to execute a deed of reconveyance sufficient in law for
fair and equitable under the premises. (Record on Appeal, pp. 2-11). purposes of registration and cancellation of transfer Certificate of
Title No. T-75171 and the issuance of another title in the names of
In its answer to the complaint, the petitioner (defendant therein) maintained that it was plaintiff-appellants, and ordering defendants-appellees Paulino Vivas
never a privy to any transaction between the private respondents (plaintiffs therein) and and Engracia Lizardo to pay the National Grains Authority the sum of
the spouses Paulino Vivas and Engracia Lizardo that it is a purchaser in good faith and P78,375.00 (Exh. 3) within thirty (30) days from the receipts of the
for value of the property formerly covered by OCT No. 1728; and that the title is now writ of execution. No damages and costs. (Rollo, p. 19).
indefeasible, hence, private respondents' cause of action has' already prescribed. (Record
on Appeal, pp. 16-22). The petitioner filed a motion for reconsideration of the said decision but the same was
denied. (Rollo, p. 26).
After due hearing, the trial court ** rendered its decision on March 17, 1981, in favor of
the petitioner, the dispositive portion of said judgment reading as follows: Hence, this petition.

WHEREFORE, judgment is hereby rendered as follows: In the resolution of May 20, 1985, the petition was given due course and the parties were
required to submit simultaneous memoranda (Rollo, p. 128). The memorandum for the
(1) declaring defendant National Grains Authority the lawful owner petitioner was filed on July 3, 1985 (Rollo, p. 129) while the memorandum for the private
of the property in question by virtue of its indefeasible title to the respondents was filed on August 26, 1985 1 Rollo p. 192).
same;
The main issue in this case is whether or not violation of the terms of the agreement
(2) ordering plaintiffs to turn over possession of the land to defendant between the spouses Vivas and Lizardo, the sellers, and private respondents, the buyers,
National Grains Authority; to deliver the certificate of title to the latter, upon its issuance, constitutes a breach of trust
sufficient to defeat the title and right acquired by petitioner NGA, an innocent purchaser
(3) ordering defendants-spouses Paulino Vivas and Engracia Lizardo for value.
to pay plaintiffs the sum of P56,000.00 representing the amount paid
pursuant to the Kasulatan Ng Bilihang Tuluyan marked Exhibit "3", It is undisputed that: (1) there are two deeds of sale of the same land in favor of private
with legal interest thereon from January 31, 1972 until the amount is respondents, namely: (a) the conditional sale with right to repurchase or the 'Kasulatan
paid, to pay an additional amount of P5,000.00 for and as attorney's Ng Bilihang Mabibiling Muli" which was registered under Act 3344 and (b) the deed of
fees, an additional amount of Pl0,000.00 as moral damages, another absolute sale or "Kasulatan ng Bilihang Tuluyan" which was not registered; (2) the
amount of P5,000.00 by way of exemplary damages and to pay the condition that the Certificate of Title will be delivered to the buyers upon its issuance and
costs of this suit. (Rollo, P. 35). upon payment of the balance of P40,000.00 is contained in the deed of absolute sale; and
(3) the land in question at the time of the execution of both sales was not yet covered by
The private respondents interposed an appeal from the decision of the trial court to the the Torrens System of registration.
Intermediate Appellate Court.
It is axiomatic, that while the registration of the conditional sale with right of repurchase Thus, under Section 44 of P.D. 1529, every registered owner receiving a certificate of
may be binding on third persons, it is by provision of law "understood to be without title in pursuance of a decree of registration, and every subsequent purchaser of registered
prejudice to third party who has better right" (Section 194 of the Administrative Code, as land taking a certificate of title for value and in good faith, shall hold the same free from
amended by Act No. 3344). In this case, it will be noted that the third party NGA, is a all encumbrances except those noted on the certificate and any of the encumbrances
registered owner under the Torrens System and has obviously a better right than private which may be subsisting, and enumerated in the law. Under said provision, claims and
respondents and that the deed of absolute sale with the suspensive condition is not liens of whatever character, except those mentioned by law as existing, against the land
registered and is necessarily binding only on the spouses Vivas and Lizardo and private prior to the issuance of certificate of title, are cut off by such certificate if not noted
respondents. thereon, and the certificate so issued binds the whole world, including the government
(Aldecoa and Co. vs. Warner Barns & Co., 30 Phil. 209 [1915]; Snyder vs. Fiscal of
In their complaint at the Regional Trial Court, private respondents prayed among others, Cebu and Avila, 42 Phil. 766 [1922]). Under said ruling, if the purchaser is the only party
for two alternative reliefs, such as: (a) to be declared the owners of the property in who appears in the deeds and the registration of titles in the property registry, no one
question or (b) to order the declared owner to reconvey or transfer the ownership of the except such purchaser may be deemed by law to be the owner of the properties in
property in their favor. question (Ibid). Moreover, no title to registered land in derogation to that of the registered
owner shall be acquired by prescription or adverse possession (Umbay vs. Alecha, 135
SCRA 427 [1985]).
Private respondents claim a better right to the property in question by virtue of the
Conditional Sale, later changed to a deed of Absolute Sale which although unregistered
under the Torrens System allegedly transferred to them the ownership and the possession It does not appear that private respondents' claim falls under any of the exceptions
of the property in question. In fact, they argue that they have been and are still in provided for under Section 44 of P.D. 1529 which can be enforced against petitioner
possession of the same openly, continuously, publicly under a claim of ownership herein.
adverse to all other claims since the purchase on December 2, 1971 (Rollo, p. 165). It is
stressed that not until the month of July, 1974 did the plaintiff learn that a title had been Thus, it has been invariably restated by this Court, that "The real purpose of the Torrens
issued covering the property in question (Rollo, p. 15). System is to quiet title to land and to stop forever any question as to its legality. "Once a
title is registered, the owner may rest secure, without the necessity of waiting in the
Time and time again, this Court has ruled that the proceedings for the registration of title portals of the court, or sitting on the "mirador su casato," avoid the possibility of losing
to land under the Torrens System is an action in rem not in personam, hence, personal his land." "An indirect or collateral attack on a Torrens Title is not allowed (Dominga vs.
notice to all claimants of the res is not necessary in order that the court may have Santos, 55 Phil. 361; Singian vs. Manila Railroad, 62 Phil. 467)."
jurisdiction to deal with and dispose of the res. Neither may lack of such personal notice
vitiate or invalidate the decree or title issued in a registration proceeding, for the State, as The only exception to this rule is where a person obtains a certificate of title to a land
sovereign over the land situated within it, may provide for the adjudication of title in a belonging to another and he has full knowledge of the rights of the true owner. He is then
proceeding in rem or one in the nature of or akin a to proceeding in rem which shall be considered as guilty of fraud and he may be compelled to transfer the land to the
binding upon all persons, known or unknown (Moscoso vs. Court of appeals, 128 SCRA defrauded owner so long as the property has not passed to the hands of an innocent
719 [1984], citing: City of Manila vs. Lack, et al., 19 Phil. 324, 337; Roxas vs. Enriquez, purchaser for value (Angeles vs. Sania, 66 Phil. 444 [1938], emphasis supplied).
29 Phil. 31; Director of Lands vs. Roman Catholic Archbishop of Manila, 41 Phil. 120;
Aguilar vs. Caogdan, 105 Phil. 661). It is thus evident that respondents' right over the It will be noted that the spouses Vivas and Lizardo never committed any fraud in
property was barred by res judicata when the decree of registration was issued to spouses procuring the registration of the property in question. On the contrary, their application
Vivas and Lizards. It does not matter that they may have had some right even the right of for registration which resulted in the issuance of OCT No. 1728 was with complete
ownership, BEFORE the grant of the Torrens Title. knowledge and implied authority of private respondents who retained a portion of the
consideration until the issuance to said spouses of a certificate of title applied for under
the Torrens Act and the corresponding delivery of said title to them. The question
therefore, is not about the validity of OCT No. 1728 but in the breach of contract between and effect forever (Domingo vs. The Mayon Realty Corp. et al., 102 Phil. 32 [19571).
private respondents and the Vivas spouses. Petitioner NGA was never a privy to this Assuming, therefore, that there was fraud committed by the sellers against the buyers in
transaction. Neither was it shown that it had any knowledge at the time of the execution the instant case, petitioner NGA who was not privy therein cannot be made to suffer the
of the mortgage, of the existence of the suspensive condition in the deed of absolute sale consequences thereof As correctly declared by the trial court, the National Grains
much less of its violation. Nothing appeared to excite suspicion. The Special Power of Authority is the lawful owner of the property in question by virtue of its indefeasible title.
Attorney was regular on its face; the OCT was in the name of the mortgagor and the
NGA was the highest bidder in the public auction. Unquestionably, therefore, the NGA is As to private respondents' alternative prayer that the declared owner be ordered to
an innocent purchaser for value, first as an innocent mortgagee under Section 32 of P.D. reconvey or transfer the ownership of the property in their favor, it is clear that there is
1529 and later as innocent purchaser for value in the public auction sale. absolutely no reason why petitioner, an innocent purchaser for value, should reconvey the
land to the private respondents.
Private respondents claim that NGA did not even field any representative to the land
which was not even in the possession of the supposed mortgagors, nor present any PREMISES CONSIDERED, the decision of the Court of Appeals is REVERSED and
witness to prove its allegations in the ANSWER nor submit its DEED OF MORTGAGE SET ASIDE, and the decision of the Court of First Instance of Laguna and San Pablo
to show its being a mortgages in good faith and for value (Rollo, p. 110). City, now Regional Trial Court, is REINSTATED.

Such contention is, however, untenable. Well settled is the rule that all persons dealing SO ORDERED.
with property covered by a torrens certificate of title are not required to go beyond what
appears on the face of the title. When there is nothing on the certificate of title to indicate
any cloud or vice in the ownership of the property, or any encumbrance thereon, the Teehankee, C.J., Narvasa, Cruz and Gancayco, JJ., concur.
purchaser is not required to explore further than what the torrens title upon its face
indicates in quest for any hidden defect or inchoate right that may subsequently defeat his
right thereto (Centeno vs. Court of Appeals, 139 SCRA 545 [1985]).

More specifically, the Court has ruled that a bank is not required before accepting a
mortgage to make an investigation of the title of the property being given as security (Phil.
National Cooperative Bank vs. Carandang Villalon, 139 SCRA 570 [1985]), and where
innocent third persons like mortgagee relying on the certificate of title acquire rights over
the property, their rights cannot be disregarded (Duran vs. IAC, 138 SCRA 489 [1985]).

Under the circumstances, the Regional Trial Court could not have erred in ruling that
plaintiffs (private respondents herein) complaint insofar as it prays that they be declared
owners of the land in question can not prosper in view of the doctrine of indefeasibility of
title under the Torrens System, because it is an established principle that a petition for
review of the decree of registration will not prosper even if filed within one year from the
entry of the decree if the title has passed into the hands of an innocent purchaser for value
(Pres. Decree No. 1529, Sec. 32). The setting aside of the decree of registration issued in
land registration proceedings is operative only between the parties to the fraud and the
parties defrauded and their privies, but not against acquirers in good faith and for value
and the successors in interest of the latter; as to them the decree shall remain in full force
Republic of the Philippines Alfonso Hitalia and Eduardo S. Baranda The Court issued a writ of
SUPREME COURT possession which Gregorio Perez, Maria P. Gotera and Susana Silao
Manila refused to honor on the ground that they also have TCT No. 25772
over the same Lot No. 4517. The Court, after considering the private
THIRD DIVISION respondents' opposition and finding TCT No. 25772 fraudulently
acquired, ordered that the writ of possession be carried out. A motion
for reconsideration having been denied, a writ of demolition was
G.R. No. 81163 September 26, 1988 issued on March 29, 1982. Perez and Gotera filed a petition for
certiorari and prohibition with the Court of Appeals. On August 6,
EDUARDO S. BARANDA and ALFONSO HITALIA, petitioners, 1982, the Court of Appeals denied the petition. Perez and Gotera filed
vs. the petition for review on certiorari denominated as G.R. No. 62042
HONORABLE JUDGE TITO GUSTILO, ACTING REGISTER OF DEEDS before the Supreme Court. As earlier stated the petition was denied in
AVITO SACLAUSO, HONORABLE COURT OF APPEALS, and ATTY. a resolution dated January 7,1983. The motion for reconsideration
HECTOR P. TEODOSIO, respondents. was denied in another resolution dated March 25, 1983, which also
stated that the denial is final. This decision in G.R. No. 62042, in
Eduardo S. Baranda for petitioners. accordance with the entry of judgment, became final on March 25,
1983. The petitioners in the instant case G.R. No. 64432--contend
that the writs of possession and demolition issued in the respondent
Rico & Associates for private respondents. court should now be implemented; that Civil Case No. 00827 before
the Intermediate Appellate Court was filed only to delay the
implementation of the writ; that counsel for the respondent should be
held in contempt of court for engaging in a concerted but futile effort
to delay the execution of the writs of possession and demolition and
GUTIERREZ, JR., J.:
that petitioners are entitled to damages because of prejudice caused
by the filing of this petition before the Intermediate Appellate Court.
Eduardo S. Baranda and Alfonso Hitalia were the petitioners in G.R. No. 64432 and the On September 26, 1983, this Court issued a Temporary Restraining
private respondents in G.R. No. 62042. The subject matter of these two (2) cases and the Order ' to maintain the status quo, both in the Intermediate Appellate
instant case is the same a parcel of land designated as Lot No. 4517 of the Cadastral Court and in the Regional Trial Court of Iloilo. Considering that
Survey of Sta. Barbara, Iloilo covered by Original Certificate of Title No. 6406. (l)there is merit in the instant petition for indeed the issues discussed
in G.R. No. 64432 as raised in Civil Case No. 00827 before the
The present petition arose from the same facts and events which triggered the filing of the respondent court have already been passed upon in G.R. No. 62042;
earlier petitions. These facts and events are cited in our resolution dated December 29, and (2) the Temporary Restraining Order issued by the Intermediate
1983 in G.R. No. 64432, as follows: Appellate Court was only intended not to render the petition moot
and academic pending the Court's consideration of the issues, the
Court RESOLVED to DIRECT the respondent Intermediate
. . . This case has its origins in a petition for reconstitution of title
Appellate Court not to take cognizance of issues already resolved by
filed with the Court of First Instance of Iloilo involving a parcel of
this Court and accordingly DISMISS the petition in Civil Case No.
land known as Lot No. 4517 of the Sta. Barbara Cadastre covered by
00827. Immediate implementation of the writs of possession and
Original Certificate of Title No. 6406 in the name of Romana Hitalia.
demolition is likewise ordered. (pp. 107-108, Rollo G.R. No.
Eventually, Original Certificate of Title No. 6406 was cancelled and
64432)
Transfer Certificate of Title No. 106098 was issued in the names of
On May 9, 1984, the Court issued a resolution denying with finality a motion for The Acting Register of Deeds of Iloilo is further ordered to register
reconsideration of the December 29, 1983 resolution in G.R. No. 64432. On this same the Subdivision Agreement of Eduardo S. Baranda and Alfonso
date, another resolution was issued, this time in G.R. No. 62042, referring to the Regional Hitalia as prayed for." (p. 466, Rollo--G.R. No. 64432)
Trial Court of Iloilo the ex-parte motion of the private respondents (Baranda and Hitalia)
for execution of the judgment in the resolutions dated January 7, 1983 and March 9, 1983. The above order was set aside on October 8, 1984 upon a motion for reconsideration and
In the meantime, the then Intermediate Appellate Court issued a resolution dated manifestation filed by the Acting Registrar of Deeds of Iloilo, Atty. Helen P. Sornito, on
February 10, 1984, dismissing Civil Case No. 00827 which covered the same subject the ground that there was a pending case before this Court, an Action for Mandamus,
matter as the Resolutions above cited pursuant to our Resolution dated December 29, Prohibition, Injunction under G.R. No. 67661 filed by Atty. Eduardo Baranda, against the
1983. The resolution dated December 29, 1983 in G.R. No. 64432 became final on May former which remained unresolved.
20, 1984.
In view of this development, the petitioners filed in G.R. No. 62042 and G.R. No. 64432
Upon motions of the petitioners, the Regional Trial Court of Iloilo, Branch 23 presided ex-parte motions for issuance of an order directing the Regional Trial Court and Acting
by Judge Tito G. Gustilo issued the following order: Register of Deeds to execute and implement the judgments of this Court. They prayed
that an order be issued:
Submitted are the following motions filed by movants Eduardo S.
Baranda and Alfonso Hitalia through counsel dated August 28, 1984: 1. Ordering both the Regional Trial Court of Iloilo Branch XXIII,
under Hon. Judge Tito G. Gustilo and the acting Register of Deeds
(a) Reiterating Motion for Execution of Judgment of Resolutions Helen P. Sornito to register the Order dated September 5, 1984 of the
dated January 7, 1983 and March 9, 1983 Promulgated by Honorable lower court;
Supreme Court (First Division) in G.R. No. 62042;
2. To cancel No.T-25772. Likewise to cancel No.T-106098 and once
(b) Motion for Execution of Judgment of Resolution dated December cancelled to issue new certificates of title to each of Eduardo S.
29, 1983 Promulgated by Honorable Supreme Court (First Division) Baranda and Alfonso Hitalia;
in G.R. No. 64432;
Plus other relief and remedies equitable under the premises. (p. 473,
(c) The Duties of the Register of Deeds are purely ministerial under 64432 Rollo)
Act 496, therefore she must register all orders, judgment, resolutions
of this Court and that of Honorable Supreme Court. Acting on these motions, we issued on September 17,1986 a Resolution in G.R. No.
62042 and G.R. No. 64432 granting the motions as prayed for. Acting on another motion
Finding the said motions meritorious and there being no opposition of the same nature filed by the petitioners, we issued another Resolution dated October 8,
thereto, the same is hereby GRANTED. 1986 referring the same to the Court Administrator for implementation by the judge
below.
WHEREFORE, Transfer Certificate of Title No. T-25772 is hereby
declared null and void and Transfer Certificate of Title No. T-106098 In compliance with our resolutions, the Regional Trial Court of Iloilo, Branch 23 presided
is hereby declared valid and subsisting title concerning the ownership by Judge Tito G. Gustilo issued two (2) orders dated November 6,1986 and January
of Eduardo S. Baranda and Alfonso Hitalia, all of Sta. Barbara 6,1987 respectively, to wit:
Cadastre.
ORDER
This is an Ex-parte Motion and Manifestation submitted by the Baranda and Alfonso Hitalia, which certificate shall contain a
movants through counsel on October 20, 1986; the Manifestation of memorandum of the annulment of the outstanding duplicate. (pp.
Atty. Helen Sornito, Register of Deeds of the City of Iloilo, and 286-287, Rollo 64432)
formerly acting register of deeds for the Province of Iloilo dated
October 23, 1986 and the Manifestation of Atty. Avito S. Saclauso, On February 9, 1987, Atty. Hector Teodosio, the counsel of Gregorio Perez, private
Acting Register of Deeds, Province of Iloilo dated November 5, 1986. respondent in G.R. No. 64432 and petitioner in G.R. No. 62042, filed a motion for
explanation in relation to the resolution dated September 17, 1986 and manifestation
Considering that the motion of movants Atty. Eduardo S. Baranda asking for clarification on the following points:
and Alfonso Hitalia dated August 12, 1986 seeking the full
implementation of the writ of possession was granted by the a. As to the prayer of Atty. Eduardo Baranda for the cancellation of
Honorable Supreme Court, Second Division per its Resolution dated TCT T-25772, should the same be referred to the Court of Appeals
September 17,1986, the present motion is hereby GRANTED. (as mentioned in the Resolution of November 27, 1985) or is it
already deemed granted by implication (by virtue of the Resolution
WHEREFORE, the Acting Register of Deeds, Province of Iloilo, is dated September 17, 1986)?
hereby ordered to register the Order of this Court dated September 5,
1984 as prayed for. b. Does the Resolution dated September 17, 1986 include not only the
implementation of the writ of possession but also the cancellation of
xxx xxx xxx TCT T-25772 and the subdivision of Lot 4517? (p. 536, Rollo
4432)
ORDER
Acting on this motion and the other motions filed by the parties, we issued a resolution
This is a Manifestation and Urgent Petition for the Surrender of dated May 25, 1987 noting all these motions and stating therein:
Transfer Certificate of Title No. T-25772 submitted by the petitioners
Atty. Eduardo S. Baranda and Alfonso Hitalia on December 2, 1986, xxx xxx xxx
in compliance with the order of this Court dated November 25, 1 986,
a Motion for Extension of Time to File Opposition filed by Maria Since entry of judgment in G.R. No. 62042 was made on January 7,
Provido Gotera through counsel on December 4, 1986 which was 1983 and in G.R. No. 64432 on May 30, 1984, and all that remains is
granted by the Court pursuant to its order dated December 15, 1986. the implementation of our resolutions, this COURT RESOLVED to
Considering that no Opposition was filed within the thirty (30) days refer the matters concerning the execution of the decisions to the
period granted by the Court finding the petition tenable, the same is Regional Trial Court of Iloilo City for appropriate action and to apply
hereby GRANTED. disciplinary sanctions upon whoever attempts to trifle with the
implementation of the resolutions of this Court. No further motions in
WHEREFORE, Maria Provido Gotera is hereby ordered to surrender these cases will be entertained by this Court. (p. 615, Rollo-64432)
Transfer Certificate of Title No. T-25772 to this Court within ten (10)
days from the date of this order, after which period, Transfer In the meantime, in compliance with the Regional Trial Court's orders dated November 6,
Certificate of Title No. T-25772 is hereby declared annulled and the 1986 and January 6, 1987, Acting Register of Deeds AvitoSaclauso annotated the order
Register of Deeds of Iloilo is ordered to issue a new Certificate of declaring Transfer Certificate of Title No. T-25772 as null and void, cancelled the same
Title in lieu thereof in the name of petitioners Atty. Eduardo S. and issued new certificates of titles numbers T-111560, T-111561 and T-111562 in the
name of petitioners Eduardo S. Baranda and Alfonso Hitalia in lieu of Transfer That the undersigned hereby asks for a reconsideration of the said
Certificate of TItle No. T-106098. order based on the second paragraph of Section 77 of P.D. 1529, to
wit:
However, a notice of lis pendens "on account of or by reason of a separate case (Civil
Case No. 15871) still pending in the Court of Appeals" was carried out and annotated in "At any time after final judgment in favor of the
the new certificates of titles issued to the petitioners. This was upheld by the trial court defendant or other disposition of the action such
after setting aside its earlier order dated February 12, 1987 ordering the cancellation of lis as to terminate finally all rights of the plaintiff in
pendens. and to the land and/or buildings involved, in any
case in which a memorandum or notice of Lis
This prompted the petitioners to file another motion in G.R, No. 62042 and G.R. No. Pendens has been registered as provided in the
64432 to order the trial court to reinstate its order dated February 12, 1987 directing the preceding section, the notice of Lis Pendens shall
Acting Register of Deeds to cancel the notice of lis pendens in the new certificates of be deemed cancelled upon the registration of a
titles. certificate of the clerk of court in which the
action or proceeding was pending stating the
manner of disposal thereof."
In a resolution dated August 17, 1987, we resolved to refer the said motion to the
Regional Trial Court of Iloilo City, Branch 23 for appropriate action.
That the lis pendens under Entry No. 427183 was annotated on T-
106098, T-111560, T-111561 and T-111562 by virtue of a case
Since respondent Judge Tito Gustilo of the Regional Trial Court of Iloilo, Branch 23 docketed as Civil Case No. 15871, now pending with the
denied the petitioners' motion to reinstate the February 12, 1987 order in another order Intermediate Court of Appeals, entitled, "Calixta Provido, Ricardo
dated September 17, 1987, the petitioners filed this petition for certiorari, prohibition and Provido, Sr., Maria Provido and Perfecto Provido, Plaintiffs, versus
mandamus with preliminary injunction to compel the respondent judge to reinstate his Eduardo Baranda and Alfonso Hitalia, Respondents."
order dated February l2, 1987 directing the Acting Register of Deeds to cancel the notice
of lis pendens annotated in the new certificates of titles issued in the name of the
petitioners. That under the above-quoted provisions of P.D. 152, the cancellation
of subject Notice of Lis Pendens can only be made or deemed
cancelled upon the registration of the certificate of the Clerk of Court
The records show that after the Acting Register of Deeds annotated a notice of in which the action or proceeding was pending, stating the manner of
is pendens on the new certificates of titles issued in the name of the petitioners, the disposal thereof.
petitioners filed in the reconstitution case an urgent ex-parte motion to immediately
cancel notice of lis pendens annotated thereon.
Considering that Civil Case No. 1587, upon which the Notice of Lis
Pendens was based is still pending with the Intermediate Court of
In his order dated February 12, 1987, respondent Judge Gustilo granted the motion and Appeals, only the Intermediate Court of Appeals and not this
directed the Acting Register of Deeds of Iloilo to cancel the lis pendens found on Honorable Court in a mere cadastral proceedings can order the
Transfer Certificate of Title Nos. T-106098; T-111560; T-111561 and T-111562. cancellation of the Notice of Lis Pendens. (pp. 68-69, Rollo)

Respondent Acting Register of Deeds Avito Saclauso filed a motion for reconsideration Adopting these arguments and on the ground that some if not all of the plaintiffs in Civil
of the February 12, 1987 order stating therein: Case No. 15871 were not privies to the case affected by the Supreme Court resolutions,
respondent Judge Tito Gustilo set aside his February 12, 1987 order and granted the
Acting Register of Deeds' motion for reconsideration.
The issue hinges on whether or not the pendency of the appeal in Civil Case No. 15871 the transfer certificate of title in the name of petitioner Maria Provido
with the Court of Appeals prevents the court from cancelling the notice of lis pendens in Gotera and her other co-owners. (p. 3, Rollo; Emphasis supplied)
the certificates of titles of the petitioners which were earlier declared valid and subsisting
by this Court in G.R. No. 62042 and G.R. No. 64432. A corollary issue is on the nature of It thus appears that the plaintiffs in Civil Case No. 15871 were privies to G.R. No. 62042
the duty of a Register of Deeds to annotate or annul a notice of lis pendens in a torrens contrary to the trial court's findings that they were not.
certificate of title.
G.R. No. 62042 affirmed the order of the then Court of First Instance of Iloilo in the
Civil Case No. 15871 was a complaint to seek recovery of Lot No. 4517 of Sta. Barbara reconstitution proceedings declaring TCT No. 25772 in the name of Providos over Lot
Cadastre Iloilo, (the same subject matter of G.R. No 62042 and G.R. No. 64432) from No. 4517, Sta. Barbara Cadastre null and void for being fraudulently obtained and
petitioners Baranda and Hitalia filed by Calixta Provido, Ricardo Provido, Maxima declaring TCT No. 106098 over the same parcel Lot No. 4517, Sta. Barbara Cadastre in
Provido and Perfecta Provido before the Regional Trial Court of Iloilo, Branch 23. At the the name of petitioners Eduardo Baranda and Alfonso Hitalia valid and subsisting.
instance of Atty. Hector P. Teodosio, the Provides' counsel, a notice of is pendens was
annotated on petitioners' Certificate of Title No. T-106098 covering Lot No. 4517, Sta.
Barbara Cadastre. The decision in G.R. No. 62042 became final and executory on March 25,1983 long
before Civil Case No. 15871 was filed.
Acting on a motion to dismiss filed by the petitioners, the court issued an order dated
October 24, 1984 dismissing Civil Case No. 15871. Under these circumstances, it is crystal clear that the Providos, private respondents herein,
in filing Civil Case No. 15871 were trying to delay the full implementation of the final
decisions in G.R. No. 62042 as well as G.R. No. 64432 wherein this Court ordered
The order was then appealed to the Court of Appeals. This appeal is the reason why immediate implementation of the writs of possession and demolition in the reconstitution
respondent Judge Gustilo recalled the February 12, 1987 order directing the Acting proceedings involving Lot No. 4517, Sta. Barbara Cadastre.
Register of Deeds to cancel the notice of lis pendens annotated on the certificates of titles
of the petitioners.
The purpose of a notice of lis pendens is defined in the following manner:
This petition is impressed with merit.
Lis pendens has been conceived to protect the real rights of the party
causing the registration thereof With the lis pendens duly recorded,
Maria Provido Gotera was one of the petitioners in G.R. No. 62042. Although Calixta he could rest secure that he would not lose the property or any part of
Provido, Ricardo Provido, Maxima Provido and Perfecta Provido, the plaintiffs in Civil it. For, notice of lis pendens serves as a warning to a prospective
Case No. 15871 were not impleaded as parties, it is very clear in the petition that Maria purchaser or incumbrancer that the particular property is in litigation;
Provido was acting on behalf of the Providos who allegedly are her co-owners in Lot No. and that he should keep his hands off the same, unless of course he
4517, Sta. Barbara Cadastre as shown by Transfer Certificate of Title No. T-25772 issued intends to gamble on the results of the litigation. (Section 24, Rule 14,
in her name and the names of the plaintiffs in Civil Case No. 15871, among others. RuIes of Court; Jamora v. Duran, et al., 69 Phil. 3, 11; I Martin, Rules
(Annex "E" G.R. No. 62042, p. 51, Rollo) In fact, one of the issues raised by petitioners of Court, p. 415, footnote 3, citing cases.) (Natanov. Esteban, 18
Maria Provido Gotera and Gregoria Perez in G.R. No. 62042 was as follows: SCRA 481, 485-486)

xxx xxx xxx The private respondents are not entitled to this protection. The facts obtaining in this case
necessitate the application of the rule enunciated in the cases of Victoriano v. Rovila (55
2. Whether or not, in the same reconstitution proceedings, respondent Phil. 1000), Municipal Council of Paranaque v. Court of First Instance of Rizal (70 Phil.,
Judge Midpantao L. Adil had the authority to declare as null and void 363) and Sarmiento v. Ortiz (10 SCRA 158), to the effect that:
We have once held that while ordinarily a notice of pendency which Barbara Cadastre in the light of the final decisions in G.R. No. 62042 and G.R. No.
has been filed in a proper case, cannot be cancelled while the action is 64432.
pending and undetermined, the proper court has the discretionary
power to cancel it under peculiar circumstances, as for instance, The next question to be determined is on the nature of the duty of the Register of Deeds
where the evidence so far presented by the plaintiff does not bear out to annotate and/or cancel the notice of lis pendens in a torrens certificate of title.
the main allegations of his complaint, and where the continuances of
the trial, for which the plaintiff is responsible, are unnecessarily
delaying the determination of the case to the prejudice of the Section 10, Presidential Decree No. 1529 states that "It shall be the duty of the Register
defendant. (Victoriano v. Rovira, supra; The Municipal Council of of Deeds to immediately register an instrument presented for registration dealing with
Paranaque v. Court of First Instance of Rizal, supra) real or personal property which complies with all the requisites for registration. ... . If the
instrument is not registrable, he shall forthwith deny registration thereof and inform the
presentor of such denial in writing, stating the ground or reasons therefore, and advising
The facts of this case in relation to the earlier cases brought all the way to the Supreme him of his right to appeal by consulta in accordance with Section 117 of this Decree."
Court illustrate how the private respondents tried to block but unsuccessfuly the already
final decisions in G.R. No. 62042 and G.R. No. 64432.
Section 117 provides that "When the Register of Deeds is in doubt with regard to the
proper step to be taken or memoranda to be made in pursuance of any deed, mortgage or
Parenthetically, respondent Judge Tito Gustilo abused his discretion in sustaining the other instrument presented to him for registration or where any party in interest does not
respondent Acting Register of Deeds' stand that, the notice of lis pendens in the agree with the action taken by the Register of Deeds with reference to any such
certificates of titles of the petitioners over Lot No. 4571, Barbara Cadastre cannot be instrument, the question shall be submitted to the Commission of Land Registration by
cancelled on the ground of pendency of Civil Case No. 15871 with the Court of Appeals. the Register of Deeds, or by the party in interest thru the Register of Deeds. ... ."
In upholding the position of the Acting Register of Deeds based on Section 77 of
Presidential Decree No. 1529, he conveniently forgot the first paragraph thereof which
provides: The elementary rule in statutory construction is that when the words and phrases of the
statute are clear and unequivocal, their meaning must be determined from the language
employed and the statute must be taken to mean exactly what it says. (Aparri v. Court of
Cancellation of lis pendens. Before final judgment, a notice of lis Appeals, 127 SCRA 231; Insular Bank of Asia and America Employees' Union
pendens may be cancelled upon Order of the Court after proper [IBAAEU] v. Inciong, 132 SCRA 663) The statute concerning the function of the
showing that the notice is for the purpose of molesting the adverse Register of Deeds to register instruments in a torrens certificate of title is clear and leaves
party, or that it is not necessary to protect the rights of the party who no room for construction. According to Webster's Third International Dictionary of the
caused it to be registered. It may also be cancelled by the Register of English Language the word shall means "ought to, must, ...obligation used to express a
Deeds upon verified petition of the party who caused the registration command or exhortation, used in laws, regulations or directives to express what is
thereof. mandatory." Hence, the function of a Register of Deeds with reference to the registration
of deeds encumbrances, instruments and the like is ministerial in nature. The respondent
This Court cannot understand how respondent Judge Gustilo could have been misled by Acting Register of Deeds did not have any legal standing to file a motion for
the respondent Acting Register of Deeds on this matter when in fact he was the same reconsideration of the respondent Judge's Order directing him to cancel the notice of lis
Judge who issued the order dismissing Civil Case No. 15871 prompting the private pendens annotated in the certificates of titles of the petitioners over the subject parcel of
respondents to appeal said order dated October 10, 1984 to the Court of Appeals. The land. In case of doubt as to the proper step to be taken in pursuance of any deed ...
records of the main case are still with the court below but based on the order, it can be or other instrument presented to him, he should have asked the opinion of the
safely assumed that the various pleadings filed by the parties subsequent to the motion to Commissioner of Land Registration now, the Administrator of the National Land Title
dismiss filed by the petitioners (the defendants therein) touched on the issue of the and Deeds Registration Administration in accordance with Section 117 of Presidential
validity of TCT No. 25772 in the name of the Providos over Lot Number 4571, Sta. Decree No. 1529.
In the ultimate analysis, however, the responsibility for the delays in the full
implementation of this Court's already final resolutions in G.R. No. 62042 and G.R. No.
64432 which includes the cancellation of the notice of lis pendens annotated in the
certificates of titles of the petitioners over Lot No. 4517 of the Sta. Barbara Cadastre falls
on the respondent Judge. He should never have allowed himself to become part of
dilatory tactics, giving as excuse the wrong impression that Civil Case No. 15871 filed by
the private respondents involves another set of parties claiming Lot No. 4517 under their
own Torrens Certificate of Title.

WHEREFORE, the instant petition is GRANTED. The February 12, 1987 order of the
Regional Trial Court of Iloilo, Branch 23 is REINSTATED. All subsequent orders issued
by the trial court which annulled the February 12, 1987 order are SET ASIDE. Costs
against the private respondents.

SO ORDERED.

Fernan, C.J., Feliciano, Bidin and Cortes, JJ., concur.


Republic of the Philippines Sale for the sum of P400.00 executed by the registered owner, conveying an undivided
SUPREME COURT portion of an area of 3,710 square meters only in favor of Florentino Gabayan, this
Manila Original Certificate of Title No. 548 is hereby cancelled with respect to said area of 3,710
square meters and in lieu thereof, the name of the vendee ... is hereby substituted to
EN BANC succeed to all rights, participation in interest of the vendor. ...

G.R. No. L-20611 May 8, 1969 Date of Instrument: January 25, 1955, ...

AURELIO BALBIN and FRANCISCO BALBIN, petitioners, xxx xxx xxx


vs.
REGISTER OF DEEDS OF ILOCOS SUR, respondent. Entry No. 5659. Sale of portion.

Vicente Llanes for petitioners. Sale for the sum of P100.00 executed by the registered owner, conveying an undivided
Office of the Solicitor General for respondent. portion of an area of 16,713 square meters in favor of Roberto Bravo, this Original
Manuel A. Argel for respondents third parties affected. Certificate of Title No. 548 is hereby cancelled with respect to said undivided portion ...
and in lieu thereof the name of the vendee ... is hereby substituted to succeed to all rights,
MAKALINTAL, J.: participation and interest of the vendor ...

Appeal from the resolution of the Commissioner of Land Registration in LRC Consulta Date of Instrument: June 9, 1953. ...
No. 366.
Entry No. 5660. Sale of portion.
On November 15, 1961 petitioners presented to the register of deeds of Ilocos Sur a
duplicate copy of the registered owner's certificate of title (OCT No. 548) and an Sale for the sum of P400.00 executed by the registered owner, conveying an
instrument entitled "Deed of Donation inter-vivos," with the request that the same be undivided portion of an area of 15,000 square meters in favor of Juana Gabayan,
annotated on the title. Under the terms of the instrument sought to be annotated one this Certificate of Title No. 548 is hereby cancelled with respect to said
Cornelio Balbin, registered owner of the parcel of land described in OCT No. 548, undivided portion ... and in lieu thereof the name of the vendee ... is hereby
appears to have donated inter-vivos an undivided two-thirds (/) portion thereof in favor substituted to succeed to all rights, participation and interest of the vendor ...
of petitioners. The entire area of the land is 11.2225 hectares.
Date of Instrument: February 12, 1952. ...
The register of deeds denied the requested annotation for being "legally defective or
otherwise not sufficient in law." It appears that previously annotated in the memorandum The final part of the annotations referring to the abovementioned sales contains an
of encumbrances on the certificate are three separate sales of undivided portions of the additional memorandum stating that "three co-owner's duplicate certificates of title No.
land earlier executed by Cornelio Balbin in favor of three different buyers. The pertinent 548 have been issued (by the register of deeds of Ilocos Sur) in the name of Florentino
entries read: Gabayan, Roberto Bravo and Juana Gabayan upon verbal request of Mr. Andres Cabeldo,
Notary Public of Caoayan, I. Sur, for and in the name of the vendees, this 5th day of
Entry No. 5658. Sales. January, 1956 at Vigan, I. Sur." Mainly because these three other co-owner's copies of the
certificate of title No. 548 had not been presented by petitioners, the Register of Deeds until otherwise declared by a court of competent jurisdiction. There being several copies
refused to make the requested annotation. of the same title in existence, it is easy to see how their integrity may be adversely
affected if an encumbrance, or an outright conveyance, is annotated on one copy and not
Unsatisfied, petitioners referred the matter to the Commissioner of Land Registration, on the others. The law itself refers to every copy authorized to be issued as a duplicate of
who subsequently upheld the action of the Register of Deeds in a resolution dated April the original, which means that both must contain identical entries of the transactions,
10, 1962. With respect to the principal point in controversy, the Commissioner observed: particularly voluntary ones, affecting the land covered by the title. If this were not so, if
different copies were permitted to carry differing annotations, the whole system of
Torrens registration would cease to be reliable.
(1) It appears that the donor is now merely a co-owner of the property described
in the Original Certificate of Title No. 548, having previously sold undivided
portions thereof on three different occasions in favor of three different buyers. One other ground relied upon by the Land Registration Commissioner in upholding the
Consequently, aside from the owner's duplicate issued to Cornelio Balbin, there action taken by the Register of Deeds of Ilocos Sur is that since the property subject of
are now three co-owner's duplicates which are presumably in the possession of the donation is presumed conjugal, that is, property of the marriage of the donor, Cornelio
the three buyers. Accordingly, in addition to the owner's duplicate of Original Balbin, and his deceased wife, Nemesia Mina, "there should first be a liquidation of the
Certificate of Title No. 548, the three co-owner's duplicates must likewise be partnership before the surviving spouse may make such a conveyance." This legal
surrendered. The claim of counsel for the donees that the issuance of the three conclusion may appear too general and sweeping in its implications, for without a
co-owner's duplicates was unauthorized is beside the point. Unless and until a previous settlement of the partnership a surviving spouse may dispose of his aliquot share
court of competent jurisdiction rules to the contrary, these titles are presumed to or interest therein subject of course to the result of future liquidation. Nevertheless, it
have been lawfully issued.lawphi1.et is not to be denied that, if the conjugal character of the property is assumed, the deed of
donation executed by the husband, Cornelio Balbin, bears on its face an infirmity which
justified the denial of its registration, namely, the fact that the two-thirds portion of said
Without presenting those three (3) other duplicates of the title, petitioners would want to property which he donated was more than his one-half share, not to say more than what
compel annotation of the deed of donation upon the copy in their possession, citing remained of such share after he had sold portions of the same land to three other parties.
section 55 of Act 496, which provides that "the production of the owner's duplicate
certificate of title whenever any voluntary instrument is presented for registration shall be
conclusive authority from the registered owner to the register of deeds to make a It appears that there is a case pending in the Court of First Instance of Ilocos Sur (CC No.
memorandum of registration in accordance with such instrument." Under this provision, 2221), wherein the civil status of the donor Cornelio Balbin and the character of the land
according to petitioners, the presentation of the other copies of the title is not required, in question are in issue, as well as the validity of the different conveyances executed by
first, because it speaks of "registered owner" and not one whose claim to or interest in the him. The matter of registration of the deed of donation may well await the outcome of
property is merely annotated on the title, such as the three vendees-co-owners in this case; that case, and in the meantime the rights of the interested parties could be protected by
and secondly, because the issuance of the duplicate copies in their favor was illegal or filing the proper notices of lis pendens.
unauthorized.
IN VIEW OF THE FOREGOING, the decisions of the Register of Deeds of Ilocos Sur
We find no merit in petitioners' contention. Section 55, supra, obviously assumes that and that of the Commissioner of Land Registration are affirmed. No pronouncement as to
there is only one duplicate copy of the title in question, namely, that of the registered costs.Reyes, J.B.L., Dizon, Zaldivar, Sanchez, Fernando, Teehankee and Barredo, JJ.,
owner himself, such that its production whenever a voluntary instrument is presented concur.
constitutes sufficient authority from him for the register of deeds to make the Capistrano, J., took no part. Concepcion, C.J., and Castro, J., are on
corresponding memorandum of registration. In the case at bar, the three other copies of leave.Footnotes*Section 43. Certificates where land registered in names of two or more
the title were in existence, presumably issued under section 43 * of Act 496. As correctly persons. Where two or more persons are registered owners as tenants in common, or
observed by the Land Registration Commissioner, petitioners' claim that the issuance of otherwise, one owner's duplicate certificate may be issued for the whole land, or a
those copies was unauthorized or illegal is beside the point, its legality being presumed separate duplicate may be issued to each for his undivided share.
Republic of the Philippines To effect the registration of the aforesaid deed of absolute Sale, it is
SUPREME COURT necessary that the property be first liquidated and transferred in the name of the
Manila surviving spouse and the heirs of the deceased wife by means of extrajudicial
settlement or partition and that the consent of such other heir or heirs must be
EN BANC procured by means of another document ratifying this sale executed by their
father.
G.R. No. L-22486 March 20, 1968
In view of such refusal, Almirol went to the Court of First Instance of Agusan
on a petition for mandamus (sp. civ. case 151), to compel the Register of Deeds to
TEODORO ALMIROL, petitioner-appellant, register the deed of sale and to issue to him the corresponding transfer certificate of title,
vs. and to recover P5,000 in moral damages and P1,000 attorney's fees and expenses of
THE REGISTER OF DEEDS OF AGUSAN, respondent-appellee. litigation. It is Almirol's assertion that it is but a ministerial duty of the respondent to
perform the acts required of him, and that he (Almirol) has no other plain, speedy and
Tranquilino O. Calo, Jr. for petitioner-appellant. adequate remedy in the ordinary course of law.
Office of the Solicitor General for respondent-appellee.
In his answer with counterclaim for P10,000 damages, the respondent reiterated
CASTRO, J.: the grounds stated in his letter of May 21, 1962, averred that the petitioner has "other
legal, plain, speedy and adequate remedy at law by appealing the decision of the
On June 28, 1961 Teodoro Almirol purchased from Arcenio Abalo a parcel of respondent to the Honorable Commissioner of Land Registration," and prayed for
land situated in the municipality of Esperanza, province of Agusan, and covered by dismissal of the petition.
original certificate of title P-1237 in the name of "Arcenio Abalo, married to Nicolasa M.
Abalo." Sometime in May, 1962 Almirol went to the office of the Register of Deeds of In its resolution of October 16, 1963 the lower court, declaring that
Agusan in Butuan City to register the deed of sale and to secure in his name a transfer "mandamus does not lie . . . because the adequate remedy is that provided by Section 4 of
certificate of title. Registration was refused by the Register of Deeds upon the following Rep. Act 1151", dismissed the petition, with costs against the petitioner.
grounds, inter alia, stated in his letter of May 21, 1962:
Hence the present appeal by Almirol.
1. That Original Certificate of Title No. P-1237 is registered in the name of
Arcenio Abalo, married to Nicolasa M. Abalo, and by legal presumption, is The only question of law tendered for resolution is whether mandamus will lie
considered conjugal property; to compel the respondent to register the deed of sale in question.

2. That in the sale of a conjugal property acquired after the effectivity of the Although the reasons relied upon by the respondent evince a sincere desire on
New Civil Code it is necessary that both spouses sign the document; but his part to maintain inviolate the law on succession and transmission of rights over real
properties, these do not constitute legal grounds for his refusal to register the deed.
3. Since, as in this case, the wife has already died when the sale was made, the Whether a document is valid or not, is not for the register of deeds to determine; this
surviving husband can not dispose of the whole property without violating the function belongs properly to a court of competent jurisdiction.1
existing law (LRC Consulta No. 46 dated June 10, 1958).
Whether the document is invalid, frivolous or intended to harass, is
not the duty of a Register of Deeds to decide, but a court of competent
jurisdiction. (Gabriel vs. Register of Deeds of Rizal, et al., L-17956, Sept. 30, issue involves a question of law, said decision may be appealed to the Supreme
1953). Court within thirty days from and after receipt of the notice thereof.

. . . the supposed invalidity of the contracts of lease is no valid The foregoing notwithstanding, the court a quo correctly dismissed the petition
objection to their registration, because invalidity is no proof of their non- for mandamus. Section 4 abovequoted provides that "where any party in interest does not
existence or a valid excuse for denying their registration. The law on agree with the Register of Deeds . . . the question shall be submitted to the Commissioner
registration does not require that only valid instruments shall be registered. of Land Registration," who thereafter shall "enter an order prescribing the step to be
How can parties affected thereby be supposed to know their invalidity before taken or memorandum to be made," which shall be "conclusive and binding upon all
they become aware, actually or constructively, of their existence or of their Registers of Deeds." This administrative remedy must be resorted to by the petitioner
provisions? If the purpose of registration is merely to give notice, then before he can have recourse to the courts.
questions regarding the effect or invalidity of instruments are expected to be
decided after, not before, registration. It must follow as a necessary ACCORDINGLY, the Resolution of the lower court of October 16, 1969, is
consequence that registration must first be allowed, and validity or effect affirmed, at petitioner's cost.1wph1.t
litigated afterwards. (Gurbax Singh Pablo & Co. vs. Reyes and Tantoco, 92
Phil. 182-183).
Reyes, J.B.L., Dizon, Makalintal, Bengzon, J.P., Zaldivar, Angeles and Fernando, JJ.,
concur.
Indeed, a register of deeds is entirely precluded by section 4 of Republic Act Concepcion, C.J., is on leave.
1151 from exercising his personal judgment and discretion when confronted with the Sanchez, J., concurs in the result.
problem of whether to register a deed or instrument on the ground that it is invalid. For
under the said section, when he is in doubt as to the proper step to be taken with respect
to any deed or other instrument presented to him for registration, all that he is supposed to Footnotes
do is to submit and certify the question to the Commissioner of Land Registration who
shall, after notice and hearing, enter an order prescribing the step to be taken on the 1Inre Consulta filed by Francisco on behalf of Cabantog, 67 Phil. 222, 228;
doubtful question. Section 4 of R.A. 1151 reads as follows: Smith Bell & Co., Ltd. vs. Register of Deeds of Davao, 96 Phil. 53, 61;
Register of Deeds, Pasig, Rizal vs. heirs of Hi Caiji, et al., 99 Phil. 25, 29-31;
Reference of doubtful matters to Commissioner of Land Registration. Mendoza vs. Abrera, et al., L-10519, April 30, 1959; Agricultural Credit
When the Register of Deeds is in doubt with regard to the proper step to be Cooperative Association of Hinibiran vs. Yulo Yusay, et al., L-13313, April 28,
taken or memorandum to be made in pursuance of any deed, mortgage, or other 1960; Dulay, et al., vs. Herrera, L-17084, August 30, 1962.
instrument presented to him for registration, or where any party in interest does
not agree with the Register of Deeds with reference to any such matter, the
question shall be submitted to the Commissioner of Land Registration either
upon the certification of the Register of Deeds, stating the question upon which
he is in doubt, or upon the suggestion in writing by the party in interest; and
thereupon the Commissioner, after consideration of the matter shown by the
records certified to him, and in case of registered lands, after notice to the
parties and hearing, shall enter an order prescribing the step to be taken or
memorandum to be made. His decision in such cases shall be conclusive and
binding upon all Registers of Deeds: Provided, further, That when a party in
interest disagrees with the ruling or resolution of the Commissioner and the
Republic of the Philippines for registration of title. Respondent asserted that neither applicants nor their predecessors-
SUPREME COURT in-interest have been in open, continuous, exclusive and notorious possession and
Manila occupation of the subject lot since June 12, 1945 or earlier as required by Section 48(b) of
Commonwealth Act No. 141, as amended by Presidential Decree (P.D.) No. 1073; that
THIRD DIVISION applicants failed to adduce any muniment of title to prove their claims; that the tax
declaration appended to the application does not appear genuine and merely shows
pretended possession of recent vintage; that the application was filed beyond the period
G.R. No. 175746 March 12, 2008 allowed under P.D. No. 892; and that the subject lot is part of the public domain which
cannot be the subject of private appropriation.
CHARLES L. ONG, Petitioner,
vs. On January 16, 2002, the trial court rendered a Decision in favor of petitioner and his
REPUBLIC OF THE PHILIPPINES, Respondent. brothers, viz:

DECISION The foregoing evidences presented by the applicant indubitably established sufficient
basis to grant the applicant (sic) for registration. Originally, the whole parcel of land was
YNARES-SANTIAGO, J.: owned by spouses Teofilo Abellara and Abella Charmine who acquired the same by
virtue of a Deed of Sale from Cynthia Cacho, Agustin Cacho, Jr., Jasmin Cacho, Jover
This petition for review on certiorari assails the April 25, 2006 Decision1 of the Court of Cacho and Lauro Cacho. Later, they sold the same parcel of land to spouses Tony C.
Appeals in CA-G.R. CV No. 76085, which reversed and set aside the January 16, 2002 Villamil and Alicia Bautista, who in turn sold the same land to herein applicants.
Decision2 of the Municipal Trial Court of Mangaldan, Pangasinan in Land Registration
Case No. 99-023, and the November 20, 2006 Resolution 3 which denied petitioners The same parcel of land has been declared in the name of the applicant and her
motion for reconsideration. predecessors-in-interest and its taxes has (sic) been religiously paid.

The antecedent facts are as follows. The said circumstances further show that the possession and ownership of the applicant
and her (sic) predecessors-in-interest over the same parcel of land has (sic) been
On July 1, 1999, petitioner Charles L. Ong (petitioner) in his behalf and as duly continuous and peaceful under bona fide claim of ownership before the filing of the
authorized representative of his brothers, namely, Roberto, Alberto and Cesar, filed an instant application for registration on [July 1, 1999].
Application for Registration of Title4 over Lot 15911 (subject lot) situated in Barangay
Anolid, Mangaldan, Pangasinan with an area of five hundred seventy four (574) square WHEREFORE, after confirming the Order of General Default, the Court hereby orders
meters, more or less. They alleged that they are the co-owners of the subject lot; that the and decrees the registration of a parcel of land as shown on plan ap-01-004897 approved
subject lot is their exclusive property having acquired the same by purchase from spouses by the Bureau of Land(s) situated in Barangay Anolid, Mangaldan, Pangasinan,
Tony Bautista and Alicia Villamil on August 24, 1998; that the subject lot is presently containing an area of Five Hundred Seventy Four (574) square meters, subject of the
unoccupied; and that they and their predecessors-in-interest have been in open, application for registration of title, in accordance with Presidential Decree No. 1529, in
continuous and peaceful possession of the subject lot in the concept of owners for more favor of CHARLIE L. ONG in his behalf and as representative of his brothers namely,
than thirty (30) years. ROBERTO L. ONG, ALBERTO L. ONG and CESAR L. ONG.

After due notice and publication, only respondent Republic of the Philippines
(respondent), represented by the Office of the Solicitor General, opposed the application
Furnish copies of this Decision to the Office of the Solicitor General, Makati City, Metro The petition lacks merit.
Manila, the Office of the Provincial Prosecutor, Dagupan City, Atty. Celestino Domingo
Jr., the Office of the Land Registration Authority, Quezon City, as well as the applicant. Section 14(1) of P.D. 1529 ("Property Registration Decree"), as amended, provides

SO ORDERED.5 SEC. 14. Who may apply. The following persons may file in the proper Court of First
Instance an application for registration of title to land, whether personally or through their
Aggrieved, respondent appealed to the Court of Appeals which rendered the assailed duly authorized representatives:
Decision, the dispositive portion of which reads:
(1) Those who by themselves or through their predecessors-in-interest have been in open,
WHEREFORE, the instant appeal is GRANTED. Accordingly, the decision of the court a continuous, exclusive and notorious possession and occupation of alienable and
quo granting the application for registration of title of applicants-appellees is disposable lands of the public domain under a bona fide claim of ownership since June 12,
REVERSED and SET ASIDE. No pronouncement as to costs. 1945, or earlier.

SO ORDERED.6 Thus, pursuant to the aforequoted provision of law, applicants for registration of title
must prove: (1) that the subject land forms part of the disposable and alienable lands of
In reversing the decision of the trial court, the Court of Appeals found that the subject lot the public domain, and (2) that they have been in open, continuous, exclusive and
is part of the alienable and disposable lands of the public domain. Thus, it was incumbent notorious possession and occupation of the same under a bona fide claim of ownership
upon petitioner to prove that they possessed the subject lot in the nature and for the since June 12, 1945, or earlier.8 These requisites involve questions of fact which are not
duration required by law. However, petitioner failed to prove that he or his predecessors- proper in a petition for review on certiorari. Factual findings of the court a quo are
in-interest have been in adverse possession of the subject lot in the concept of owner generally binding on this Court except for certain recognized exceptions, as is the case
since June 12, 1945 or earlier as mandated by Section 14(1) of P.D. 1529. It noted that here, where the trial court and the Court of Appeals arrived at conflicting findings. 9 After
the earliest tax declaration which petitioner presented is dated 1971. Consequently, a careful review of the records, we sustain the findings and conclusions of the Court of
petitioner could not fairly claim possession of the land prior to 1971. Neither was Appeals.
petitioner able to prove that he or his predecessors-in-interest actually occupied the
subject lot prior to the filing of the application. Thus, the trial court erred in granting the There is no dispute that the subject lot is classified as alienable and disposable land of the
application for registration of title over the subject lot. public domain. The Report10 dated January 17, 2000 of the Bureau of Lands stated that
the subject lot is "within the alienable and disposable zone as classified under Project 50
Hence, this petition raising the following issues: L.C. Map No. 698 and released and classified as such on November 21, 1927."11 This
finding is, likewise, embodied in the Report12 dated January 7, 1999 of the Department of
Environment and Natural Resources Community Environment and Natural Resources
1. WHETHER OR NOT PETITIONER, TOGETHER WITH HIS BROTHERS, Office (DENR-CENRO) and the blue print Copy13 of the plan covering the subject lot.
NAMELY, ROBERTO L. ONG, ALBERTO L. ONG AND CEZAR L. ONG, However, petitioner failed to prove that he or his predecessors-in-interest have been in
HAVE REGISTRABLE OWNERSHIP OVER THE REAL PROPERTY open, continuous, exclusive and notorious possession and occupation of the subject lot
SUBJECT MATTER OF LAND REGISTRATION CASE NO. 99-023, AND since June 12, 1945 or earlier.

2. WHETHER OR NOT THE FINDINGS AND CONCLUSION OF THE The records show that petitioner and his brothers bought the subject lot from spouses
FORMER SPECIAL FOURTH DIVISION OF THE COURT OF APPEALS Tony Bautista and Alicia Villamil on August 24, 1998, 14 who in turn purchased the same
THAT THE SUBJECT REAL PROPERTY IS A PUBLIC LAND IS from spouses Teofilo Abellera and Abella Sarmen on January 16, 1997. 15 The latter
CORRECT.7
bought the subject lot from Cynthia, Agustin Jr., Jasmin, Omir and Lauro, all surnamed do not constitute the "well-nigh incontrovertible" evidence necessary in cases of this
Cacho, on July 10, 1979.16 The earliest tax declaration which was submitted in evidence nature.25 Accordingly, the Court of Appeals did not err in reversing the Decision of the
was Tax Declaration No. 2560617 issued in 1971 in the names of spouses Agustin Cacho trial court and in denying his application for registration of title over the subject lot.
and Eufrosinia Baustista. While tax declarations are not conclusive proof of ownership,
they constitute good indicia of possession in the concept of owner and a claim of title WHEREFORE, in view of the foregoing, the petition is DENIED. The April 25, 2006
over the subject property.18 Even if we were to tack petitioners claim of ownership over Decision of the Court of Appeals in CA-G.R. CV No. 76085 which reversed and set aside
the subject lot to that of their alleged predecessors-in-interest, spouses Agustin Cacho and the January 16, 2002 Decision of the Municipal Trial Court of Mangaldan, Pangasinan in
Eufrosinia Baustista in 1971, still this would fall short of the required possession from Land Registration Case No. 99-023, and the November 20, 2006 Resolution denying the
June 12, 1945 or earlier.1avvphi1 motion for reconsideration, are AFFIRMED.

Further, as correctly pointed by the Court of Appeals, possession alone is not sufficient to Costs against petitioner.
acquire title to alienable lands of the public domain because the law requires possession
and occupation. As held in Republic v. Alconaba:19
SO ORDERED.
The law speaks of possession and occupation. Since these words are separated by the
conjunction and, the clear intention of the law is not to make one synonymous with the CONSUELO YNARES-SANTIAGO
other. Possession is broader than occupation because it includes constructive possession.
When, therefore, the law adds the word occupation, it seeks to delimit the all
encompassing effect of constructive possession. Taken together with the words open,
continuous, exclusive and notorious, the word occupation serves to highlight the fact that
for an applicant to qualify, his possession must not be a mere fiction. Actual possession
of a land consists in the manifestation of acts of dominion over it of such a nature as a
party would naturally exercise over his own property.20

Petitioner admitted that after he and his brothers bought the subject lot from spouses
Tony Bautista and Alicia Villamil in 1998, neither he nor his brothers actually occupied
the subject lot.21 No improvements were made thereon and the most that they did was to
visit the lot on several occasions.22 Petitioners predecessor-in-interest, Tony Bautista
testified that he and his wife never actually occupied the subject lot from the time they
bought the same from spouses Teofilo Abellera and Abella Sarmen in 1997. 23 Aside from
these two testimonies, no other evidence was presented to establish the character of the
possession of the subject lot by petitioners other alleged predecessors-in-interest. Clearly,
petitioners evidence failed to establish specific acts of ownership to substantiate the
claim that he and his predecessors-in-interest possessed and occupied the subject lot in
the nature and duration required by law.

The burden of proof in land registration cases rests on the applicant who must show by
clear, positive and convincing evidence that his alleged possession and occupation of the
land is of the nature and duration required by law.24 Unfortunately, petitioners evidence
Republic of the Philippines This petition for review on certiorari seeks to set aside the Decision 1 of the Court of
SUPREME COURT Appeals dated January 9, 2002 in CA-G.R. SP No. 64337 entitled "Genuino Ice Company,
Manila Inc. vs. Hon. Victorino P. Evangelista, Nelsie B. Caete, et al.," and its Resolution2 dated
June 26, 2002, dismissing petitioners "Second Amended Complaint" in Civil Case No.
THIRD DIVISION Q-99-36483 filed in Branch 223 of the Regional Trial Court of Quezon City.

G.R. No. 154080 January 22, 2008 Records show that on January 11, 1999, petitioners filed a complaint for cancellation of
title to property covered by Transfer Certificate of Title (TCT) Nos. N-
140441;3 14399;4 RT-94384 (292245);5 RT-94794 (292246);6 and 292247.7 Petitioners
NELSIE B. CAETE, RONA ANAS, MILAGROSA APUAN, ERLINDA AQUINO, alleged that said titles are spurious, fictitious and were issued "under mysterious
GODOFREDO AQUINO, CORITA BARREDO, TESSIE BARREDO, JESUS circumstances," considering that the holders thereof including their predecessors-in-
BATRINA, ALBERTO BUENAVENTURA, BONIFACIO BUENAVENTURA, interest were never in actual, adverse and physical possession of the property, rendering
EUSEBIO CAPIRAL, MARIO CAPIRAL, LOLITA CAPIRAL, ELENA them ineligible to acquire title to the said property under the Friar Lands Act. 8 Petitioners
CAPIRAL, LETICIA CAPIRAL, RENATO CAPIRAL, ELY CABANGON, also sought to nullify Original Certificate of Title (OCT) No. 614 from which the
ERWIN CATALUNA, JESSIE CONRADO, JOEL CONRADO, NARCISIO foregoing titles sought to be cancelled originated or were derived.
CONRADO, RICARDO CALAMPIANO, ALUMNIO CORSANES, NILO
COLATOY, MARJETO DAYAN, HENRY DIAZ, SALVACION ESMANDE,
REYNALDO FUENTEBELLA, GERRY GEQUILLANA, DELSIE GARCIA, Respondent Genuino Ice Co., Inc. filed a motion to dismiss9 on the ground that the
NERISSA GONZALES, VISITACION JUNSAY, ESTELA JOVEN, JOSE complaint states no cause of action because petitioners are not real parties-in-interest; that
LANZUELA, MARLON MALANGAYON, RENATO MARCELO, ANITA no relief may be granted as a matter of law; and that petitioners failed to exhaust
MARZONIA, MARCELINO MONTALBO, AMADO MULI, JR., LEONITA administrative remedies, but it was denied by the trial court. Respondent moved for
MULI, EDUARDO OLVIDO, ALMARIO PACON, ASUNCION PACON, reconsideration but the same was denied.
SALVACION PAGAYUNAN, ESTER PANTALEON, SHERLITA RABE, ANITA
REYES, MEDELYN RIOS, BERTITO RIVAS, ENGRACIA RIVERA, GERALYN On November 4, 1999, petitioners filed a "Second Amended Complaint" 10 which sought
RIVERA, ARMANDO RIVERA, MA. MERCY SHERVA, ALEXANDER to annul, in addition to the titles already alleged in the original complaint, TCT Nos.
SANGALAN, ERNESTO SANTIAGO, JOY SANTIAGO, ELENA TALION, JOE 274095 and 274096;11 274097 and 274098;12 and 274099.13
RANDY TRESVALLES, ELIAS VALENZUELA, GERRY VALENZUELA,
LILIBETH VALENZUELA, JOSEPHINE VICTORINO, JOJO VICTORINO, The Second Amended Complaint alleged the following causes of action, as well as the
MAXIMINO VICTORINO, NOEL VICTORINO, REYNANTE VICTORINO, remedy sought to be obtained, thus:
ROBERTO VICTORINO and JOVITO VILLAREAL, represented by NELSIE B.
CAETE, petitioners,
vs. 4. That plaintiffs (petitioners) and their predecessors-in-interest are among
GENUINO ICE COMPANY, INC., respondent. those who have been in actual, adverse, peaceful and continuous possession in
concept of owners of unregistered parcels of land situated at Sitio Mabilog,
Barangay Culiat, Quezon City, Metro Manila, which parcels of land are more
DECISION particularly described as follows:

YNARES-SANTIAGO, J.: (1) "A parcel of unregistered land known as Lot 668, situated at
Barangay Culiat, Quezon City x x x."
(2) "A parcel of unregistered land known as Lot 669, situated at (1) Declaring as null and void ab initio OCT 614 and all transfer certificates of
Barangay Culiat, Quezon City x x x." title derived therefrom;

5. That the above-described real property is a portion of a friar land known as (2) Declaring as null and void defendants transfer certificates of title over the
"Piedad Estate," which property is intended for distribution among the bona property in litigation;
fide occupants thereof pursuant to the Friar Lands Act.
(3) Ordering defendant Register of Deeds of Quezon City to cancel defendants
6. That transfer certificates of title allegedly having originated or derived from transfer certificates of title and all transfer certificates of title derived therefrom;
Original Certificate of Title No. 614 were issued by the Register of Deeds of
Quezon City, which transfer certificates of title are in truth and in fact fictitious, (4) Declaring the plaintiffs as bona fide occupants of the property in litigation
spurious and null and void, for the following reasons: (a) that no record of any pursuant to the provisions of the Friar Lands Act and other existing laws.14
agency of the government shows as to how and in what manner was OCT 614
issued; (b) that no record of any proceedings whatsoever, whether judicial or
administrative, can support defendants claim that the above-described property Respondent moved to dismiss the Second Amended Complaint on the following grounds:
originated from OCT 614; and (c) that the transfer certificates of title over the
above-described property were issued under mysterious circumstances for the a) The complaint states no cause of action because: (1) on the allegations alone,
above-named defendants and their so-called predecessors-in-interest never had plaintiffs (petitioners) are not real parties in interest who may bring suit to
any actual, adverse, physical possession of the said property, thus, not allowed cancel defendants (including respondent) titles; (2) based on the allegations
to acquire title over the property in litigation pursuant to the Friar Lands Act. and prayer of the complaint, no relief, as a matter of law, may be granted;

7. That defendants are holders of transfer certificates of title of the above- b) Prescription has set in;
described property, which transfer certificates of title are null and void, for
reasons specifically mentioned in Paragraph 6 hereof x x x; c) There are earlier similar complaints (Civil Case Nos. Q-95-22834 and Q-95-
23111) filed by a different set of plaintiffs against a different set of defendants
8. That the acts in acquiring and keeping the said transfer certificates of title in but which involve the same subject matter, cause of action and allegations of
violation of the Friar Lands Act and other existing laws are prejudicial to the plaintiffs, with respect to the cancellation of OCT 614 and succeeding titles
plaintiffs rights over the above-described property. derived from it. Said complaints have since been dismissed by Branch 93 of the
Regional Trial Court of Quezon City, the dismissal of which is the subject of a
9. That equity demands that defendants transfer certificates of title as specified pending certiorari proceeding in the appellate court.15
in Paragraph 7 hereof be declared fictitious, spurious and null and void ab initio.
On January 3, 2001,16 the trial court denied respondents motion to dismiss the Second
PRAYER Amended Complaint. Its motion for reconsideration was likewise denied hence
respondent filed a petition for certiorari with the Court of Appeals.
WHEREFORE, premises considered, it is most respectfully prayed of this
Honorable Court that judgment be rendered in favor of plaintiffs and against The appellate court granted respondents petition for certiorari and dismissed petitioners
defendants: Second Amended Complaint for failure to state a cause of action. Hence, the instant
petition raising the following issues:
A. THAT THE COURT OF APPEALS ERRED IN DECLARING THAT THE States, approved on July 1, 1902, not from individual persons but from certain companies,
COMPLAINT FILED BY THE PETITIONERS WITH THE REGIONAL a society and a religious order. Under the Friar Lands Act, only "actual settlers and
TRIAL COURT OF QUEZON CITY IN CIVIL CASE NO. Q-99-36483 DOES occupants at the time said lands are acquired by the Government" were given preference
NOT STATE A VALID CAUSE OF ACTION; to lease, purchase, or acquire their holdings, in disregard of the settlement and occupation
of persons before the government acquired the lands. 21
B. THAT THE COURT OF APPEALS ERRED IN DECLARING THAT THE
PETITIONERS ARE NOT REAL PARTIES IN INTEREST; The basic rules of proper pleading and procedure require that every pleading shall contain
in a methodical and logical form, a plain, concise and direct statement of the ultimate
C. THAT THE COURT OF APPEALS ERRED IN APPLYING THE facts on which the party pleading relies for his claim or defense, as the case may be,
DOCTRINE OF "EXHAUSTION OF ADMINISTRATIVE REMEDIES"; and, omitting the statement of mere evidentiary facts.22 And in all averments of fraud or
mistake, the circumstances constituting fraud or mistake must be stated with
particularity.23
D. THAT THE COURT OF APPEALS ACTED WITH GRAVE ABUSE OF
DISCRETION AND DENIED PETITIONERS RIGHT TO DUE PROCESS
WHEN IT DISMISSED THEIR COMPLAINT.17 It is axiomatic that the averments of the complaint determine the nature of the
action, and consequently, the jurisdiction of the courts. This is because the
complaint must contain a concise statement of the ultimate facts constituting
We deny the petition. the plaintiff's cause of action and must specify the relief sought. No rule is
better established than that which requires the complaint to contain a statement
The subject lots are part of the Piedad Estate, Quezon City, a Friar Land acquired on of all the facts constituting the plaintiff's cause of action. Additionally, Section
December 23, 1903 by the Philippine Government from the Philippine Sugar Estates 5, Rule 8 of the Rules of Court provides that in all averments of fraud or
Development Company, Ltd., La Sociedad Agricola de Ultramar, the British-Manila mistake, the circumstances constituting fraud or mistake must be stated with
Estate Company, Ltd., and the Recoleto Order of the Philippine Islands, as indicated in particularity. In the case at bar, while there are allegations of fraud in the above
Public Act No. 1120 (Friar Lands Act) enacted on April 26, 1904. 18 quoted complaints, the same are not particular enough to bring the controversy
within the SEC's jurisdiction. The said allegations are not statements of
After the Piedad Estate was registered in OCT No. 614 in the name of the Philippine ultimate facts but are mere conclusions of law.
Government in 1910 under the provisions of Act 496, the area was subdivided originally
into 874 lots. As a result of subsequent surveys executed in the course of disposition, the A pleading should state the ultimate facts essential to the rights of action or
number of lots increased to 1,305. Disposition of these lots was made by the Bureau of defense asserted, as distinguished from mere conclusions of fact, or conclusions
Lands thru sales, under the Friar Lands Act, as early as 1910 and records show that even of law. General allegations that a contract is valid or legal, or is just, fair and
before the Second World War, all lots in the Piedad Estate have been disposed of.19 The reasonable, are mere conclusions of law. Likewise, allegations that a contract is
Piedad Estate has long been segregated from the mass of the public domain and has void, voidable, invalid, illegal, ultra vires, or against public policy, without
become private land duly registered under the Torrens system following the procedure for stating facts showing its invalidity, are mere conclusions of law.24
the confirmation of private lands prescribed in Act 496. Thus the lands inside the Piedad
Estate are no longer lands of the public domain.20 "Ultimate facts" means the essential facts constituting the plaintiff's cause of action, or
such facts as are so essential that they cannot be stricken out without leaving the
One who acquires land under the Friar Lands Act, as well as his successors-in-interest, statement of the cause of action inadequate.25"Cause of action" has been defined as an act
may not claim successional rights to purchase by reason of occupation from time or omission of one party in violation of the legal right or rights of the other; 26 and its
immemorial, as this contravenes the historical fact that friar lands were bought by the essential elements are: 1) a right in favor of the plaintiff by whatever means and under
Government of the Philippine Islands, pursuant to an Act of Congress of the United whatever law it arises or is created; 2) an obligation on the part of the named defendant to
respect or not to violate such right; and 3) an act or omission on the part of the named Second, the Ad Hoc Committee of the then Ministry of Natural Resources, which was
defendant violative of the right of the plaintiff or constituting a breach of the obligation of specifically tasked to investigate the historical background of the Piedad Estate, found
defendant to the plaintiff for which the latter may maintain an action for recovery of that as early as the period prior to the Second World War, all lots in the Piedad Estate had
damages. If these elements are not extant, the complaint becomes vulnerable to a motion already been disposed of.
to dismiss on the ground of failure to state a cause of action.27 In the resolution of a
motion to dismiss based on failure to state a cause of action, only the facts alleged in the Third, the Piedad Estate has been placed under the Torrens system of land registration,
complaint as well as its annexes must be considered.28 The test in such case is whether a which means that all lots therein are titled.
court can render a valid judgment on the complaint based upon the facts alleged and
pursuant to the prayer therein.29
Fourth, as held in the Balicudiong case, one who acquires land under the Friar Lands Act,
as well as his successors-in-interest, may not claim successional rights to purchase by
Corollarily, the question of whether or not a complaint states a cause of action against a reason of occupation from time immemorial, which means that petitioners claimed actual,
defendant or the action is premature is one of law. The trial court can consider all the adverse, peaceful and continuous possession of the subject property is really of no
pleadings filed, including annexes, motions and the evidence on record. However in so moment unless it is shown that their predecessors-in-interest were actual settlers and
doing, the trial court does not rule on the truth or falsity of such documents. It merely occupants at the time said lands were acquired by the Government, and whose rights were
includes such documents in the hypothetical admission. Any review of a finding of lack not disregarded even though they were in occupation of the same before the government
of cause of action based on these documents would not involve a calibration of the acquired the land; yet, no period of time in relation to adverse possession is alleged.
probative value of such pieces of evidence but would only limit itself to the inquiry of
whether the law was properly applied given the facts and these supporting documents.
Therefore, what would inevitably arise from such a review are pure questions of law, and Petitioners Second Amended Complaint betrays no more than an incomplete narration of
not questions of fact. facts unsupported by documentary or other exhibits; the allegations therein partake of
conclusions of law unsupported by a particular averment of circumstances that will show
why or how such inferences or conclusions were arrived at. It is replete with sweeping
The trial court must likewise apply relevant statutes and jurisprudence in determining generalizations and inferences derived from facts that are not found therein. While there
whether the allegations in a complaint establish a cause of action. While it focuses on the are allegations of fraud upon the claim that the subject titles were fictitious, spurious and
complaint, a court clearly cannot disregard decisions material to the proper appreciation obtained under "mysterious circumstances," the same are not specific to bring the
of the questions before it. In resolving a motion to dismiss, every court must take controversy within the trial courts jurisdiction. There is no explanation or narration of
cognizance of decisions this Court has rendered because they are proper subjects of facts as would show why said titles are claimed to be fictitious or spurious, contrary to
mandatory judicial notice. The said decisions, more importantly, form part of the legal the requirement of the Rules that the circumstances constituting fraud must be stated with
system, and failure of any court to apply them shall constitute an abdication of its duty to particularity; otherwise, the allegation of fraud would simply be an unfounded conclusion
resolve a dispute in accordance with law, and shall be a ground for administrative action of law. In the absence of specific averments, the complaint is defective, for it presents no
against an inferior court magistrate.30 basis upon which the court should act, or for the defendant to meet it with an intelligent
answer.
Considering the foregoing, it is not difficult to see the need for particularity and incipient
substantiation in the petitioners Second Amended Complaint. As to the second issue raised, petitioners claim that they are bona fide occupants of the
subject property within the contemplation of the Friar Lands Act, having allegedly been
First, their initial claim that OCT 614 of which all the other subject titles are in actual, adverse, peaceful and continuous possession of the property, although it is not
derivatives is null and void, has been proven wrong. As has been held in Pinlac and stated for how long and since when. In their second amended complaint, they seek
other cases, OCT 614 did legally exist and was previously issued in the name of the judgment
Philippine Government in 1910 under the provisions of Act 496.
(4) Declaring the plaintiffs as bona fide occupants of the property in litigation In the case at bar, the plaintiffs own averments negate the existence of such
pursuant to the provisions of the Friar Lands Act and other existing laws. right, for it would appear therefrom that whatever right might have been
(Emphasis supplied) violated by the defendant belonged to the government, not to the plaintiff.
Plaintiff-appellant argues that although his complaint is captioned as one for
They do not pray to be declared owners of the subject property despite their alleged cancellation of title, he has nevertheless stated therein several causes of action
adverse possession but only to be adjudged as the "bona fide occupants" thereof. In based on his alleged rights of possession and ownership over the improvements,
other words, petitioners concede the States ownership of the property. on defendant-appellees alleged fraudulent acquisition of the land, and on the
damages allegedly incurred by him (plaintiff-appellant) in relation to the
improvements. These matters are merely ancillary to the central issue of
Being so, petitioners may not be considered the real parties in interest for the purpose of whether or not defendant-appellees title should be canceled or amended, and
maintaining the suit for cancellation of the subject titles. The Court of Appeals is correct they may not be leaned upon in an effort to make out a cause of action in
in declaring that only the State, through the Solicitor General, may institute such suit. relation to the said focal issue. Indeed, the principal relief prayed for in the
Jurisprudence on the matter has been settled and the issue need not be belabored. Thus amended complaint is the cancellation or amendment of defendant-appellees
title."31
The Court also holds that private respondents are not the proper parties to
initiate the present suit. The complaint, praying as it did for the cancellation of Under Rule 3, Section 2 of the Rules of Court, a real party in interest is the party who
the transfer certificates of title of petitioners on the ground that they were stands to be benefited or injured by the judgment in the suit, or the party entitled to the
derived from a "spurious" OCT No. 4216, assailed in effect the validity of said avails of the suit. "Interest" within the meaning of the rule means material interest, an
title. While private respondents did not pray for the reversion of the land to the interest in issue and to be affected by the decree, as distinguished from mere interest in
government, we agree with the petitioners that the prayer in the complaint will the question involved, or a mere incidental interest. The interest of the party must also be
have the same result of reverting the land to the government under the Regalian personal and not one based on a desire to vindicate the constitutional right of some third
doctrine. Gabila vs. Barriga ruled that only the government is entitled to this and unrelated party. Real interest, on the other hand, means a present substantial interest,
relief. The Court in that case held: as distinguished from a mere expectancy or a future, contingent, subordinate, or
consequential interest.32
"The present motion to dismiss is actually predicated on Section 1(g),
Rule 16 of the Revised Rules of Court, i.e., failure of the complaint to If petitioners are to be believed, they would possess a mere inchoate interest in the
state a cause of action, for it alleges in paragraph 12 thereof that the properties covered by the subject titles, a mere expectancy conditioned upon the fact that
plaintiff admits that he has no right to demand the cancellation or if the questioned titles are cancelled and the property is reverted to the State, they
amendment of the defendants title, because, even if the said title would probably or possibly be given preferential treatment as qualified buyers or lessees
were canceled or amended, the ownership of the land embraced of the property under the Friar Lands Act. But this certainly is not the "interest" required
therein, or of the portion thereof affected by the amendment, would by law that grants them license or the personality to prosecute their case. Only to the
revert to the public domain. In his amended complaint the plaintiff State does the privilege belong.
makes no pretense at all that any part of the land covered by the
defendants title was privately owned by him or by his predecessors-
in-interest. Indeed, it is admitted therein that the said land was at all On the issue of exhaustion of administrative remedies, suffice it to state that since
times a part of the public domain until December 18, 1964, when the petitioners do not possess the necessary interest to prosecute the case for cancellation of
government issued a title thereon in favor of defendant. Thus, if there title in the courts, neither do they have the right to pursue administrative remedies outside
is any person or entity to relief, it can only be the government. thereof. They are not the owners; nor are they qualified applicants therefor. It has not
been shown by their complaint that they have previously taken steps to avail of the
benefits under the Friar Lands Act, since all they seek, should the questioned titles be
nullified, is to be declared bona fide occupants of the property covered by the questioned
titles. Neither is there any indication that they possess the qualifications necessary to
enable them to avail of the preference granted under the Act.

Finally, there is no merit in petitioners contention that respondent belatedly filed the
petition for certiorari with the Court of Appeals, and that the appellate court gravely
abused its discretion when it entertained and resolved the same.

The Order of the trial court dated January 3, 2001 denying respondents motion to
dismiss the Second Amended Complaint was received by the respondent on January 16,
2001. Respondent filed a motion for reconsideration on January 18, 2001 which was
denied on February 28, 2001. Respondent received the order denying its motion for
reconsideration on March 27, 2001. On the same day, it filed a Notice to File Petition for
Certiorari. On April 2, 2001, the petition for certiorari was filed with the Court of
Appeals. Clearly, the same was timely filed hence, the appellate court correctly
entertained the same.

WHEREFORE, the petition is DENIED. The Decision of the Court of Appeals dated
January 9, 2002 in CA-G.R. SP No. 64337 dismissing petitioners "Second Amended
Complaint" in Civil Case No. Q-99-36483 and the Resolution dated June 26, 2002
denying the motion for reconsideration, are AFFIRMED.

SO ORDERED.

Austria-Martinez, Corona * , Nachura, Reyes, JJ., concur.


Republic of the Philippines To prove that the property was an alienable and disposable land of the public domain,
SUPREME COURT Malabanan presented during trial a certification dated June 11, 2001 issued by the
Manila Community Environment and Natural Resources Office (CENRO) of the Department of
Environment and Natural Resources (DENR), which reads:
EN BANC
This is to certify that the parcel of land designated as Lot No. 9864 Cad 452-D, Silang
G.R. No. 179987 September 3, 2013 Cadastre as surveyed for Mr. Virgilio Velasco located at Barangay Tibig, Silang, Cavite
containing an area of 249,734 sq. meters as shown and described on the Plan Ap-04-
00952 is verified to be within the Alienable or Disposable land per Land Classification
HEIRS OF MARIO MALABANAN, (Represented by Sally A. Map No. 3013 established under Project No. 20-A and approved as such under FAO 4-
Malabanan), Petitioners, 1656 on March 15, 1982.2
vs.
REPUBLIC OF THE PHILIPPINES, Respondent.
After trial, on December 3, 2002, the RTC rendered judgment granting Malabanans
application for land registration, disposing thusly:
RESOLUTION
WHEREFORE, this Court hereby approves this application for registration and thus
BERSAMIN, J.: places under the operation of Act 141, Act 496 and/or P.D. 1529, otherwise known as
Property Registration Law, the lands described in Plan Csd-04-0173123-D, Lot 9864-A
For our consideration and resolution are the motions for reconsideration of the parties and containing an area of Seventy One Thousand Three Hundred Twenty Four (71,324)
who both assail the decision promulgated on April 29, 2009, whereby we upheld the Square Meters, as supported by its technical description now forming part of the record of
ruling of the Court of Appeals (CA) denying the application of the petitioners for the this case, in addition to other proofs adduced in the name of MARIO MALABANAN,
registration of a parcel of land situated in Barangay Tibig, Silang, Cavite on the ground who is of legal age, Filipino, widower, and with residence at Munting Ilog, Silang, Cavite.
that they had not established by sufficient evidence their right to the registration in
accordance with either Section 14(1) or Section 14(2) of Presidential Decree No. 1529 Once this Decision becomes final and executory, the corresponding decree of registration
(Property Registration Decree). shall forthwith issue.

Antecedents SO ORDERED.3

The property subject of the application for registration is a parcel of land situated in The Office of the Solicitor General (OSG) appealed the judgment to the CA, arguing that
Barangay Tibig, Silang Cavite, more particularly identified as Lot 9864-A, Cad-452-D, Malabanan had failed to prove that the property belonged to the alienable and disposable
with an area of 71,324-square meters. On February 20, 1998, applicant Mario Malabanan, land of the public domain, and that the RTC erred in finding that he had been in
who had purchased the property from Eduardo Velazco, filed an application for land possession of the property in the manner and for the length of time required by law for
registration covering the property in the Regional Trial Court (RTC) in Tagaytay City, confirmation of imperfect title.
Cavite, claiming that the property formed part of the alienable and disposable land of the
public domain, and that he and his predecessors-in-interest had been in open, continuous,
uninterrupted, public and adverse possession and occupation of the land for more than 30 On February 23, 2007, the CA promulgated its decision reversing the RTC and
years, thereby entitling him to the judicial confirmation of his title.1 dismissing the application for registration of Malabanan. Citing the ruling in Republic v.
Herbieto (Herbieto),4 the CA declared that under Section 14(1) of the Property
Registration Decree, any period of possession prior to the classification of the land as
alienable and disposable was inconsequential and should be excluded from the Arlos,7 Menguito v. Republic8 and Republic v. T.A.N. Properties, Inc.,9 they argue that
computation of the period of possession. Noting that the CENRO-DENR certification the reclassification of the land as alienable or disposable opened it to acquisitive
stated that the property had been declared alienable and disposable only on March 15, prescription under the Civil Code; that Malabanan had purchased the property from
1982, Velazcos possession prior to March 15, 1982 could not be tacked for purposes of Eduardo Velazco believing in good faith that Velazco and his predecessors-in-interest
computing Malabanans period of possession. had been the real owners of the land with the right to validly transmit title and ownership
thereof; that consequently, the ten-year period prescribed by Article 1134 of the Civil
Due to Malabanans intervening demise during the appeal in the CA, his heirs elevated Code, in relation to Section 14(2) of the Property Registration Decree, applied in their
the CAs decision of February 23, 2007 to this Court through a petition for review on favor; and that when Malabanan filed the application for registration on February 20,
certiorari. 1998, he had already been in possession of the land for almost 16 years reckoned from
1982, the time when the land was declared alienable and disposable by the State.
The petitioners assert that the ruling in Republic v. Court of Appeals and Corazon
Naguit5 (Naguit) remains the controlling doctrine especially if the property involved is The Republics Motion for Partial Reconsideration
agricultural land. In this regard, Naguit ruled that any possession of agricultural land prior
to its declaration as alienable and disposable could be counted in the reckoning of the The Republic seeks the partial reconsideration in order to obtain a clarification with
period of possession to perfect title under the Public Land Act (Commonwealth Act No. reference to the application of the rulings in Naguit and Herbieto.
141) and the Property Registration Decree. They point out that the ruling in Herbieto, to
the effect that the declaration of the land subject of the application for registration as Chiefly citing the dissents, the Republic contends that the decision has enlarged, by
alienable and disposable should also date back to June 12, 1945 or earlier, was a mere implication, the interpretation of Section 14(1) of the Property Registration Decree
obiter dictum considering that the land registration proceedings therein were in fact found through judicial legislation. It reiterates its view that an applicant is entitled to
and declared void ab initio for lack of publication of the notice of initial hearing. registration only when the land subject of the application had been declared alienable and
disposable since June 12, 1945 or earlier.
The petitioners also rely on the ruling in Republic v. T.A.N. Properties, Inc.6 to support
their argument that the property had been ipso jure converted into private property by Ruling
reason of the open, continuous, exclusive and notorious possession by their predecessors-
in-interest of an alienable land of the public domain for more than 30 years. According to
them, what was essential was that the property had been "converted" into private property We deny the motions for reconsideration.
through prescription at the time of the application without regard to whether the property
sought to be registered was previously classified as agricultural land of the public domain. In reviewing the assailed decision, we consider to be imperative to discuss the different
classifications of land in relation to the existing applicable land registration laws of the
As earlier stated, we denied the petition for review on certiorari because Malabanan failed Philippines.
to establish by sufficient evidence possession and occupation of the property on his part
and on the part of his predecessors-in interest since June 12, 1945, or earlier. Classifications of land according to ownership

Petitioners Motion for Reconsideration Land, which is an immovable property,10 may be classified as either of public dominion
or of private ownership.11Land is considered of public dominion if it either: (a) is
In their motion for reconsideration, the petitioners submit that the mere classification of intended for public use; or (b) belongs to the State, without being for public use, and is
the land as alienable or disposable should be deemed sufficient to convert it into intended for some public service or for the development of the national wealth. 12 Land
patrimonial property of the State. Relying on the rulings in Spouses De Ocampo v. belonging to the State that is not of such character, or although of such character but no
longer intended for public use or for public service forms part of the patrimonial property
of the State.13 Land that is other than part of the patrimonial property of the State, agricultural. Consequently, lands classified as forest or timber, mineral, or national parks
provinces, cities and municipalities is of private ownership if it belongs to a private are not susceptible of alienation or disposition unless they are reclassified as
individual. agricultural.24 A positive act of the Government is necessary to enable such
reclassification,25 and the exclusive prerogative to classify public lands under existing
Pursuant to the Regalian Doctrine (Jura Regalia), a legal concept first introduced into the laws is vested in the Executive Department, not in the courts. 26 If, however, public land
country from the West by Spain through the Laws of the Indies and the Royal will be classified as neither agricultural, forest or timber, mineral or national park, or
Cedulas,14 all lands of the public domain belong to the State.15 This means that the State when public land is no longer intended for public service or for the development of the
is the source of any asserted right to ownership of land, and is charged with the national wealth, thereby effectively removing the land from the ambit of public dominion,
conservation of such patrimony.16 a declaration of such conversion must be made in the form of a law duly enacted by
Congress or by a Presidential proclamation in cases where the President is duly
authorized by law to that effect.27 Thus, until the Executive Department exercises its
All lands not appearing to be clearly under private ownership are presumed to belong to prerogative to classify or reclassify lands, or until Congress or the President declares that
the State. Also, public lands remain part of the inalienable land of the public domain the State no longer intends the land to be used for public service or for the development
unless the State is shown to have reclassified or alienated them to private persons.17 of national wealth, the Regalian Doctrine is applicable.

Classifications of public lands Disposition of alienable public lands


according to alienability
Section 11 of the Public Land Act (CA No. 141) provides the manner by which alienable
Whether or not land of the public domain is alienable and disposable primarily rests on and disposable lands of the public domain, i.e., agricultural lands, can be disposed of, to
the classification of public lands made under the Constitution. Under the 1935 wit:
Constitution,18 lands of the public domain were classified into three, namely, agricultural,
timber and mineral.19 Section 10, Article XIV of the 1973 Constitution classified lands of
the public domain into seven, specifically, agricultural, industrial or commercial, Section 11. Public lands suitable for agricultural purposes can be disposed of only as
residential, resettlement, mineral, timber or forest, and grazing land, with the reservation follows, and not otherwise:
that the law might provide other classifications. The 1987 Constitution adopted the
classification under the 1935 Constitution into agricultural, forest or timber, and mineral, (1) For homestead settlement;
but added national parks.20 Agricultural lands may be further classified by law according
to the uses to which they may be devoted.21 The identification of lands according to their (2) By sale;
legal classification is done exclusively by and through a positive act of the Executive
Department.22
(3) By lease; and
Based on the foregoing, the Constitution places a limit on the type of public land that may
be alienated. Under Section 2, Article XII of the 1987 Constitution, only agricultural (4) By confirmation of imperfect or incomplete titles;
lands of the public domain may be alienated; all other natural resources may not be.
(a) By judicial legalization; or
Alienable and disposable lands of the State fall into two categories, to wit: (a) patrimonial
lands of the State, or those classified as lands of private ownership under Article 425 of (b) By administrative legalization (free patent).
the Civil Code,23 without limitation; and (b) lands of the public domain, or the public
lands as provided by the Constitution, but with the limitation that the lands must only be
The core of the controversy herein lies in the proper interpretation of Section 11(4), in 3. The possession and occupation must be under a bona fide claim of
relation to Section 48(b) of the Public Land Act, which expressly requires possession by a acquisition of ownership;
Filipino citizen of the land since June 12, 1945, or earlier, viz:
4. The possession and occupation must have taken place since June 12, 1945, or
Section 48. The following-described citizens of the Philippines, occupying lands of the earlier; and
public domain or claiming to own any such lands or an interest therein, but whose titles
have not been perfected or completed, may apply to the Court of First Instance of the 5. The property subject of the application must be an agricultural land of the
province where the land is located for confirmation of their claims and the issuance of a public domain.
certificate of title thereafter, under the Land Registration Act, to wit:
Taking into consideration that the Executive Department is vested with the authority to
xxxx classify lands of the public domain, Section 48(b) of the Public Land Act, in relation to
Section 14(1) of the Property Registration Decree, presupposes that the land subject of
(b) Those who by themselves or through their predecessors-in-interest have been in open, the application for registration must have been already classified as agricultural land of
continuous, exclusive, and notorious possession and occupation of alienable and the public domain in order for the provision to apply. Thus, absent proof that the land is
disposable lands of the public domain, under a bona fide claim of acquisition of already classified as agricultural land of the public domain, the Regalian Doctrine applies,
ownership, since June 12, 1945, or earlier, immediately preceding the filing of the and overcomes the presumption that the land is alienable and disposable as laid down in
applications for confirmation of title, except when prevented by war or force majeure. Section 48(b) of the Public Land Act. However, emphasis is placed on the requirement
These shall be conclusively presumed to have performed all the conditions essential to a that the classification required by Section 48(b) of the Public Land Act is classification or
Government grant and shall be entitled to a certificate of title under the provisions of this reclassification of a public land as agricultural.
chapter. (Bold emphasis supplied)
The dissent stresses that the classification or reclassification of the land as alienable and
Note that Section 48(b) of the Public Land Act used the words "lands of the public disposable agricultural land should likewise have been made on June 12, 1945 or earlier,
domain" or "alienable and disposable lands of the public domain" to clearly signify that because any possession of the land prior to such classification or reclassification
lands otherwise classified, i.e., mineral, forest or timber, or national parks, and lands of produced no legal effects. It observes that the fixed date of June 12, 1945 could not be
patrimonial or private ownership, are outside the coverage of the Public Land Act. What minimized or glossed over by mere judicial interpretation or by judicial social policy
the law does not include, it excludes. The use of the descriptive phrase "alienable and concerns, and insisted that the full legislative intent be respected.
disposable" further limits the coverage of Section 48(b) to only the agricultural lands of
the public domain as set forth in Article XII, Section 2 of the 1987 Constitution. Bearing We find, however, that the choice of June 12, 1945 as the reckoning point of the requisite
in mind such limitations under the Public Land Act, the applicant must satisfy the possession and occupation was the sole prerogative of Congress, the determination of
following requirements in order for his application to come under Section 14(1) of the which should best be left to the wisdom of the lawmakers. Except that said date qualified
Property Registration Decree,28 to wit: the period of possession and occupation, no other legislative intent appears to be
associated with the fixing of the date of June 12, 1945. Accordingly, the Court should
1. The applicant, by himself or through his predecessor-in-interest, has been in interpret only the plain and literal meaning of the law as written by the legislators.
possession and occupation of the property subject of the application;
Moreover, an examination of Section 48(b) of the Public Land Act indicates that
2. The possession and occupation must be open, continuous, exclusive, and Congress prescribed no requirement that the land subject of the registration should have
notorious; been classified as agricultural since June 12, 1945, or earlier. As such, the applicants
imperfect or incomplete title is derived only from possession and occupation since June
12, 1945, or earlier. This means that the character of the property subject of the On the other hand, if a public land is classified as no longer intended for public use or for
application as alienable and disposable agricultural land of the public domain determines the development of national wealth by declaration of Congress or the President, thereby
its eligibility for land registration, not the ownership or title over it. converting such land into patrimonial or private land of the State, the applicable provision
concerning disposition and registration is no longer Section 48(b) of the Public Land Act
Alienable public land held by a possessor, either personally or through his predecessors- but the Civil Code, in conjunction with Section 14(2) of the Property Registration
in-interest, openly, continuously and exclusively during the prescribed statutory period is Decree.35 As such, prescription can now run against the State.
converted to private property by the mere lapse or completion of the period. 29 In fact, by
virtue of this doctrine, corporations may now acquire lands of the public domain for as To sum up, we now observe the following rules relative to the disposition of public land
long as the lands were already converted to private ownership, by operation of law, as a or lands of the public domain, namely:
result of satisfying the requisite period of possession prescribed by the Public Land
Act.30 It is for this reason that the property subject of the application of Malabanan need (1) As a general rule and pursuant to the Regalian Doctrine, all lands of the
not be classified as alienable and disposable agricultural land of the public domain for the public domain belong to the State and are inalienable. Lands that are not clearly
entire duration of the requisite period of possession. under private ownership are also presumed to belong to the State and, therefore,
may not be alienated or disposed;
To be clear, then, the requirement that the land should have been classified as alienable
and disposable agricultural land at the time of the application for registration is necessary (2) The following are excepted from the general rule, to wit:
only to dispute the presumption that the land is inalienable.
(a) Agricultural lands of the public domain are rendered alienable and
The declaration that land is alienable and disposable also serves to determine the point at disposable through any of the exclusive modes enumerated under
which prescription may run against the State. The imperfect or incomplete title being Section 11 of the Public Land Act. If the mode is judicial
confirmed under Section 48(b) of the Public Land Act is title that is acquired by reason of confirmation of imperfect title under Section 48(b) of the Public Land
the applicants possession and occupation of the alienable and disposable agricultural Act, the agricultural land subject of the application needs only to be
land of the public domain. Where all the necessary requirements for a grant by the classified as alienable and disposable as of the time of the application,
Government are complied with through actual physical, open, continuous, exclusive and provided the applicants possession and occupation of the land dated
public possession of an alienable and disposable land of the public domain, the possessor back to June 12, 1945, or earlier. Thereby, a conclusive presumption
is deemed to have acquired by operation of law not only a right to a grant, but a grant by that the applicant has performed all the conditions essential to a
the Government, because it is not necessary that a certificate of title be issued in order government grant arises,36 and the applicant becomes the owner of
that such a grant be sanctioned by the courts.31 the land by virtue of an imperfect or incomplete title. By legal fiction,
the land has already ceased to be part of the public domain and has
If one follows the dissent, the clear objective of the Public Land Act to adjudicate and become private property.37
quiet titles to unregistered lands in favor of qualified Filipino citizens by reason of their
occupation and cultivation thereof for the number of years prescribed by law32 will be (b) Lands of the public domain subsequently classified or declared as
defeated. Indeed, we should always bear in mind that such objective still prevails, as a no longer intended for public use or for the development of national
fairly recent legislative development bears out, when Congress enacted legislation wealth are removed from the sphere of public dominion and are
(Republic Act No. 10023)33 in order to liberalize stringent requirements and procedures considered converted into patrimonial lands or lands of private
in the adjudication of alienable public land to qualified applicants, particularly residential ownership that may be alienated or disposed through any of the
lands, subject to area limitations.34 modes of acquiring ownership under the Civil Code. If the mode of
acquisition is prescription, whether ordinary or extraordinary, proof
that the land has been already converted to private ownership prior to
the requisite acquisitive prescriptive period is a condition sine qua
non in observance of the law (Article 1113, Civil Code) that property
of the State not patrimonial in character shall not be the object of
prescription.

To reiterate, then, the petitioners failed to present sufficient evidence to establish that
they and their predecessors-in-interest had been in possession of the land since June 12,
1945. Without satisfying the requisite character and period of possession - possession and
occupation that is open, continuous, exclusive, and notorious since June 12, 1945, or
earlier - the land cannot be considered ipso jure converted to private property even upon
the subsequent declaration of it as alienable and disposable. Prescription never began to
run against the State, such that the land has remained ineligible for registration under
Section 14(1) of the Property Registration Decree. Likewise, the land continues to be
ineligible for land registration under Section 14(2) of the Property Registration Decree
unless Congress enacts a law or the President issues a proclamation declaring the land as
no longer intended for public service or for the development of the national
wealth.1wphi1

WHEREFORE, the Court DENIES the petitioners' Motion for Reconsideration and the
respondent's Partial Motion for Reconsideration for their lack of merit.

SO ORDERED.

LUCAS P. BERSAMIN
Associate Justice

WE CONCUR:

MARIA LOURDES P. A.

You might also like